Π€ΠΎΡ€ΠΌΡƒΠ»Π° сопротивлСния участка Ρ†Π΅ΠΏΠΈ: Π—Π°ΠΊΠΎΠ½ Ома для участка Ρ†Π΅ΠΏΠΈ: Ρ„ΠΎΡ€ΠΌΡƒΠ»ΠΈΡ€ΠΎΠ²ΠΊΠ° ΠΈ Ρ„ΠΎΡ€ΠΌΡƒΠ»Π°, ΠΏΡ€ΠΈΠΌΠ΅Π½Π΅Π½ΠΈΠ΅

Π‘ΠΎΠ΄Π΅Ρ€ΠΆΠ°Π½ΠΈΠ΅

Π—Π°ΠΊΠΎΠ½ Ома для участка Ρ†Π΅ΠΏΠΈ: Ρ„ΠΎΡ€ΠΌΡƒΠ»ΠΈΡ€ΠΎΠ²ΠΊΠ° ΠΈ Ρ„ΠΎΡ€ΠΌΡƒΠ»Π°, ΠΏΡ€ΠΈΠΌΠ΅Π½Π΅Π½ΠΈΠ΅

Β 

ΠžΡ‚ силы Ρ‚ΠΎΠΊΠ° Π² Ρ†Π΅ΠΏΠΈ зависит Π²Π΅Π»ΠΈΡ‡ΠΈΠ½Π° воздСйствия, ΠΊΠΎΡ‚ΠΎΡ€ΠΎΠ΅ Ρ‚ΠΎΠΊ ΠΌΠΎΠΆΠ΅Ρ‚ ΠΎΠΊΠ°Π·Ρ‹Π²Π°Ρ‚ΡŒ Π½Π° ΠΏΡ€ΠΎΠ²ΠΎΠ΄Π½ΠΈΠΊ, Π±ΡƒΠ΄ΡŒ Ρ‚ΠΎ Ρ‚Π΅ΠΏΠ»ΠΎΠ²ΠΎΠ΅, химичСскоС ΠΈΠ»ΠΈ ΠΌΠ°Π³Π½ΠΈΡ‚Π½ΠΎΠ΅ дСйствиС Ρ‚ΠΎΠΊΠ°. Π’ΠΎ Π΅ΡΡ‚ΡŒ, рСгулируя силу Ρ‚ΠΎΠΊΠ°, ΠΌΠΎΠΆΠ½ΠΎ ΡƒΠΏΡ€Π°Π²Π»ΡΡ‚ΡŒ Π΅Π³ΠΎ воздСйствиСм. ЭлСктричСский Ρ‚ΠΎΠΊ, Π² свою ΠΎΡ‡Π΅Ρ€Π΅Π΄ΡŒ – это упорядочСнноС Π΄Π²ΠΈΠΆΠ΅Π½ΠΈΠ΅ частиц ΠΏΠΎΠ΄ дСйствиСм элСктричСского поля.

Π—Π°Π²ΠΈΡΠΈΠΌΠΎΡΡ‚ΡŒ силы Ρ‚ΠΎΠΊΠ° ΠΈ напряТСния

ΠžΡ‡Π΅Π²ΠΈΠ΄Π½ΠΎ, Ρ‡Ρ‚ΠΎ Ρ‡Π΅ΠΌ сильнСС ΠΏΠΎΠ»Π΅ дСйствуСт Π½Π° частицы, Ρ‚Π΅ΠΌ большС Π±ΡƒΠ΄Π΅Ρ‚ сила Ρ‚ΠΎΠΊΠ° Π² Ρ†Π΅ΠΏΠΈ. ЭлСктричСскоС ΠΏΠΎΠ»Π΅ характСризуСтся Π²Π΅Π»ΠΈΡ‡ΠΈΠ½ΠΎΠΉ, Π½Π°Π·Ρ‹Π²Π°Π΅ΠΌΠΎΠΉ напряТСниСм. Π‘Π»Π΅Π΄ΠΎΠ²Π°Ρ‚Π΅Π»ΡŒΠ½ΠΎ, ΠΌΡ‹ ΠΏΡ€ΠΈΡ…ΠΎΠ΄ΠΈΡ‚ ΠΊ Π²Ρ‹Π²ΠΎΠ΄Ρƒ, Ρ‡Ρ‚ΠΎ сила Ρ‚ΠΎΠΊΠ° зависит ΠΎΡ‚ напряТСния.

И Π΄Π΅ΠΉΡΡ‚Π²ΠΈΡ‚Π΅Π»ΡŒΠ½ΠΎ, ΠΎΠΏΡ‹Ρ‚Π½Ρ‹ΠΌ ΠΏΡƒΡ‚Π΅ΠΌ ΡƒΠ΄Π°Π»ΠΎΡΡŒ ΡƒΡΡ‚Π°Π½ΠΎΠ²ΠΈΡ‚ΡŒ, Ρ‡Ρ‚ΠΎ сила Ρ‚ΠΎΠΊΠ° связана с напряТСниСм прямо ΠΏΡ€ΠΎΠΏΠΎΡ€Ρ†ΠΈΠΎΠ½Π°Π»ΡŒΠ½ΠΎ. Π’ случаях, ΠΊΠΎΠ³Π΄Π° измСняли Π²Π΅Π»ΠΈΡ‡ΠΈΠ½Ρƒ напряТСния Π² Ρ†Π΅ΠΏΠΈ, Π½Π΅ мСняя всСх ΠΎΡΡ‚Π°Π»ΡŒΠ½Ρ‹Ρ… ΠΏΠ°Ρ€Π°ΠΌΠ΅Ρ‚Ρ€ΠΎΠ², сила Ρ‚ΠΎΠΊΠ° возрастала ΠΈΠ»ΠΈ ΡƒΠΌΠ΅Π½ΡŒΡˆΠ°Π»Π°ΡΡŒ Π²ΠΎ ΡΡ‚ΠΎΠ»ΡŒΠΊΠΎ ΠΆΠ΅ Ρ€Π°Π·, Π²ΠΎ сколько мСняли напряТСниС.

Бвязь с сопротивлСниСм

Однако любая Ρ†Π΅ΠΏΡŒ ΠΈΠ»ΠΈ участок Ρ†Π΅ΠΏΠΈ Ρ…Π°Ρ€Π°ΠΊΡ‚Π΅Ρ€ΠΈΠ·ΡƒΡŽΡ‚ΡΡ Π΅Ρ‰Π΅ ΠΎΠ΄Π½ΠΎΠΉ Π½Π΅ΠΌΠ°Π»ΠΎΠ²Π°ΠΆΠ½ΠΎΠΉ Π²Π΅Π»ΠΈΡ‡ΠΈΠ½ΠΎΠΉ, Π½Π°Π·Ρ‹Π²Π°Π΅ΠΌΠΎΠΉ сопротивлСниСм элСктричСскому Ρ‚ΠΎΠΊΡƒ. Π‘ΠΎΠΏΡ€ΠΎΡ‚ΠΈΠ²Π»Π΅Π½ΠΈΠ΅ связано с силой Ρ‚ΠΎΠΊΠ° ΠΎΠ±Ρ€Π°Ρ‚Π½ΠΎ ΠΏΡ€ΠΎΠΏΠΎΡ€Ρ†ΠΈΠΎΠ½Π°Π»ΡŒΠ½ΠΎ. Если Π½Π° ΠΊΠ°ΠΊΠΎΠΌ-Π»ΠΈΠ±ΠΎ участкС Ρ†Π΅ΠΏΠΈ ΠΈΠ·ΠΌΠ΅Π½ΠΈΡ‚ΡŒ Π²Π΅Π»ΠΈΡ‡ΠΈΠ½Ρƒ сопротивлСния, Π½Π΅ мСняя напряТСния Π½Π° ΠΊΠΎΠ½Ρ†Π°Ρ… этого участка, сила Ρ‚ΠΎΠΊΠ° Ρ‚Π°ΠΊΠΆΠ΅ измСнится. ΠŸΡ€ΠΈΡ‡Π΅ΠΌ Ссли ΠΌΡ‹ ΡƒΠΌΠ΅Π½ΡŒΡˆΠΈΠΌ Π²Π΅Π»ΠΈΡ‡ΠΈΠ½Ρƒ сопротивлСния, Ρ‚ΠΎ сила Ρ‚ΠΎΠΊΠ° возрастСт Π²ΠΎ ΡΡ‚ΠΎΠ»ΡŒΠΊΠΎ ΠΆΠ΅ Ρ€Π°Π·. И, Π½Π°ΠΎΠ±ΠΎΡ€ΠΎΡ‚, ΠΏΡ€ΠΈ ΡƒΠ²Π΅Π»ΠΈΡ‡Π΅Π½ΠΈΠΈ сопротивлСния сила Ρ‚ΠΎΠΊΠ° ΠΏΡ€ΠΎΠΏΠΎΡ€Ρ†ΠΈΠΎΠ½Π°Π»ΡŒΠ½ΠΎ ΡƒΠΌΠ΅Π½ΡŒΡˆΠ°Π΅Ρ‚ΡΡ.

Π€ΠΎΡ€ΠΌΡƒΠ»Π° Π·Π°ΠΊΠΎΠ½Π° Ома для участка Ρ†Π΅ΠΏΠΈ

Бопоставив Π΄Π²Π΅ эти зависимости, ΠΌΠΎΠΆΠ½ΠΎ ΠΏΡ€ΠΈΠΉΡ‚ΠΈ ΠΊ Ρ‚Π°ΠΊΠΎΠΌΡƒ ΠΆΠ΅ Π²Ρ‹Π²ΠΎΠ΄Ρƒ, ΠΊ ΠΊΠΎΡ‚ΠΎΡ€ΠΎΠΌΡƒ ΠΏΡ€ΠΈΡˆΠ΅Π» Π½Π΅ΠΌΠ΅Ρ†ΠΊΠΈΠΉ ΡƒΡ‡Π΅Π½Ρ‹ΠΉ Π“Π΅ΠΎΡ€Π³ Ом Π² 1827 Π³. Он связал Π²ΠΎΠ΅Π΄ΠΈΠ½ΠΎ Ρ‚Ρ€ΠΈ Π²Ρ‹ΡˆΠ΅ΡƒΠΊΠ°Π·Π°Π½Π½Ρ‹Π΅ физичСскиС Π²Π΅Π»ΠΈΡ‡ΠΈΠ½Ρ‹ ΠΈ Π²Ρ‹Π²Π΅Π» Π·Π°ΠΊΠΎΠ½, ΠΊΠΎΡ‚ΠΎΡ€Ρ‹ΠΉ Π½Π°Π·Π²Π°Π»ΠΈ Π΅Π³ΠΎ ΠΈΠΌΠ΅Π½Π΅ΠΌ. Π—Π°ΠΊΠΎΠ½ Ома для участка Ρ†Π΅ΠΏΠΈ гласит:

Π‘ΠΈΠ»Π° Ρ‚ΠΎΠΊΠ° Π² участкС Ρ†Π΅ΠΏΠΈ прямо ΠΏΡ€ΠΎΠΏΠΎΡ€Ρ†ΠΈΠΎΠ½Π°Π»ΡŒΠ½Π° Π½Π°ΠΏΡ€ΡΠΆΠ΅Π½ΠΈΡŽ Π½Π° ΠΊΠΎΠ½Ρ†Π°Ρ… этого участка ΠΈ ΠΎΠ±Ρ€Π°Ρ‚Π½ΠΎ ΠΏΡ€ΠΎΠΏΠΎΡ€Ρ†ΠΈΠΎΠ½Π°Π»ΡŒΠ½Π° Π΅Π³ΠΎ ΡΠΎΠΏΡ€ΠΎΡ‚ΠΈΠ²Π»Π΅Π½ΠΈΡŽ.

I=U/R,

Π³Π΄Π΅ I – сила Ρ‚ΠΎΠΊΠ°,
U – напряТСниС,
R – сопротивлСниС.

ΠŸΡ€ΠΈΠΌΠ΅Π½Π΅Π½ΠΈΠ΅ Π·Π°ΠΊΠΎΠ½Π° Ома

Π—Π°ΠΊΠΎΠ½ Ома – ΠΎΠ΄ΠΈΠ½ ΠΈΠ· ΠΎΡΠ½ΠΎΠ²ΠΎΠΏΠΎΠ»Π°Π³Π°ΡŽΡ‰ΠΈΡ… Π·Π°ΠΊΠΎΠ½ΠΎΠ² Ρ„ΠΈΠ·ΠΈΠΊΠΈ. ΠžΡ‚ΠΊΡ€Ρ‹Ρ‚ΠΈΠ΅ Π΅Π³ΠΎ Π² своС врСмя ΠΏΠΎΠ·Π²ΠΎΠ»ΠΈΠ»ΠΎ ΡΠ΄Π΅Π»Π°Ρ‚ΡŒ ΠΎΠ³Ρ€ΠΎΠΌΠ½Ρ‹ΠΉ скачок Π² Π½Π°ΡƒΠΊΠ΅. Π’ настоящСС врСмя Π½Π΅Π²ΠΎΠ·ΠΌΠΎΠΆΠ½ΠΎ сСбС ΠΏΡ€Π΅Π΄ΡΡ‚Π°Π²ΠΈΡ‚ΡŒ любой самый элСмСнтарный расчСт основных элСктричСских Π²Π΅Π»ΠΈΡ‡ΠΈΠ½ для любой Ρ†Π΅ΠΏΠΈ Π±Π΅Π· использования Π·Π°ΠΊΠΎΠ½Π° Ома. ΠŸΡ€Π΅Π΄ΡΡ‚Π°Π²Π»Π΅Π½ΠΈΠ΅ ΠΎΠ± этом Π·Π°ΠΊΠΎΠ½Π΅ – это Π½Π΅ ΡƒΠ΄Π΅Π» ΠΈΡΠΊΠ»ΡŽΡ‡ΠΈΡ‚Π΅Π»ΡŒΠ½ΠΎ ΠΈΠ½ΠΆΠ΅Π½Π΅Ρ€ΠΎΠ²-элСктронщиков, Π° нСобходимая Ρ‡Π°ΡΡ‚ΡŒ Π±Π°Π·ΠΎΠ²Ρ‹Ρ… Π·Π½Π°Π½ΠΈΠΉ любого ΠΌΠ°Π»ΠΎ-мальски ΠΎΠ±Ρ€Π°Π·ΠΎΠ²Π°Π½Π½ΠΎΠ³ΠΎ Ρ‡Π΅Π»ΠΎΠ²Π΅ΠΊΠ°. НСдаром Π΅ΡΡ‚ΡŒ ΠΏΠΎΠ³ΠΎΠ²ΠΎΡ€ΠΊΠ°: «НС знаСшь Π·Π°ΠΊΠΎΠ½ Ома – сиди Π΄ΠΎΠΌΠ°Β».

Из Ρ„ΠΎΡ€ΠΌΡƒΠ»Ρ‹ для Π·Π°ΠΊΠΎΠ½Π° Ома ΠΌΠΎΠΆΠ½ΠΎ Ρ€Π°ΡΡΡ‡ΠΈΡ‚Π°Ρ‚ΡŒ Ρ‚Π°ΠΊΠΆΠ΅ Π²Π΅Π»ΠΈΡ‡ΠΈΠ½Ρ‹ напряТСния ΠΈ сопротивлСния участка Ρ†Π΅ΠΏΠΈ:

U=IRΒ  Β  ΠΈ Β  Β R=U/I

ΠŸΡ€Π°Π²Π΄Π°, слСдуСт ΠΏΠΎΠ½ΠΈΠΌΠ°Ρ‚ΡŒ, Ρ‡Ρ‚ΠΎ Π² собранной Ρ†Π΅ΠΏΠΈ Π²Π΅Π»ΠΈΡ‡ΠΈΠ½Π° сопротивлСния Π½Π΅ΠΊΠΎΡ‚ΠΎΡ€ΠΎΠ³ΠΎ участка Ρ†Π΅ΠΏΠΈ Π΅ΡΡ‚ΡŒ Π²Π΅Π»ΠΈΡ‡ΠΈΠ½Π° постоянная, поэтому ΠΏΡ€ΠΈ ΠΈΠ·ΠΌΠ΅Π½Π΅Π½ΠΈΠΈ силы Ρ‚ΠΎΠΊΠ° Π±ΡƒΠ΄Π΅Ρ‚ ΠΈΠ·ΠΌΠ΅Π½ΡΡ‚ΡŒΡΡ Ρ‚ΠΎΠ»ΡŒΠΊΠΎ напряТСниС ΠΈ Π½Π°ΠΎΠ±ΠΎΡ€ΠΎΡ‚. Для измСнСния сопротивлСния участка Ρ†Π΅ΠΏΠΈ слСдуСт ΡΠΎΠ±Ρ€Π°Ρ‚ΡŒ Ρ†Π΅ΠΏΡŒ Π·Π°Π½ΠΎΠ²ΠΎ. РасчСт ΠΆΠ΅ Ρ‚Ρ€Π΅Π±ΡƒΠ΅ΠΌΠΎΠΉ Π²Π΅Π»ΠΈΡ‡ΠΈΠ½Ρ‹ сопротивлСния ΠΏΡ€ΠΈ ΠΏΡ€ΠΎΠ΅ΠΊΡ‚ΠΈΡ€ΠΎΠ²Π°Π½ΠΈΠΈ ΠΈ сборкС Ρ†Π΅ΠΏΠΈ ΠΌΠΎΠΆΠ½ΠΎ произвСсти ΠΏΠΎ Π·Π°ΠΊΠΎΠ½Ρƒ Ома, исходя ΠΈΠ· ΠΏΡ€Π΅Π΄ΠΏΠΎΠ»Π°Π³Π°Π΅ΠΌΡ‹Ρ… Π·Π½Π°Ρ‡Π΅Π½ΠΈΠΉ силы Ρ‚ΠΎΠΊΠ° ΠΈ напряТСния, ΠΊΠΎΡ‚ΠΎΡ€Ρ‹Π΅ Π±ΡƒΠ΄ΡƒΡ‚ ΠΏΡ€ΠΎΠΏΡƒΡ‰Π΅Π½Ρ‹ Ρ‡Π΅Ρ€Π΅Π· Π΄Π°Π½Π½Ρ‹ΠΉ участок Ρ†Π΅ΠΏΠΈ.

НуТна ΠΏΠΎΠΌΠΎΡ‰ΡŒ Π² ΡƒΡ‡Π΅Π±Π΅?



ΠŸΡ€Π΅Π΄Ρ‹Π΄ΡƒΡ‰Π°Ρ Ρ‚Π΅ΠΌΠ°: Π‘ΠΎΠΏΡ€ΠΎΡ‚ΠΈΠ²Π»Π΅Π½ΠΈΠ΅ Ρ‚ΠΎΠΊΠ°: притяТСниС ядСр, ΠΏΡ€ΠΎΠ²ΠΎΠ΄Π½ΠΈΠΊΠΈ ΠΈ Π½Π΅ΠΏΡ€ΠΎΠ²ΠΎΠ΄Π½ΠΈΠΊΠΈ
Π‘Π»Π΅Π΄ΡƒΡŽΡ‰Π°Ρ Ρ‚Π΅ΠΌΠ°:&nbsp&nbsp&nbspРасчёт сопротивлСния ΠΏΡ€ΠΎΠ²ΠΎΠ΄Π½ΠΈΠΊΠΎΠ² ΠΈ рСостаты: Ρ„ΠΎΡ€ΠΌΡƒΠ»Ρ‹

ЭлСктричСскоС сопротивлСниС участка Ρ†Π΅ΠΏΠΈ

ЭлСктричСскоС сопротивлСниС для участка Ρ†Π΅ΠΏΠΈ опрСдСляСтся ΠΏΡ€ΠΈ ΠΏΠΎΠΌΠΎΡ‰ΠΈ Π·Π°ΠΊΠΎΠ½Π° Ома. Для Ρ‚ΠΎΠ³ΠΎ, Ρ‡Ρ‚ΠΎΠ±Ρ‹ ΠΏΠΎΠ½ΡΡ‚ΡŒ процСссы, происходящиС Π² элСмСнтах элСктричСской Ρ†Π΅ΠΏΠΈ постоянного Ρ‚ΠΎΠΊΠ°, Π½Π΅ΠΎΠ±Ρ…ΠΎΠ΄ΠΈΠΌΠΎ Π΄Π°Ρ‚ΡŒ ΠΎΠ±Ρ‰Π΅Π΅ ΠΎΠΏΡ€Π΅Π΄Π΅Π»Π΅Π½ΠΈΠ΅ Π·Π°ΠΊΠΎΠ½Π° Ома.

Π—Π°ΠΊΠΎΠ½ Ома

Π‘ΠΈΠ»Π° Ρ‚ΠΎΠΊΠ° Π½Π° участкС Ρ†Π΅ΠΏΠΈ всСгда прямо ΠΏΡ€ΠΎΠΏΠΎΡ€Ρ†ΠΈΠΎΠ½Π°Π»ΡŒΠ½Π° Π½Π°ΠΏΡ€ΡΠΆΠ΅Π½ΠΈΡŽ Π½Π° Π΄Π°Π½Π½ΠΎΠΌ участкС ΠΈ ΠΎΠ±Ρ€Π°Ρ‚Π½ΠΎ ΠΏΡ€ΠΎΠΏΠΎΡ€Ρ†ΠΈΠΎΠ½Π°Π»ΡŒΠ½Π° ΡΠΎΠΏΡ€ΠΎΡ‚ΠΈΠ²Π»Π΅Π½ΠΈΡŽ участка.

ПодобноС ΠΎΠΏΡ€Π΅Π΄Π΅Π»Π΅Π½ΠΈΠ΅ Π±ΡƒΠ΄Π΅Ρ‚ Π²Π΅Ρ€Π½ΠΎ Ρ‚Π°ΠΊΠΆΠ΅ для растворов элСктролитов. ΠžΠ±Ρ‰ΠΈΠΉ Π·Π°ΠΊΠΎΠ½ Ома Ρ…Π°Ρ€Π°ΠΊΡ‚Π΅Ρ€Π΅Π½ ΠΏΡ€ΠΈ описании ΠΎΠ΄Π½ΠΎΡ€ΠΎΠ΄Π½ΠΎΠ³ΠΎ участка Ρ†Π΅ΠΏΠΈ, ΠΊΠΎΡ‚ΠΎΡ€Ρ‹ΠΉ Π½Π΅ содСрТит источников Ρ‚ΠΎΠΊΠ°.

ΠŸΡ€ΠΈ составлСнии Ρ„ΠΎΡ€ΠΌΡƒΠ» вводятся Π΄ΠΎΠΏΠΎΠ»Π½ΠΈΡ‚Π΅Π»ΡŒΠ½Ρ‹Π΅ характСристики. Π‘Ρ€Π΅Π΄ΠΈ Π½ΠΈΡ… коэффициСнт ΠΏΡ€ΠΎΠΏΠΎΡ€Ρ†ΠΈΠΎΠ½Π°Π»ΡŒΠ½ΠΎΡΡ‚ΠΈ. Π•Π³ΠΎ Π·Π°ΠΏΠΈΡΡ‹Π²Π°ΡŽΡ‚ Π² Π²ΠΈΠ΄Π΅ $1=R$. ΠžΡ‚ΡΡŽΠ΄Π° слСдуСт, Ρ‡Ρ‚ΠΎ $I = \frac{U}{R}$.

$R$ – сопротивлСниС ΠΏΡ€ΠΎΠ²ΠΎΠ΄Π½ΠΈΠΊΠ°.

Π‘ΠΎΠΏΡ€ΠΎΡ‚ΠΈΠ²Π»Π΅Π½ΠΈΠ΅ принято ΠΈΠ·ΠΌΠ΅Ρ€ΡΡ‚ΡŒ Π² ΠΎΠΌΠ°Ρ… (Ом).

Π—Π°ΠΊΠΎΠ½ Ома являСтся Π³Π»Π°Π²Π½Ρ‹ΠΌ Π·Π°ΠΊΠΎΠ½ΠΎΠΌ Π² элСктротСхникС. Π‘ ΠΏΠΎΠΌΠΎΡ‰ΡŒΡŽ Π΅Π³ΠΎ:

  • ΠΈΠ·ΡƒΡ‡Π°ΡŽΡ‚ΡΡ ΠΈ Ρ€Π°ΡΡΡ‡ΠΈΡ‚Ρ‹Π²Π°ΡŽΡ‚ΡΡ элСктричСскиС Ρ†Π΅ΠΏΠΈ;
  • устанавливаСтся логичСскоС ΡΠΎΠΎΡ‚Π½ΠΎΡˆΠ΅Π½ΠΈΠ΅ ΠΌΠ΅ΠΆΠ΄Ρƒ сопротивлСниСм ΠΈ напряТСниСм.

ΠžΠΏΡ€Π΅Π΄Π΅Π»Π΅Π½ΠΈΠ΅ 1

Π’ΠΎΠ»ΡŒΡ‚Π°ΠΌΠΏΠ΅Ρ€Π½Π°Ρ характСристика – Ρ„ΡƒΠ½ΠΊΡ†ΠΈΠΎΠ½Π°Π»ΡŒΠ½Π°Ρ Π·Π°Π²ΠΈΡΠΈΠΌΠΎΡΡ‚ΡŒ элСмСнта участка Ρ†Π΅ΠΏΠΈ. Она являСтся ΠΎΡ‡Π΅Π½ΡŒ Π²Π°ΠΆΠ½ΠΎΠΉ Π²Π΅Π»ΠΈΡ‡ΠΈΠ½ΠΎΠΉ элСктричСских свойств элСмСнта. Π’Π°ΠΊΡƒΡŽ Π·Π°Π²ΠΈΡΠΈΠΌΠΎΡΡ‚ΡŒ ΠΌΠΎΠΆΠ½ΠΎ ΠΏΡ€Π΅Π΄ΡΡ‚Π°Π²ΠΈΡ‚ΡŒ Π² Π²ΠΈΠ΄Π΅ $I = I(U)$.

ΠŸΠΎΠ΄ΠΎΠ±Π½Ρ‹Π΅ характСристики Π² зависимости ΠΎΡ‚ ситуации ΠΌΠΎΠ³ΡƒΡ‚ ΠΏΡ€ΠΈΠΎΠ±Ρ€Π΅Ρ‚Π°Ρ‚ΡŒ Ρ€Π°Π·Π»ΠΈΡ‡Π½Ρ‹Π΅ Ρ„ΠΎΡ€ΠΌΡ‹ ΠΈ выраТСния. НаиболСС простой Π²ΠΈΠ΄ Π²ΠΎΠ»ΡŒΡ‚Π°ΠΌΠΏΠ΅Ρ€Π½ΠΎΠΉ характСристики Π²Ρ‹Ρ€Π°Π·ΠΈΠ» Π² Ρ„ΠΎΡ€ΠΌΡƒΠ»Π΅ Π“Π΅ΠΎΡ€Π³ Ом, Π² Ρ‡Π΅ΡΡ‚ΡŒ ΠΊΠΎΡ‚ΠΎΡ€ΠΎΠ³ΠΎ Π±Ρ‹Π»Π° Π½Π°Π·Π²Π°Π½Π° Π΅Π΄ΠΈΠ½ΠΈΡ†Π° сопротивлСния Ρ‚ΠΎΠΊΠ°. Π£Ρ‡Π΅Π½Ρ‹ΠΉ ΠΏΠΎΠ΄Ρ‚Π²Π΅Ρ€Π΄ΠΈΠ» свою Ρ‚Π΅ΠΎΡ€ΠΈΡŽ многочислСнными экспСримСнтами, примСняя ΠΎΠΏΡ‹Ρ‚Ρ‹ ΠΊ мСталличСскому ΠΏΡ€ΠΎΠ²ΠΎΠ΄Π½ΠΈΠΊΡƒ.

Π—Π°ΠΊΠΎΠ½ Ома Π½Π΅ΠΎΠ±Ρ…ΠΎΠ΄ΠΈΠΌΠΎ ΠΏΠΎΠ½ΠΈΠΌΠ°Ρ‚ΡŒ Π½Π° тСорСтичСском ΠΈ практичСском ΡƒΡ€ΠΎΠ²Π½Π΅, Ρ‡Ρ‚ΠΎΠ±Ρ‹ Ρ€Π΅ΡˆΠ°Ρ‚ΡŒ Ρ€Π°Π·Π»ΠΈΡ‡Π½Ρ‹Π΅ Π·Π°Π΄Π°Ρ‡ΠΈ. Если Π½Π΅ΠΏΡ€Π°Π²ΠΈΠ»ΡŒΠ½ΠΎ ΠΏΡ€ΠΈΠΌΠ΅Π½ΡΡ‚ΡŒ основныС ΠΏΠ°Ρ€Π°ΠΌΠ΅Ρ‚Ρ€Ρ‹ Π·Π°ΠΊΠΎΠ½Π°, Ρ‚ΠΎ Ρ€Π΅Π·ΡƒΠ»ΡŒΡ‚Π°Ρ‚ ΠΏΡ€ΠΈΠΎΠ±Ρ€Π΅Ρ‚Π°Π΅Ρ‚ Π½Π΅ΠΏΡ€Π°Π²ΠΈΠ»ΡŒΠ½Ρ‹Π΅ Ρ‡Π΅Ρ€Ρ‚Ρ‹, поэтому Π΄ΠΎΠΏΡƒΡΠΊΠ°ΡŽΡ‚ΡΡ многочислСнныС ошибки.

Π“ΠΎΡ‚ΠΎΠ²Ρ‹Π΅ Ρ€Π°Π±ΠΎΡ‚Ρ‹ Π½Π° Π°Π½Π°Π»ΠΎΠ³ΠΈΡ‡Π½ΡƒΡŽ Ρ‚Π΅ΠΌΡƒ

ΠŸΡ€ΠΈΠΌΠ΅Π½Π΅Π½ΠΈΠ΅ Π·Π°ΠΊΠΎΠ½Π° Ома для участка Ρ†Π΅ΠΏΠΈ

ΠšΠ°ΠΆΠ΄Ρ‹ΠΉ участок элСктричСской Ρ†Π΅ΠΏΠΈ ΠΌΠΎΠΆΠ½ΠΎ ΠΎΠΏΠΈΡΠ°Ρ‚ΡŒ с ΠΏΠΎΠΌΠΎΡ‰ΡŒΡŽ Ρ‚Ρ€Π΅Ρ… основных Π²Π΅Π»ΠΈΡ‡ΠΈΠ½:

  • сопротивлСния;
  • напряТСния;
  • Ρ‚ΠΎΠΊΠ°.

Π’Π°ΠΊΠΎΠ΅ сочСтаниС Ρ‚Π°ΠΊΠΆΠ΅ Π½Π°Π·Ρ‹Π²Π°ΡŽΡ‚ Β«Ρ‚Ρ€Π΅ΡƒΠ³ΠΎΠ»ΡŒΠ½ΠΈΠΊΠΎΠΌ Ома», ΠΏΠΎΡΠΊΠΎΠ»ΡŒΠΊΡƒ Π²Π΅Π»ΠΈΡ‡ΠΈΠ½Ρ‹ Ρ…Π°Ρ€Π°ΠΊΡ‚Π΅Ρ€ΠΈΠ·ΡƒΡŽΡ‚ всС процСссы элСктротСхники.

ВсС ΠΏΡ€ΠΎΠΈΠ·Π²ΠΎΠ΄ΠΈΠΌΡ‹Π΅ расчСты ΠΈΠΌΠ΅ΡŽΡ‚ смысл Ρ‚ΠΎΠ»ΡŒΠΊΠΎ Π² Ρ‚Π΅Ρ… случаях, ΠΊΠΎΠ³Π΄Π° напряТСниС Π½Π° участкС Ρ†Π΅ΠΏΠΈ выраТаСтся Π² Π²ΠΎΠ»ΡŒΡ‚Π°Ρ… (Π’), сопротивлСниС — Π² ΠΎΠΌΠ°Ρ… (Ом), Π° Ρ‚ΠΎΠΊ – Π² Π°ΠΌΠΏΠ΅Ρ€Π°Ρ… (А). ΠŸΡ€ΠΈ использовании ΠΈΠ½Ρ‹Ρ… Π΅Π΄ΠΈΠ½ΠΈΡ† ΠΈΠ·ΠΌΠ΅Ρ€Π΅Π½ΠΈΠΉ ΠΈΠ»ΠΈ ΠΈΡ… ΠΊΡ€Π°Ρ‚Π½Ρ‹Ρ… Π·Π½Π°Ρ‡Π΅Π½ΠΈΠΉ Π½Π΅ΠΎΠ±Ρ…ΠΎΠ΄ΠΈΠΌΠΎ ΠΎΡΡƒΡ‰Π΅ΡΡ‚Π²Π»ΡΡ‚ΡŒ Π΄ΠΎΠΏΠΎΠ»Π½ΠΈΡ‚Π΅Π»ΡŒΠ½Ρ‹ΠΉ ряд дСйствий, Ρ‡Ρ‚ΠΎΠ±Ρ‹ искомый Ρ€Π΅Π·ΡƒΠ»ΡŒΡ‚Π°Ρ‚ ΠΏΠΎΠ»Π½ΠΎΡΡ‚ΡŒΡŽ соотвСтствовал Π·Π°Π΄Π°Ρ‡Π°ΠΌ ΠΈ цСлям расчСтов. Для этого ΠΊΡ€Π°Ρ‚Π½Ρ‹Π΅ Π΅Π΄ΠΈΠ½ΠΈΡ†Ρ‹ ΠΈΡΠΏΠΎΠ»ΡŒΠ·ΡƒΠ΅ΠΌΡ‹Ρ… Π²Π΅Π»ΠΈΡ‡ΠΈΠ½ пСрСводят Π² Ρ‚Ρ€Π°Π΄ΠΈΡ†ΠΈΠΎΠ½Π½Ρ‹Π΅ Π²Π΅Π»ΠΈΡ‡ΠΈΠ½Ρ‹.

ΠšΡ€Π°Ρ‚Π½Ρ‹Π΅ Π΅Π΄ΠΈΠ½ΠΈΡ†Ρ‹ ΠΈΠ·ΠΌΠ΅Ρ€Π΅Π½ΠΈΠΉ:

  • ΠΌΠΈΠ»Π»ΠΈΠ²ΠΎΠ»ΡŒΡ‚Ρ‹;
  • ΠΌΠΈΠ»Π»ΠΈΠ°ΠΌΠΏΠ΅Ρ€Ρ‹;
  • ΠΌΠ΅Π³Π°ΠΎΠΌΡ‹.

ΠŸΡ€ΠΈ ΠΏΡ€ΠΎΠΈΠ·Π²Π΅Π΄Π΅Π½ΠΈΠΈ расчСтов Π² ΠΊΡ€Π°Ρ‚Π½Ρ‹Ρ… Π΅Π΄ΠΈΠ½ΠΈΡ†Π°Ρ… ΠΈΠ·ΠΌΠ΅Ρ€Π΅Π½ΠΈΠΉ Π²Π΅Π»ΠΈΡ‡ΠΈΠ½ напряТСниС всСгда выраТаСтся Π² Π²ΠΎΠ»ΡŒΡ‚Π°Ρ….

Для расчСта сопротивлСния Π½Π° участкС Ρ†Π΅ΠΏΠΈ ΠΏΠΎ Π·Π°ΠΊΠΎΠ½Ρƒ Ома Π½Π΅ΠΎΠ±Ρ…ΠΎΠ΄ΠΈΠΌΠΎ сначала ΠΎΠΏΡ€Π΅Π΄Π΅Π»ΠΈΡ‚ΡŒ Ρ‚ΠΎΠΊ Π½Π° Π·Π°Π΄Π°Π½Π½ΠΎΠΌ участкС Ρ†Π΅ΠΏΠΈ. НапряТСниС ΠΏΡ€ΠΈ этом дСлят Π½Π° сопротивлСниС ΠΊΠΎΠ½ΠΊΡ€Π΅Ρ‚Π½ΠΎΠ³ΠΎ участка Ρ†Π΅ΠΏΠΈ. Π­Ρ‚ΠΈ дСйствия ΠΌΠΎΠΆΠ½ΠΎ ΠΏΡ€ΠΎΠΈΠ·Π²ΠΎΠ΄ΠΈΡ‚ΡŒ Π½Π° любом участкС Π±Π΅Π· ΠΏΠΎΠ³Ρ€Π΅ΡˆΠ½ΠΎΡΡ‚ΠΈ.

Для опрСдСлСния напряТСния Π² Ρ†Π΅ΠΏΠΈ ΠΈΡΠΏΠΎΠ»ΡŒΠ·ΡƒΡŽΡ‚ Ρ„ΠΎΡ€ΠΌΡƒΠ»Ρƒ $U = IR$.

Богласно ΡƒΠΊΠ°Π·Π°Π½Π½ΠΎΠΉ Ρ„ΠΎΡ€ΠΌΡƒΠ»Π΅, напряТСниС Π½Π° ΠΎΠ±ΠΎΠΈΡ… ΠΊΠΎΠ½Ρ†Π°Ρ… участка элСктричСской Ρ†Π΅ΠΏΠΈ прямо ΠΏΡ€ΠΎΠΏΠΎΡ€Ρ†ΠΈΠΎΠ½Π°Π»ΡŒΠ½ΠΎ ΡΠΎΠΏΡ€ΠΎΡ‚ΠΈΠ²Π»Π΅Π½ΠΈΡŽ ΠΈ Ρ‚ΠΎΠΊΡƒ. Π˜Π½Ρ‹ΠΌΠΈ словами, Ссли Π½Π΅ ΡΡ‚Ρ€Π΅ΠΌΠΈΡ‚ΡŒΡΡ всС врСмя ΠΈΠ·ΠΌΠ΅Π½ΡΡ‚ΡŒ сопротивлСниС Π½Π° Π΄Π°Π½Π½ΠΎΠΌ участкС, Ρ‚ΠΎ ΠΏΡ€ΠΈ ΡƒΠ²Π΅Π»ΠΈΡ‡Π΅Π½ΠΈΠΈ Ρ‚ΠΎΠΊΠ° примСняСтся способ увСличСния напряТСния.

Π—Π½Π°Ρ‡ΠΈΡ‚Π΅Π»ΡŒΠ½ΠΎΠΌΡƒ Π½Π°ΠΏΡ€ΡΠΆΠ΅Π½ΠΈΡŽ Π² Ρ†Π΅ΠΏΠΈ Π±ΡƒΠ΄Π΅Ρ‚ ΡΠΎΠΎΡ‚Π²Π΅Ρ‚ΡΡ‚Π²ΠΎΠ²Π°Ρ‚ΡŒ больший Ρ‚ΠΎΠΊ. Π­Ρ‚ΠΈ ΠΏΡ€Π°Π²ΠΈΠ»Π° Π΄Π΅ΠΉΡΡ‚Π²ΡƒΡŽΡ‚ ΠΏΡ€ΠΈ постоянном сопротивлСнии. Для ΠΏΠΎΠ»ΡƒΡ‡Π΅Π½ΠΈΠΈ ΠΎΠ΄ΠΈΠ½Π°ΠΊΠΎΠ²ΠΎΠ³ΠΎ Ρ‚ΠΎΠΊΠ° ΠΏΡ€ΠΈ Ρ€Π°Π·Π»ΠΈΡ‡Π½Ρ‹Ρ… сопротивлСниях большСС напряТСниС Π΄ΠΎΠ»ΠΆΠ½ΠΎ ΡΠΎΠΎΡ‚Π²Π΅Ρ‚ΡΡ‚Π²ΠΎΠ²Π°Ρ‚ΡŒ Π±ΠΎΠ»ΡŒΡˆΠ΅ΠΌΡƒ ΡΠΎΠΏΡ€ΠΎΡ‚ΠΈΠ²Π»Π΅Π½ΠΈΡŽ.

ПадСниС напряТСния – это напряТСниС Π½Π° ΠΎΠΏΡ€Π΅Π΄Π΅Π»Π΅Π½Π½ΠΎΠΌ участкС Ρ†Π΅ΠΏΠΈ. Π­Ρ‚ΠΎ ΠΎΠ·Π½Π°Ρ‡Π°Π΅Ρ‚, Ρ‡Ρ‚ΠΎ напряТСниС ΠΈ ΠΏΠ°Π΄Π΅Π½ΠΈΠ΅ напряТСния – ΠΈΠ΄Π΅Π½Ρ‚ΠΈΡ‡Π½Ρ‹Π΅ понятия, Π° слово Β«ΠΏΠ°Π΄Π΅Π½ΠΈΠ΅Β» Π½ΠΈΠΊΠ°ΠΊ Π½Π΅ связано с ΠΏΠΎΡ‚Π΅Ρ€Π΅ΠΉ Π½Π΅ΠΊΠΎΡ‚ΠΎΡ€ΠΎΠ³ΠΎ количСства напряТСния Π² Ρ†Π΅ΠΏΠΈ. ΠŸΠΎΡ‚Π΅Ρ€ΡŽ напряТСния слСдуСт Ρ€Π°Π·Π»ΠΈΡ‡Π°Ρ‚ΡŒ ΠΎΡ‚ падСния напряТСния.

РасчСт сопротивлСния

Π‘ΠΎΠΏΡ€ΠΎΡ‚ΠΈΠ²Π»Π΅Π½ΠΈΠ΅ Π½Π° участкС Ρ†Π΅ΠΏΠΈ рассчитываСтся ΠΏΠΎ классичСской Ρ„ΠΎΡ€ΠΌΡƒΠ»Π΅ $R = \frac{U}{I}$. Для этого Π½Π΅ΠΎΠ±Ρ…ΠΎΠ΄ΠΈΠΌΠΎ ΡƒΡΡ‚Π°Π½ΠΎΠ²ΠΈΡ‚ΡŒ значСния напряТСния ΠΈ Ρ‚ΠΎΠΊΠ°. Π‘ΠΎΠΏΡ€ΠΎΡ‚ΠΈΠ²Π»Π΅Π½ΠΈΠ΅ – ΠΎΡ‚Π½ΠΎΡˆΠ΅Π½ΠΈΠ΅ напряТСния ΠΊ Ρ‚ΠΎΠΊΡƒ.

ΠŸΡ€ΠΈ ΠΌΠ½ΠΎΠ³ΠΎΠΊΡ€Π°Ρ‚Π½ΠΎΠΌ ΡƒΠ²Π΅Π»ΠΈΡ‡Π΅Π½ΠΈΠΈ ΠΈΠ»ΠΈ ΡƒΠΌΠ΅Π½ΡŒΡˆΠ΅Π½ΠΈΠΈ напряТСния Ρ‚ΠΎΠΊ Ρ‚Π°ΠΊΠΆΠ΅ измСняСтся Π² нСсколько Ρ€Π°Π· Π² Ρ‚Ρƒ ΠΈΠ»ΠΈ ΠΈΠ½ΡƒΡŽ сторону. ΠžΡ‚Π½ΠΎΡˆΠ΅Π½ΠΈΠ΅ напряТСния ΠΊ Ρ‚ΠΎΠΊΡƒ, ΠΊΠΎΡ‚ΠΎΡ€ΠΎΠ΅ Ρ€Π°Π²Π½ΠΎ ΡΠΎΠΏΡ€ΠΎΡ‚ΠΈΠ²Π»Π΅Π½ΠΈΡŽ, всСгда остаСтся Π½Π° Π½Π΅ΠΈΠ·ΠΌΠ΅Π½Π½ΠΎΠΌ ΡƒΡ€ΠΎΠ²Π½Π΅.

Π‘ΠΎΠΏΡ€ΠΎΡ‚ΠΈΠ²Π»Π΅Π½ΠΈΠ΅ ΠΎΠΏΡ€Π΅Π΄Π΅Π»Π΅Π½Π½ΠΎΠ³ΠΎ ΠΏΡ€ΠΎΠ²ΠΎΠ΄Π½ΠΈΠΊΠ° Π½Π΅ зависит ΠΎΡ‚ напряТСния ΠΈ Ρ‚ΠΎΠΊΠ°. Оно Π±ΡƒΠ΄Π΅Ρ‚ Π»Π΅ΠΆΠ°Ρ‚ΡŒ Π² зависимости ΠΎΡ‚ ΠΌΠ°Ρ‚Π΅Ρ€ΠΈΠ°Π»Π° ΠΏΡ€ΠΎΠ²ΠΎΠ΄Π½ΠΈΠΊΠ°, Π΅Π³ΠΎ Π΄Π»ΠΈΠ½Ρ‹ ΠΈ ΠΏΠ»ΠΎΡ‰Π°Π΄ΠΈ сСчСния. Π€ΠΎΡ€ΠΌΡƒΠ»Π° для расчСта сопротивлСния Π½Π° участкС Ρ†Π΅ΠΏΠΈ ΠΎΡ‡Π΅Π½ΡŒ ΠΏΠΎΡ…ΠΎΠΆΠ° Π½Π° Ρ„ΠΎΡ€ΠΌΡƒΠ»Ρƒ для опрСдСлСния Ρ‚ΠΎΠΊΠ°, ΠΎΠ΄Π½Π°ΠΊΠΎ сущСствуСт ΠΌΠ΅ΠΆΠ΄Ρƒ Π½ΠΈΠΌΠΈ ΠΏΡ€ΠΈΠ½Ρ†ΠΈΠΏΠΈΠ°Π»ΡŒΠ½ΠΎΠ΅ Ρ€Π°Π·Π»ΠΈΡ‡ΠΈΠ΅.

Оно состоит Π² Ρ‚ΠΎΠΌ, Ρ‡Ρ‚ΠΎ Ρ‚ΠΎΠΊ Π½Π° ΠΊΠΎΠ½ΠΊΡ€Π΅Ρ‚Π½ΠΎΠΌ участкС Ρ†Π΅ΠΏΠΈ зависит ΠΎΡ‚ напряТСния ΠΈ сопротивлСния, поэтому измСняСтся Ρ‚Π°ΠΊΠΈΠΌ ΠΆΠ΅ ΠΎΠ±Ρ€Π°Π·ΠΎΠΌ. Π‘ΠΎΠΏΡ€ΠΎΡ‚ΠΈΠ²Π»Π΅Π½ΠΈΠ΅ Π½Π° Π΄Π°Π½Π½ΠΎΠΌ участкС Ρ†Π΅ΠΏΠΈ – постоянная Π²Π΅Π»ΠΈΡ‡ΠΈΠ½Π°. Она Π½Π΅ зависит ΠΎΡ‚ измСнСния Π·Π½Π°Ρ‡Π΅Π½ΠΈΠΉ Ρ‚ΠΎΠΊΠ° ΠΈ напряТСния, ΠΎΠ΄Π½Π°ΠΊΠΎ Ρ€Π°Π²Π½ΠΎ ΠΎΡ‚Π½ΠΎΡˆΠ΅Π½ΠΈΡŽ этих Π²Π΅Π»ΠΈΡ‡ΠΈΠ½.

Π’ΠΎΠ»ΡŒΡ‚Π°ΠΌΠΏΠ΅Ρ€Π½Π°Ρ характСристика

Π—Π°ΠΊΠΎΠ½ Ома ΠΏΡ€Π΅Π΄ΡΡ‚Π°Π²Π»ΡΡŽΡ‚ Π² Π²ΠΈΠ΄Π΅ Π²ΠΎΠ»ΡŒΡ‚Π°ΠΌΠΏΠ΅Ρ€Π½ΠΎΠΉ характСристики. Π—Π°Π²ΠΈΡΠΈΠΌΠΎΡΡ‚ΡŒ ΠΌΠ΅ΠΆΠ΄Ρƒ двумя ΠΏΡ€ΠΎΠΏΠΎΡ€Ρ†ΠΈΠΎΠ½Π°Π»ΡŒΠ½Ρ‹ΠΌΠΈ Π²Π΅Π»ΠΈΡ‡ΠΈΠ½Π°ΠΌΠΈ выраТаСтся прямой Π»ΠΈΠ½ΠΈΠ΅ΠΉ Π½Π° Π³Ρ€Π°Ρ„ΠΈΠΊΠ΅. Она ΠΏΡ€ΠΎΡ…ΠΎΠ΄ΠΈΡ‚ Ρ‡Π΅Ρ€Π΅Π· Π½Π°Ρ‡Π°Π»ΠΎ ΠΊΠΎΠΎΡ€Π΄ΠΈΠ½Π°Ρ‚. ΠŸΠΎΠ΄ΠΎΠ±Π½ΡƒΡŽ ΠΏΡ€ΡΠΌΡƒΡŽ ΠΏΡ€ΠΎΠΏΠΎΡ€Ρ†ΠΈΠΎΠ½Π°Π»ΡŒΠ½ΡƒΡŽ Π·Π°Π²ΠΈΡΠΈΠΌΠΎΡΡ‚ΡŒ Π²Π΅Π»ΠΈΡ‡ΠΈΠ½ Ρ‚Π°ΠΊΠΆΠ΅ Π½Π°Π·Ρ‹Π²Π°ΡŽΡ‚ Π»ΠΈΠ½Π΅ΠΉΠ½ΠΎΠΉ Π·Π°Π²ΠΈΡΠΈΠΌΠΎΡΡ‚ΡŒΡŽ.

Π’ графичСском Π²Ρ‹Ρ€Π°ΠΆΠ΅Π½ΠΈΠΈ Π·Π°ΠΊΠΎΠ½Π° Ома для участка Ρ†Π΅ΠΏΠΈ ΠΏΡ€ΠΈ ΠΎΡ‚Ρ€ΠΈΡ†Π°Ρ‚Π΅Π»ΡŒΠ½Ρ‹Ρ… значСниях напряТСния ΠΈ Ρ‚ΠΎΠΊΠ° Ρ‚Π°ΠΊΠΆΠ΅ Ρ€ΠΈΡΡƒΡŽΡ‚ ΠΏΡ€ΡΠΌΡƒΡŽ линию. Π­Ρ‚ΠΎ ΠΎΠ·Π½Π°Ρ‡Π°Π΅Ρ‚, Ρ‡Ρ‚ΠΎ Ρ‚ΠΎΠΊ Π² Ρ†Π΅ΠΏΠΈ ΠΏΡ€ΠΎΡ…ΠΎΠ΄ΠΈΡ‚ Π² Ρ€Π°Π·Π½Ρ‹Ρ… направлСниях ΠΎΠ΄ΠΈΠ½Π°ΠΊΠΎΠ²ΠΎ. ΠŸΡ€ΠΈ большСм сопротивлСнии мСньшСС Π·Π½Π°Ρ‡Π΅Π½ΠΈΠ΅ ΠΈΠΌΠ΅Π΅Ρ‚ Ρ‚ΠΎΠΊ с Ρ‚Π°ΠΊΠΈΠΌ ΠΆΠ΅ напряТСниСм.

Π’ΠΎΠ»ΡŒΡ‚Π°ΠΌΠΏΠ΅Ρ€Π½ΡƒΡŽ характСристику ΡΠΎΡΡ‚Π°Π²Π»ΡΡŽΡ‚ ΠΏΡ€ΠΈ ΠΏΠΎΠΌΠΎΡ‰ΠΈ ΡΠΏΠ΅Ρ†ΠΈΠ°Π»ΡŒΠ½Ρ‹Ρ… ΠΏΡ€ΠΈΠ±ΠΎΡ€ΠΎΠ². Π›ΠΈΠ½Π΅ΠΉΠ½Ρ‹ΠΌΠΈ Π½Π°Π·Ρ‹Π²Π°ΡŽΡ‚ Ρ‚Π°ΠΊΠΈΠ΅ ΠΏΡ€ΠΈΠ±ΠΎΡ€Ρ‹, Ρƒ ΠΊΠΎΡ‚ΠΎΡ€Ρ‹Ρ… характСристика выраТаСтся прямой Π»ΠΈΠ½ΠΈΠ΅ΠΉ, ΠΈ ΠΎΠ½Π° ΠΏΡ€ΠΎΡ…ΠΎΠ΄ΠΈΡ‚ Ρ‡Π΅Ρ€Π΅Π· Π½Π°Ρ‡Π°Π»ΠΎ ΠΊΠΎΠΎΡ€Π΄ΠΈΠ½Π°Ρ‚.

БпСциалисты ΠΏΡ€ΠΈ составлСнии Π²ΠΎΠ»ΡŒΡ‚Π°ΠΌΠΏΠ΅Ρ€Π½ΠΎΠΉ характСристики ΠΏΡ€ΠΈΠΌΠ΅Π½ΡΡŽΡ‚ Ρ‚Π°ΠΊΠΆΠ΅ понятия Π»ΠΈΠ½Π΅ΠΉΠ½Ρ‹Π΅ сопротивлСния ΠΈ Π»ΠΈΠ½Π΅ΠΉΠ½Ρ‹Π΅ Ρ†Π΅ΠΏΠΈ.

ΠžΠΏΡ€Π΅Π΄Π΅Π»Π΅Π½ΠΈΠ΅ 2

НСлинСйными Π½Π°Π·Ρ‹Π²Π°ΡŽΡ‚ ΠΏΡ€ΠΈΠ±ΠΎΡ€Ρ‹, Ρƒ ΠΊΠΎΡ‚ΠΎΡ€Ρ‹Ρ… сопротивлСниС мСняСтся ΠΏΡ€ΠΈ ΠΈΠ·ΠΌΠ΅Π½Π΅Π½ΠΈΠΈ Ρ‚ΠΎΠΊΠ° ΠΈΠ»ΠΈ напряТСния. Для Ρ‚Π°ΠΊΠΈΡ… случаСв ΡƒΠΆΠ΅ Π½Π΅ дСйствуСт Π·Π°ΠΊΠΎΠ½ Ома.

Π—Π°ΠΊΠΎΠ½ Ома для участка Ρ†Π΅ΠΏΠΈ

Π‘ΠΊΠ°ΠΆΡƒ сразу, Ρ‡Ρ‚ΠΎ Π·Π°ΠΊΠΎΠ½ Ома – основной Π·Π°ΠΊΠΎΠ½ элСктротСхники ΠΈ примСняСтся для расчСта Ρ‚Π°ΠΊΠΈΡ… Π²Π΅Π»ΠΈΡ‡ΠΈΠ½, ΠΊΠ°ΠΊ: Ρ‚ΠΎΠΊ, напряТСниС ΠΈ сопротивлСниС Π² Ρ†Π΅ΠΏΠΈ.

Рассмотрим ΡΠ»Π΅ΠΊΡ‚Ρ€ΠΈΡ‡Π΅ΡΠΊΡƒΡŽ Ρ†Π΅ΠΏΡŒ, ΠΏΡ€ΠΈΠ²Π΅Π΄Π΅Π½Π½ΡƒΡŽ Π½Π° рисункС 1.

Рисунок 1. ΠŸΡ€ΠΎΡΡ‚Π΅ΠΉΡˆΠ°Ρ Ρ†Π΅ΠΏΡŒ, ΠΏΠΎΡΡΠ½ΡΡŽΡ‰Π΅Ρ Π·Π°ΠΊΠΎΠ½ Ома.

ΠœΡ‹ Π·Π½Π°Π΅ΠΌ, Ρ‡Ρ‚ΠΎ элСктричСский Ρ‚ΠΎΠΊ, Ρ‚ΠΎ Π΅ΡΡ‚ΡŒ ΠΏΠΎΡ‚ΠΎΠΊ элСктронов, Π²ΠΎΠ·Π½ΠΈΠΊΠ°Π΅Ρ‚ Π² Ρ†Π΅ΠΏΠΈ ΠΌΠ΅ΠΆΠ΄Ρƒ двумя Ρ‚ΠΎΡ‡ΠΊΠ°ΠΌΠΈ (Π½Π° рисункС А ΠΈ Π‘) с Ρ€Π°Π·Π½Ρ‹ΠΌΠΈ ΠΏΠΎΡ‚Π΅Π½Ρ†ΠΈΠ°Π»Π°ΠΌΠΈ. Π’ΠΎΠ³Π΄Π° слСдуСт ΡΡ‡ΠΈΡ‚Π°Ρ‚ΡŒ, Ρ‡Ρ‚ΠΎ Ρ‡Π΅ΠΌ большС Ρ€Π°Π·Π½ΠΎΡΡ‚ΡŒ ΠΏΠΎΡ‚Π΅Π½Ρ†ΠΈΠ°Π»ΠΎΠ², Ρ‚Π΅ΠΌ большСС количСство элСктронов пСрСмСстятся ΠΈΠ· Ρ‚ΠΎΡ‡ΠΊΠΈ с Π½ΠΈΠ·ΠΊΠΈΠΌ ΠΏΠΎΡ‚Π΅Π½Ρ†ΠΈΠ°Π»ΠΎΠΌ (Π‘) Π² Ρ‚ΠΎΡ‡ΠΊΡƒ с высоким ΠΏΠΎΡ‚Π΅Π½Ρ†ΠΈΠ°Π»ΠΎΠΌ (А). ΠšΠΎΠ»ΠΈΡ‡Π΅ΡΡ‚Π²Π΅Π½Π½ΠΎ Ρ‚ΠΎΠΊ выраТаСтся суммой зарядов ΠΏΡ€ΠΎΡˆΠ΅Π΄ΡˆΠΈΡ… Ρ‡Π΅Ρ€Π΅Π· Π·Π°Π΄Π°Π½Π½ΡƒΡŽ Ρ‚ΠΎΡ‡ΠΊΡƒ ΠΈ ΡƒΠ²Π΅Π»ΠΈΡ‡Π΅Π½ΠΈΠ΅ разности ΠΏΠΎΡ‚Π΅Π½Ρ†ΠΈΠ°Π»ΠΎΠ², Ρ‚ΠΎ Π΅ΡΡ‚ΡŒ ΠΏΡ€ΠΈΠ»ΠΎΠΆΠ΅Π½Π½ΠΎΠ³ΠΎ напряТСния ΠΊ рСзистору R, ΠΏΡ€ΠΈΠ²Π΅Π΄Π΅Ρ‚ ΠΊ ΡƒΠ²Π΅Π»ΠΈΡ‡Π΅Π½ΠΈΡŽ Ρ‚ΠΎΠΊΠ° Ρ‡Π΅Ρ€Π΅Π· рСзистор.

Π‘ Π΄Ρ€ΡƒΠ³ΠΎΠΉ стороны сопротивлСниС рСзистора противодСйствуСт элСктричСскому Ρ‚ΠΎΠΊΡƒ. Π’ΠΎΠ³Π΄Π° слСдуСт ΡΠΊΠ°Π·Π°Ρ‚ΡŒ, Ρ‡Ρ‚ΠΎ Ρ‡Π΅ΠΌ большС сопротивлСниС рСзистора, Ρ‚Π΅ΠΌ мСньшС Π±ΡƒΠ΄Π΅Ρ‚ срСдняя ΡΠΊΠΎΡ€ΠΎΡΡ‚ΡŒ элСктронов Π² Ρ†Π΅ΠΏΠΈ, Π° это Π²Π΅Π΄Π΅Ρ‚ ΠΊ ΡƒΠΌΠ΅Π½ΡŒΡˆΠ΅Π½ΠΈΡŽ Ρ‚ΠΎΠΊΠ° Ρ‡Π΅Ρ€Π΅Π· рСзистор.

Π‘ΠΎΠ²ΠΎΠΊΡƒΠΏΠ½ΠΎΡΡ‚ΡŒ Π΄Π²ΡƒΡ… этих зависимостСй (Ρ‚ΠΎΠΊΠ° ΠΎΡ‚ напряТСния ΠΈ сопротивлСния) извСстна ΠΊΠ°ΠΊ Π·Π°ΠΊΠΎΠ½ Ома для участка Ρ†Π΅ΠΏΠΈ ΠΈ записываСтся Π² ΡΠ»Π΅Π΄ΡƒΡŽΡ‰Π΅ΠΌ Π²ΠΈΠ΄Π΅:

I=U/R

Π­Ρ‚ΠΎ Π²Ρ‹Ρ€Π°ΠΆΠ΅Π½ΠΈΠ΅ читаСтся ΡΠ»Π΅Π΄ΡƒΡŽΡ‰ΠΈΠΌ ΠΎΠ±Ρ€Π°Π·ΠΎΠΌ: сила Ρ‚ΠΎΠΊΠ° прямо ΠΏΡ€ΠΎΠΏΠΎΡ€Ρ†ΠΈΠΎΠ½Π°Π»ΡŒΠ½Π° Π½Π°ΠΏΡ€ΡΠΆΠ΅Π½ΠΈΡŽ ΠΈ ΠΎΠ±Ρ€Π°Ρ‚Π½ΠΎ ΠΏΡ€ΠΎΠΏΠΎΡ€Ρ†ΠΈΠΎΠ½Π°Π»ΡŒΠ½Π° ΡΠΎΠΏΡ€ΠΎΡ‚ΠΈΠ²Π»Π΅Π½ΠΈΡŽ.

Π‘Π»Π΅Π΄ΡƒΠ΅Ρ‚ Π·Π½Π°Ρ‚ΡŒ Ρ‡Ρ‚ΠΎ:

I – Π²Π΅Π»ΠΈΡ‡ΠΈΠ½Π° Ρ‚ΠΎΠΊΠ°, ΠΏΡ€ΠΎΡ‚Π΅ΠΊΠ°ΡŽΡ‰Π΅Π³ΠΎ Ρ‡Π΅Ρ€Π΅Π· участок Ρ†Π΅ΠΏΠΈ;

U – Π²Π΅Π»ΠΈΡ‡ΠΈΠ½Π° ΠΏΡ€ΠΈΠ»ΠΎΠΆΠ΅Π½Π½ΠΎΠ³ΠΎ напряТСния ΠΊ участку Ρ†Π΅ΠΏΠΈ;

R – Π²Π΅Π»ΠΈΡ‡ΠΈΠ½Π° сопротивлСния рассматриваСмого участка Ρ†Π΅ΠΏΠΈ.

ΠŸΡ€ΠΈ ΠΏΠΎΠΌΠΎΡ‰ΠΈ Π·Π°ΠΊΠΎΠ½Π° Ома для участка Ρ†Π΅ΠΏΠΈ ΠΌΠΎΠΆΠ½ΠΎ Π²Ρ‹Ρ‡ΠΈΡΠ»ΠΈΡ‚ΡŒ ΠΏΡ€ΠΈΠ»ΠΎΠΆΠ΅Π½Π½ΠΎΠ΅ напряТСниС ΠΊ участку Ρ†Π΅ΠΏΠΈ (рисунок 1), Π»ΠΈΠ±ΠΎ напряТСниС Π½Π° Π²Ρ…ΠΎΠ΄Π½Ρ‹Ρ… Π·Π°ΠΆΠΈΠΌΠ°Ρ… Ρ†Π΅ΠΏΠΈ (рисунок 2).

Рисунок 2. ΠŸΠΎΡΠ»Π΅Π΄ΠΎΠ²Π°Ρ‚Π΅Π»ΡŒΠ½Π°Ρ Ρ†Π΅ΠΏΡŒ, ΠΏΠΎΡΡΠ½ΡΡŽΡ‰Π°Ρ расчСт напряТСния Π½Π° Π·Π°ΠΆΠΈΠΌΠ°Ρ… Ρ†Π΅ΠΏΠΈ.

Π’ этом случаС Ρ„ΠΎΡ€ΠΌΡƒΠ»Π° (1) ΠΏΡ€ΠΈΠΌΠ΅Ρ‚ ΡΠ»Π΅Π΄ΡƒΡŽΡ‰ΠΈΠΉ Π²ΠΈΠ΄:

U = I *R

Но ΠΏΡ€ΠΈ этом Π½Π΅ΠΎΠ±Ρ…ΠΎΠ΄ΠΈΠΌΠΎ Π·Π½Π°Ρ‚ΡŒ Ρ‚ΠΎΠΊ ΠΈ сопротивлСниС участка Ρ†Π΅ΠΏΠΈ.

Π’Ρ€Π΅Ρ‚ΠΈΠΉ Π²Π°Ρ€ΠΈΠ°Π½Ρ‚ Π·Π°ΠΊΠΎΠ½Π° Ома для участка Ρ†Π΅ΠΏΠΈ, ΠΏΠΎΠ·Π²ΠΎΠ»ΡΡŽΡ‰ΠΈΠΉ Ρ€Π°ΡΡΡ‡ΠΈΡ‚Π°Ρ‚ΡŒ сопротивлСниС участка Ρ†Π΅ΠΏΠΈ ΠΏΠΎ извСстным значСниям Ρ‚ΠΎΠΊΠ° ΠΈ напряТСния ΠΈΠΌΠ΅Π΅Ρ‚ ΡΠ»Π΅Π΄ΡƒΡŽΡ‰ΠΈΠΉ Π²ΠΈΠ΄:

R =U/I

Как Π·Π°ΠΏΠΎΠΌΠ½ΠΈΡ‚ΡŒ Π·Π°ΠΊΠΎΠ½ Ома: малСнькая Ρ…ΠΈΡ‚Ρ€ΠΎΡΡ‚ΡŒ!

Для Ρ‚ΠΎΠ³ΠΎ, Ρ‡Ρ‚ΠΎ Π±Ρ‹ быстро ΠΏΠ΅Ρ€Π΅Π²ΠΎΠ΄ΠΈΡ‚ΡŒ ΡΠΎΠΎΡ‚Π½ΠΎΡˆΠ΅Π½ΠΈΠ΅, ΠΊΠΎΡ‚ΠΎΡ€ΠΎΠ΅ называСтся Π·Π°ΠΊΠΎΠ½ Ома, Π½Π΅ ΠΏΡƒΡ‚Π°Ρ‚ΡŒΡΡ, ΠΊΠΎΠ³Π΄Π° Π½Π΅ΠΎΠ±Ρ…ΠΎΠ΄ΠΈΠΌΠΎ Π΄Π΅Π»ΠΈΡ‚ΡŒ, Π° ΠΊΠΎΠ³Π΄Π° ΡƒΠΌΠ½ΠΎΠΆΠ°Ρ‚ΡŒ входящиС Π² Ρ„ΠΎΡ€ΠΌΡƒΠ»Ρƒ Π·Π°ΠΊΠΎΠ½Π° Ома Π²Π΅Π»ΠΈΡ‡ΠΈΠ½Ρ‹, поступайтС ΡΠ»Π΅Π΄ΡƒΡŽΡ‰ΠΈΠΌ ΠΎΠ±Ρ€Π°Π·ΠΎΠΌ. ΠΠ°ΠΏΠΈΡˆΠΈΡ‚Π΅ Π½Π° листС Π±ΡƒΠΌΠ°Π³ΠΈ Π²Π΅Π»ΠΈΡ‡ΠΈΠ½Ρ‹, ΠΊΠΎΡ‚ΠΎΡ€Ρ‹Π΅ входят Π² Π·Π°ΠΊΠΎΠ½ Ома, Ρ‚Π°ΠΊ ΠΊΠ°ΠΊ ΠΏΠΎΠΊΠ°Π·Π°Π½ΠΎ Π½Π° рисункС 3.

Рисунок 3. Как Π·Π°ΠΏΠΎΠΌΠ½ΠΈΡ‚ΡŒ Π·Π°ΠΊΠΎΠ½ Ома.

Π’Π΅ΠΏΠ΅Ρ€ΡŒ Π·Π°ΠΊΡ€ΠΎΠΉΡ‚Π΅ ΠΏΠ°Π»ΡŒΡ†Π΅ΠΌ, Ρ‚Ρƒ Π²Π΅Π»ΠΈΡ‡ΠΈΠ½Ρƒ, ΠΊΠΎΡ‚ΠΎΡ€ΡƒΡŽ Π½Π΅ΠΎΠ±Ρ…ΠΎΠ΄ΠΈΠΌΠΎ Π½Π°ΠΉΡ‚ΠΈ. Π’ΠΎΠ³Π΄Π° ΠΎΡ‚Π½ΠΎΡΠΈΡ‚Π΅Π»ΡŒΠ½ΠΎΠ΅ располоТСниС ΠΎΡΡ‚Π°Π²ΡˆΠΈΡ…ΡΡ Π½Π΅Π·Π°ΠΊΡ€Ρ‹Ρ‚Ρ‹ΠΌΠΈ Π²Π΅Π»ΠΈΡ‡ΠΈΠ½ подскаТСт, ΠΊΠ°ΠΊΠΎΠ΅ дСйствиС Π½Π΅ΠΎΠ±Ρ…ΠΎΠ΄ΠΈΠΌΠΎ ΡΠΎΠ²Π΅Ρ€ΡˆΠΈΡ‚ΡŒ для вычислСния нСизвСстной Π²Π΅Π»ΠΈΡ‡ΠΈΠ½Ρ‹.

ΠŸΠΎΠ΄Ρ€ΠΎΠ±Π½Π΅Π΅ ΠΌΠΎΠΆΠ½ΠΎ ΡƒΠ·Π½Π°Ρ‚ΡŒ Π² ΠΌΡƒΠ»ΡŒΡ‚ΠΈΠΌΠ΅Π΄ΠΈΠΉΠ½ΠΎΠΌ ΡƒΡ‡Π΅Π±Π½ΠΈΠΊΠ΅ ΠΏΠΎ основам элСктротСхники ΠΈ элСктроники.

ΠŸΠžΠΠ ΠΠ’Π˜Π›ΠΠ‘Π¬ БВАВЬЯ? ΠŸΠžΠ”Π•Π›Π˜Π‘Π¬ Π‘ Π”Π Π£Π—Π¬Π―ΠœΠ˜ Π’ Π‘ΠžΠ¦Π˜ΠΠ›Π¬ΠΠ«Π₯ Π‘Π•Π’Π―Π₯!

ΠŸΠΎΡ…ΠΎΠΆΠΈΠ΅ ΠΌΠ°Ρ‚Π΅Ρ€ΠΈΠ°Π»Ρ‹:

Π”ΠΎΠ±Π°Π²ΠΈΡ‚ΡŒ ΠΊΠΎΠΌΠΌΠ΅Π½Ρ‚Π°Ρ€ΠΈΠΉ

Π—Π°ΠΊΠΎΠ½ Ома для участка Ρ†Π΅ΠΏΠΈ, ΠΏΡ€ΠΈΠΌΠ΅Ρ€ расчСта.

21 Января 2017

6569

ВсСм ΠΏΡ€ΠΈΠ²Π΅Ρ‚.
Π’ ΠΏΡ€Π΅Π΄Ρ‹Π΄ΡƒΡ‰Π΅ΠΉ ΡΡ‚Π°Ρ‚ΡŒΠ΅ ΠΌΡ‹ собрали ΠΏΡ€ΠΎΡΡ‚ΡƒΡŽ Π·Π°ΠΌΠΊΠ½ΡƒΡ‚ΡƒΡŽ Ρ†Π΅ΠΏΡŒ, состоящий ΠΈΠ· источника питания, ΠΏΡ€ΠΎΠ²ΠΎΠ΄Π½ΠΈΠΊΠΎΠ² ΠΏΠΎ ΠΊΠΎΡ‚ΠΎΡ€Ρ‹ΠΌ ΠΏΡ€ΠΎΡ‚Π΅ΠΊΠ°Π΅Ρ‚ Ρ‚ΠΎΠΊ ΠΈ Π½Π°Π³Ρ€ΡƒΠ·ΠΊΠΈ. Выяснили, Ρ‡Ρ‚ΠΎ Ρ‚Π°ΠΊΠΎΠ΅ сопротивлСниС ΠΏΡ€ΠΎΠ²ΠΎΠ΄Π½ΠΈΠΊΠ° ΠΈ сопротивлСниС Π½Π°Π³Ρ€ΡƒΠ·ΠΊΠΈ. Π’Π°ΠΊ ΠΆΠ΅ рассмотрСли взаимосвязь ΠΌΠ΅ΠΆΠ΄Ρƒ напряТСниСм Ρ‚ΠΎΠΊΠ°, силой Ρ‚ΠΎΠΊΠ° ΠΈ сопротивлСниСм Π½Π° Ρ€Π°Π·Π½Ρ‹Ρ… участках Ρ†Π΅ΠΏΠΈ (ΠΏΡ€ΠΎΠ²ΠΎΠ΄Π½ΠΈΠΊΠ° ΠΈ Π½Π°Π³Ρ€ΡƒΠ·ΠΊΠΈ). ВсС эти ΠΎΡ‚Π½ΠΎΡˆΠ΅Π½ΠΈΡ установлСны Π² основном Π·Π°ΠΊΠΎΠ½Π΅ элСктротСхники – Π² Π·Π°ΠΊΠΎΠ½Π΅ Ома.

Π’ этой ΡΡ‚Π°Ρ‚ΡŒΠ΅, ΠΌΡ‹ рассмотрим Π—Π°ΠΊΠΎΠ½ Ома для участка Ρ†Π΅ΠΏΠΈ.

Π—Π°ΠΊΠΎΠ½ Ома для участка Ρ†Π΅ΠΏΠΈ

Π‘ΠΈΠ»Π° Ρ‚ΠΎΠΊΠ° Π² участкС Ρ†Π΅ΠΏΠΈ прямо ΠΏΡ€ΠΎΠΏΠΎΡ€Ρ†ΠΈΠΎΠ½Π°Π»ΡŒΠ½Π° Π½Π°ΠΏΡ€ΡΠΆΠ΅Π½ΠΈΡŽ ΠΈ ΠΎΠ±Ρ€Π°Ρ‚Π½ΠΎ ΠΏΡ€ΠΎΠΏΠΎΡ€Ρ†ΠΈΠΎΠ½Π°Π»ΡŒΠ½Π° элСктричСскому ΡΠΎΠΏΡ€ΠΎΡ‚ΠΈΠ²Π»Π΅Π½ΠΈΡŽ Π΄Π°Π½Π½ΠΎΠ³ΠΎ участка Ρ†Π΅ΠΏΠΈ.

Π”Π°Π²Π°ΠΉΡ‚Π΅ рассмотрим этот Π·Π°ΠΊΠΎΠ½ Π½Π° ΠΏΡ€ΠΈΠΌΠ΅Ρ€Π΅. Π‘ΠΎΠ±Π΅Ρ€Π΅ΠΌ ΡΠ»Π΅Π΄ΡƒΡŽΡ‰ΡƒΡŽ схСму:

Π’Π°ΠΊ ΠΊΠ°ΠΊ сопротивлСниС ΠΏΡ€ΠΎΠ²ΠΎΠ΄Π½ΠΈΠΊΠΎΠ² Π±Π»ΠΈΠ·ΠΊΠΎ ΠΊ Π½ΡƒΠ»ΡŽ, Π±ΡƒΠ΄Π΅ΠΌ ΡΡ‡ΠΈΡ‚Π°Ρ‚ΡŒ, Ρ‡Ρ‚ΠΎ ΠΎΠ½ΠΈ Ρ€Π°Π²Π½Ρ‹ Π½ΡƒΠ»ΡŽ. Π’ Π½Π°ΡˆΡƒ ΡΠ»Π΅ΠΊΡ‚Ρ€ΠΈΡ‡Π΅ΡΠΊΡƒΡŽ Ρ†Π΅ΠΏΡŒ, ΠΊΡ€ΠΎΠΌΠ΅ Π½Π°Π³Ρ€ΡƒΠ·ΠΊΠΈ, ΠΌΡ‹ Π΅Ρ‰Π΅ Π΄ΠΎΠ±Π°Π²ΠΈΠ»ΠΈ Π΄Π²Π° ΠΏΡ€ΠΈΠ±ΠΎΡ€Π°.
АмпСрмСтр – ΠΏΡ€ΠΈΠ±ΠΎΡ€ для измСрСния силы Ρ‚ΠΎΠΊΠ°, ΠΈΠ»ΠΈ Π΄Ρ€ΡƒΠ³ΠΈΠΌΠΈ словами измСряСт сколько потрСбляСт Π½Π°Π³Ρ€ΡƒΠ·ΠΊΠ°, Ρ‚Π°ΠΊ Π»Π΅Π³Ρ‡Π΅ Π·Π°ΠΏΠΎΠΌΠ½ΠΈΡ‚ΡŒ. БоСдиняСтся ΠΏΠΎΡΠ»Π΅Π΄ΠΎΠ²Π°Ρ‚Π΅Π»ΡŒΠ½ΠΎ с Π½Π°Π³Ρ€ΡƒΠ·ΠΊΠΎΠΉ.

Π’ΠΎΠ»ΡŒΡ‚ΠΌΠ΅Ρ‚Ρ€ – ΠΏΡ€ΠΈΠ±ΠΎΡ€ для измСрСния напряТСния Ρ‚ΠΎΠΊΠ°, ΠΏΡ€ΠΈ ΠΏΠΎΠ΄ΠΊΠ»ΡŽΡ‡Π΅Π½ΠΈΠΈ ΠΊ Π½Π°Π³Ρ€ΡƒΠ·ΠΊΠ΅, ΠΏΠΎΠΊΠ°Π·Ρ‹Π²Π°Π΅Ρ‚ сколько ΠΏΠ°Π΄Π°Π΅Ρ‚ напряТСниС Π½Π° Π½Π°Π³Ρ€ΡƒΠ·ΠΊΡƒ. БоСдинятся ΠΏΠ°Ρ€Π°Π»Π»Π΅Π»ΡŒΠ½ΠΎ с Π½Π°Π³Ρ€ΡƒΠ·ΠΊΠΎΠΉ.

Π”Π°Π²Π°ΠΉΡ‚Π΅ Π½Π°Π³Ρ€ΡƒΠ·ΠΊΡƒ поставим сопротивлСниСм Ρ€Π°Π²Π½ΠΎΠΉ 100 Ом, с источника питания пустим напряТСниС 5 Π’ (Π²ΠΎΠ»ΡŒΡ‚). Π‘Π½ΠΈΠΌΠ΅ΠΌ показания с ΠΏΡ€ΠΈΠ±ΠΎΡ€ΠΎΠ². Нас интСрСсуСт ΠΏΠΎΠΊΠ°Π·Π°Ρ‚Π΅Π»ΡŒ Π°ΠΌΠΏΠ΅Ρ€ΠΌΠ΅Ρ‚Ρ€Π°. АмпСрмСтр ΠΏΠΎΠΊΠ°Π·Ρ‹Π²Π°Π΅Ρ‚ — 0,05 А (Π°ΠΌΠΏΠ΅Ρ€) для удобства ΠΌΠΎΠΆΠ½ΠΎ пСрСвСсти Π² ΠΌΠΈΠ»Π»ΠΈΠ°ΠΌΠΏΠ΅Ρ€Ρ‹ – 50 мА (ΠΌΠΈΠ»Π»ΠΈΠ°ΠΌΠΏΠ΅Ρ€).

Π’Π΅ΠΏΠ΅Ρ€ΡŒ помСняСм напряТСниС Ρ‚ΠΎΠΊΠ°, вмСсто 5 Π’ установим 10 Π’. Π‘Π½ΠΈΠΌΠ΅ΠΌ ΠΏΠΎΠΊΠ°Π·Π°Ρ‚Π΅Π»ΡŒ Π°ΠΌΠΏΠ΅Ρ€ΠΌΠ΅Ρ‚Ρ€Π°. АмпСрмСтр ΠΏΠΎΠΊΠ°Π·Ρ‹Π²Π°Π΅Ρ‚ — 0,1 А ΠΏΠ΅Ρ€Π΅Π²ΠΎΠ΄ΠΈΠΌ Π² ΠΌΠΈΠ»Π»ΠΈΠ°ΠΌΠΏΠ΅Ρ€Ρ‹ – 100 мА. Π‘Ρ€Π°Π·Ρƒ ΠΎΡ‚ΠΌΠ΅Ρ‚ΠΈΠΌ для сСбя — с ΡƒΠ²Π΅Π»ΠΈΡ‡Π΅Π½ΠΈΠ΅ΠΌ напряТСния ΡƒΠ²Π΅Π»ΠΈΡ‡ΠΈΠ»Π°ΡΡŒ сила Ρ‚ΠΎΠΊΠ°.
Π’ Π·Π°ΠΊΠΎΠ½Π΅ ΠΎΠΌΠ°: «сила Ρ‚ΠΎΠΊΠ° Π² участкС Ρ†Π΅ΠΏΠΈ прямо ΠΏΡ€ΠΎΠΏΠΎΡ€Ρ†ΠΈΠΎΠ½Π°Π»ΡŒΠ½Π° Π½Π°ΠΏΡ€ΡΠΆΠ΅Π½ΠΈΡŽ … Β».

Π’Π΅ΠΏΠ΅Ρ€ΡŒ вСрнСмся ΠΊ ΠΏΠ΅Ρ€Π²ΠΎΠΌΡƒ ΠΎΠΏΡ‹Ρ‚Ρƒ, Ρ‚ΠΎ Π΅ΡΡ‚ΡŒ установим напряТСниС ΠΎΠ±Ρ€Π°Ρ‚Π½ΠΎ Π½Π° Π·Π½Π°Ρ‡Π΅Π½ΠΈΠ΅ 5 Π’. ΠŸΠΎΠΏΡ€ΠΎΠ±ΡƒΠ΅ΠΌ ΠΈΠ·ΠΌΠ΅Π½ΠΈΡ‚ΡŒ сопротивлСниС Π½Π°Π³Ρ€ΡƒΠ·ΠΊΠΈ. ПомСняСм Π½Π°Π³Ρ€ΡƒΠ·ΠΊΡƒ со Π·Π½Π°Ρ‡Π΅Π½ΠΈΠ΅ сопротивлСния 200 Ом. Π‘Π½ΠΈΠΌΠ΅ΠΌ ΠΏΠΎΠΊΠ°Π·Π°Ρ‚Π΅Π»ΠΈ с Π°ΠΌΠΏΠ΅Ρ€ΠΌΠ΅Ρ‚Ρ€Π° ΠΈ сравним с показатСлями ΠΏΠ΅Ρ€Π²ΠΎΠ³ΠΎ ΠΎΠΏΡ‹Ρ‚Π°. АмпСрмСтр ΠΏΠΎΠΊΠ°Π·Ρ‹Π²Π°Π΅Ρ‚ — 0,025 А ΠΏΠ΅Ρ€Π΅Π²ΠΎΠ΄ΠΈΠΌ Π² ΠΌΠΈΠ»Π»ΠΈΠ°ΠΌΠΏΠ΅Ρ€Ρ‹ – 25 мА. Π’Π°ΠΊΠΈΠΌ ΠΎΠ±Ρ€Π°Π·ΠΎΠΌ ΡƒΠ²Π΅Π»ΠΈΡ‡Π΅Π½ΠΈΠ΅ сопротивлСния Π½Π°Π³Ρ€ΡƒΠ·ΠΊΠΈ, ΡƒΠΌΠ΅Π½ΡŒΡˆΠΈΠ»ΠΎ силу Ρ‚ΠΎΠΊΠ°.
Π’ Π·Π°ΠΊΠΎΠ½Π΅ ΠΎΠΌΠ°: «сила Ρ‚ΠΎΠΊΠ° Π² участкС Ρ†Π΅ΠΏΠΈ … ΠΎΠ±Ρ€Π°Ρ‚Π½ΠΎ ΠΏΡ€ΠΎΠΏΠΎΡ€Ρ†ΠΈΠΎΠ½Π°Π»ΡŒΠ½Π° элСктричСскому ΡΠΎΠΏΡ€ΠΎΡ‚ΠΈΠ²Π»Π΅Π½ΠΈΡŽΒ».

Π—Π°ΠΊΠΎΠ½ Ома для участка цСпи записываСтся ΡΠ»Π΅Π΄ΡƒΡŽΡ‰Π΅ΠΉ Ρ„ΠΎΡ€ΠΌΡƒΠ»ΠΎΠΉ: I = U/R
Как Π½Π°ΠΌ ΡƒΠΆΠ΅ извСстно:
I = сила Ρ‚ΠΎΠΊΠ°
U = напряТСниС Ρ‚ΠΎΠΊΠ°
R = сопротивлСниС (сопротивлСниС Π½Π°Π³Ρ€ΡƒΠ·ΠΊΠΈ)

Π’Π°ΠΊ ΠΆΠ΅ эту Ρ„ΠΎΡ€ΠΌΡƒΠ»Ρƒ ΠΌΠΎΠΆΠ½ΠΎ ΠΏΡ€Π΅ΠΎΠ±Ρ€Π°Π·ΠΎΠ²Ρ‹Π²Π°Ρ‚ΡŒ для опрСдСлСния напряТСния Ρ‚ΠΎΠΊΠ° ΠΈΠ»ΠΈ сопротивлСния Π½Π°Π³Ρ€ΡƒΠ·ΠΊΠΈ. Π§Ρ‚ΠΎ Π±Ρ‹ Π»Π΅Π³Ρ‡Π΅ Π·Π°ΠΏΠΎΠΌΠ½ΠΈΡ‚ΡŒ Ρ„ΠΎΡ€ΠΌΡƒΠ»Ρ‹, Π½Π°Π΄ΠΎ Π·Π°ΠΏΠΎΠΌΠ½ΠΈΡ‚ΡŒ Ρ‚Ρ€Π΅ΡƒΠ³ΠΎΠ»ΡŒΠ½ΠΈΠΊ Ома, ΠΊΠΎΡ‚ΠΎΡ€Ρ‹ΠΉ ΠΈΠ·ΠΎΠ±Ρ€Π°ΠΆΠ΅Π½ Π²Ρ‹ΡˆΠ΅. Закрывая ΠΈΡΠΊΠΎΠΌΡƒΡŽ Π²Π΅Π»ΠΈΡ‡ΠΈΠ½Ρƒ ΠΏΠ°Π»ΡŒΡ†Π΅ΠΌ, ΠΌΠΎΠΆΠ½ΠΎ ΡƒΠ²ΠΈΠ΄Π΅Ρ‚ΡŒ Ρ„ΠΎΡ€ΠΌΡƒΠ»Ρƒ для Π½Π΅Π΅.

Π€ΠΎΡ€ΠΌΡƒΠ»Π° для опрСдСлСния напряТСния:

Π€ΠΎΡ€ΠΌΡƒΠ»Π° для опрСдСлСния сопротивлСния:

Рассмотрим простой ΠΏΡ€ΠΈΠΌΠ΅Ρ€ расчСта ΠΈΡΠΏΠΎΠ»ΡŒΠ·ΡƒΡ Π·Π°ΠΊΠΎΠ½ Ома для участка Ρ†Π΅ΠΏΠΈ. Если Π² ΠΏΡ€ΠΈΠΌΠ΅Ρ€Π΅ Π²Ρ‹ΡˆΠ΅, ΠΌΡ‹ Π±Ρ‹ Π½Π΅ использовали Π°ΠΌΠΏΠ΅Ρ€ΠΌΠ΅Ρ‚Ρ€, зная напряТСниС Ρ‚ΠΎΠΊΠ° 5 Π’ (U) ΠΈ сопротивлСниС Π½Π°Π³Ρ€ΡƒΠ·ΠΊΠΈ 100 Ом (R). Π˜ΡΠΏΠΎΠ»ΡŒΠ·ΡƒΡŽ ΡΠ»Π΅Π΄ΡƒΡŽΡ‰ΡƒΡŽ Ρ„ΠΎΡ€ΠΌΡƒΠ»Ρƒ I = U/R, ΠΌΡ‹ Π±Ρ‹ ΠΏΠΎΠ»ΡƒΡ‡ΠΈΠ»ΠΈ Ρ€Π΅Π·ΡƒΠ»ΡŒΡ‚Π°Ρ‚: 5/100 = 0,05. ΠžΡ‚Π²Π΅Ρ‚ 0,05 А = 50 мА.

ΠœΡ‹ Ρ€Π°Π·ΠΎΠ±Ρ€Π°Π»ΠΈ Π·Π°ΠΊΠΎΠ½ Ома для участка Ρ†Π΅ΠΏΠΈ, ознакомились с Ρ„ΠΎΡ€ΠΌΡƒΠ»Π°ΠΌΠΈ для опрСдСлСния силы Ρ‚ΠΎΠΊΠ°, напряТСниС Ρ‚ΠΎΠΊΠ° ΠΈ сопротивлСния. Π’Π°ΠΊ ΠΆΠ΅ Ρ…ΠΎΡ‡Ρƒ Π΄ΠΎΠ±Π°Π²ΠΈΡ‚ΡŒ, ΠΏΡ€ΠΈ расчСтах, Π½Π΅ΠΎΠ±Ρ…ΠΎΠ΄ΠΈΠΌΠΎ ΠΏΠ΅Ρ€Π΅Π²ΠΎΠ΄ΠΈΡ‚ΡŒ Π΅Π΄ΠΈΠ½ΠΈΡ†Ρ‹ измСрСния Π² систСму БИ. Π’ ΠΏΡ€ΠΈΠΌΠ΅Ρ€Π°Ρ… Π²Ρ‹ΡˆΠ΅ для дСмонстраций Π·Π°ΠΌΠΊΠ½ΡƒΡ‚ΠΎΠΉ Ρ†Π΅ΠΏΠΈ, я использовал ΠΏΡ€ΠΎΠ³Ρ€Π°ΠΌΠΌΡƒ — Electronics Workbench. ΠŸΡ€ΠΎΠ³Ρ€Π°ΠΌΠΌΠ° ΠΏΡ€Π΅Π΄Π½Π°Π·Π½Π°Ρ‡Π΅Π½Π° для модСлирования ΠΈ Π°Π½Π°Π»ΠΈΠ·Π° элСктронных схСм.

Π“Π»Π°Π²Π° 21. ЭлСктричСский Ρ‚ΠΎΠΊ. Π—Π°ΠΊΠΎΠ½Ρ‹ Ома ΠΈ ДТоуля-Π›Π΅Π½Ρ†Π°

Для Ρ€Π΅ΡˆΠ΅Π½ΠΈΡ Π·Π°Π΄Π°Ρ‡ Π•Π“Π­ Π½Π° постоянный Ρ‚ΠΎΠΊ Π½Π°Π΄ΠΎ Π·Π½Π°Ρ‚ΡŒ опрСдСлСния Ρ‚ΠΎΠΊΠ°, напряТСния, сопротивлСния, Π·Π°ΠΊΠΎΠ½ Ома для участка Ρ†Π΅ΠΏΠΈ ΠΈ Π·Π°ΠΌΠΊΠ½ΡƒΡ‚ΠΎΠΉ Ρ†Π΅ΠΏΠΈ, Π·Π°ΠΊΠΎΠ½ ДТоуля-Π›Π΅Π½Ρ†Π°, Π° Ρ‚Π°ΠΊΠΆΠ΅ ΡƒΠΌΠ΅Ρ‚ΡŒ Π½Π°Ρ…ΠΎΠ΄ΠΈΡ‚ΡŒ эквивалСнтныС сопротивлСния ΠΏΡ€ΠΎΡΡ‚Π΅ΠΉΡˆΠΈΡ… элСктричСски Ρ†Π΅ΠΏΠ΅ΠΉ. Рассмотрим эти вопросы.

ЭлСктричСским Ρ‚ΠΎΠΊΠΎΠΌ Π½Π°Π·Ρ‹Π²Π°ΡŽΡ‚ упорядочСнноС Π΄Π²ΠΈΠΆΠ΅Π½ΠΈΠ΅ заряТСнных частиц. Π‘ΠΈΠ»ΠΎΠΉ Ρ‚ΠΎΠΊΠ° Π² Π½Π΅ΠΊΠΎΡ‚ΠΎΡ€ΠΎΠΌ сСчСнии ΠΏΡ€ΠΎΠ²ΠΎΠ΄Π½ΠΈΠΊΠ° называСтся ΠΎΡ‚Π½ΠΎΡˆΠ΅Π½ΠΈΠ΅ заряда , ΠΏΡ€ΠΎΡ‚Π΅ΠΊΡˆΠ΅Π³ΠΎ Ρ‡Π΅Ρ€Π΅Π· это сСчСниС Π·Π° ΠΈΠ½Ρ‚Π΅Ρ€Π²Π°Π» Π²Ρ€Π΅ΠΌΠ΅Π½ΠΈ , ΠΊ этому ΠΈΠ½Ρ‚Π΅Ρ€Π²Π°Π»Ρƒ Π²Ρ€Π΅ΠΌΠ΅Π½ΠΈ

(21.1)

Π§Ρ‚ΠΎΠ±Ρ‹ Π² ΠΏΡ€ΠΎΠ²ΠΎΠ΄Π½ΠΈΠΊΠ΅ Ρ‚Π΅ΠΊ элСктричСский Ρ‚ΠΎΠΊ, Π² ΠΏΡ€ΠΎΠ²ΠΎΠ΄Π½ΠΈΠΊΠ΅ Π΄ΠΎΠ»ΠΆΠ½ΠΎ Π±Ρ‹Ρ‚ΡŒ элСктричСскоС ΠΏΠΎΠ»Π΅, ΠΈΠ»ΠΈ, Π΄Ρ€ΡƒΠ³ΠΈΠΌΠΈ словами, ΠΏΠΎΡ‚Π΅Π½Ρ†ΠΈΠ°Π»Ρ‹ Ρ€Π°Π·Π»ΠΈΡ‡Π½Ρ‹Ρ… Ρ‚ΠΎΡ‡Π΅ΠΊ ΠΏΡ€ΠΎΠ²ΠΎΠ΄Π½ΠΈΠΊΠ° Π΄ΠΎΠ»ΠΆΠ½Ρ‹ Π±Ρ‹Ρ‚ΡŒ Ρ€Π°Π·Π½Ρ‹ΠΌΠΈ. Но ΠΏΡ€ΠΈ Π΄Π²ΠΈΠΆΠ΅Π½ΠΈΠΈ элСктричСских зарядов ΠΏΠΎ ΠΏΡ€ΠΎΠ²ΠΎΠ΄Π½ΠΈΠΊΡƒ ΠΏΠΎΡ‚Π΅Π½Ρ†ΠΈΠ°Π»Ρ‹ Ρ€Π°Π·Π»ΠΈΡ‡Π½Ρ‹Ρ… Ρ‚ΠΎΡ‡Π΅ΠΊ ΠΏΡ€ΠΎΠ²ΠΎΠ΄Π½ΠΈΠΊΠ° Π±ΡƒΠ΄ΡƒΡ‚ Π²Ρ‹Ρ€Π°Π²Π½ΠΈΠ²Π°Ρ‚ΡŒΡΡ (см. Π³Π». 19). ΠŸΠΎΡΡ‚ΠΎΠΌΡƒ для протСкания Ρ‚ΠΎΠΊΠ° Π² Ρ‚Π΅Ρ‡Π΅Π½ΠΈΠ΅ Π΄Π»ΠΈΡ‚Π΅Π»ΡŒΠ½ΠΎΠ³ΠΎ Π²Ρ€Π΅ΠΌΠ΅Π½ΠΈ Π½Π° ΠΊΠ°ΠΊΠΈΡ…-Ρ‚ΠΎ участках Ρ†Π΅ΠΏΠΈ Π½Π΅ΠΎΠ±Ρ…ΠΎΠ΄ΠΈΠΌΠΎ ΠΎΠ±Π΅ΡΠΏΠ΅Ρ‡ΠΈΡ‚ΡŒ Π΄Π²ΠΈΠΆΠ΅Π½ΠΈΠ΅ зарядов Π² Π½Π°ΠΏΡ€Π°Π²Π»Π΅Π½ΠΈΠΈ ΠΏΡ€ΠΎΡ‚ΠΈΠ²ΠΎΠΏΠΎΠ»ΠΎΠΆΠ½ΠΎΠΌ полю. Π’Π°ΠΊΠΎΠ΅ Π΄Π²ΠΈΠΆΠ΅Π½ΠΈΠ΅ ΠΌΠΎΠΆΠ΅Ρ‚ Π±Ρ‹Ρ‚ΡŒ обСспСчСно Ρ‚ΠΎΠ»ΡŒΠΊΠΎ силами нСэлСктричСской ΠΏΡ€ΠΈΡ€ΠΎΠ΄Ρ‹, ΠΊΠΎΡ‚ΠΎΡ€Ρ‹Π΅ Π² этом контСкстС принято Π½Π°Π·Ρ‹Π²Π°Ρ‚ΡŒ сторонними. Π’ Π³Π°Π»ΡŒΠ²Π°Π½ΠΈΡ‡Π΅ΡΠΊΠΈΡ… элСмСнтах (Β«Π±Π°Ρ‚Π°Ρ€Π΅ΠΉΠΊΠ°Ρ…Β») сторонниС силы Π²ΠΎΠ·Π½ΠΈΠΊΠ°ΡŽΡ‚ Π² Ρ€Π΅Π·ΡƒΠ»ΡŒΡ‚Π°Ρ‚Π΅ элСктрохимичСских ΠΏΡ€Π΅Π²Ρ€Π°Ρ‰Π΅Π½ΠΈΠΉ Π½Π° Π³Ρ€Π°Π½ΠΈΡ†Π°Ρ… элСктродов ΠΈ элСктролита. Π­Ρ‚ΠΈ прСвращСния ΠΎΠ±Π΅ΡΠΏΠ΅Ρ‡ΠΈΠ²Π°ΡŽΡ‚ ΠΏΠ΅Ρ€Π΅ΠΌΠ΅Ρ‰Π΅Π½ΠΈΠ΅ заряда ΠΏΡ€ΠΎΡ‚ΠΈΠ²ΠΎΠΏΠΎΠ»ΠΎΠΆΠ½ΠΎ Π½Π°ΠΏΡ€Π°Π²Π»Π΅Π½ΠΈΡŽ поля, поддСрТивая Π΄Π²ΠΈΠΆΠ΅Π½ΠΈΠ΅ зарядов ΠΏΠΎ Π·Π°ΠΌΠΊΠ½ΡƒΡ‚ΠΎΠΌΡƒ ΠΏΡƒΡ‚ΠΈ.

Π‘ΠΈΠ»Π° Ρ‚ΠΎΠΊΠ° Π² ΠΎΠ΄Π½ΠΎΡ€ΠΎΠ΄Π½ΠΎΠΌ участкС ΠΏΡ€ΠΎΠ²ΠΎΠ΄Π½ΠΈΠΊΠ° ΠΏΡ€ΠΎΠΏΠΎΡ€Ρ†ΠΈΠΎΠ½Π°Π»ΡŒΠ½Π° напряТСнности элСктричСского поля Π²Π½ΡƒΡ‚Ρ€ΠΈ ΠΏΡ€ΠΎΠ²ΠΎΠ΄Π½ΠΈΠΊΠ°. А ΠΏΠΎΡΠΊΠΎΠ»ΡŒΠΊΡƒ Π½Π°ΠΏΡ€ΡΠΆΠ΅Π½Π½ΠΎΡΡ‚ΡŒ поля Π²Π½ΡƒΡ‚Ρ€ΠΈ ΠΏΡ€ΠΎΠ²ΠΎΠ΄Π½ΠΈΠΊΠ° связана с Ρ€Π°Π·Π½ΠΎΡΡ‚ΡŒΡŽ ΠΏΠΎΡ‚Π΅Π½Ρ†ΠΈΠ°Π»ΠΎΠ² Π΅Π³ΠΎ ΠΊΠΎΠ½Ρ†ΠΎΠ² (ΠΈΠ»ΠΈ элСктричСским напряТСниСм Π½Π° ΠΏΡ€ΠΎΠ²ΠΎΠ΄Π½ΠΈΠΊΠ΅ ), Ρ‚ΠΎ

(21.2)

ΠšΠΎΡΡ„Ρ„ΠΈΡ†ΠΈΠ΅Π½Ρ‚ ΠΏΡ€ΠΎΠΏΠΎΡ€Ρ†ΠΈΠΎΠ½Π°Π»ΡŒΠ½ΠΎΡΡ‚ΠΈ , ΠΊΠΎΡ‚ΠΎΡ€Ρ‹ΠΉ принято Π·Π°ΠΏΠΈΡΡ‹Π²Π°Ρ‚ΡŒ Π² Π·Π½Π°ΠΌΠ΅Π½Π°Ρ‚Π΅Π»ΡŒ Ρ„ΠΎΡ€ΠΌΡƒΠ»Ρ‹ (21.2), являСтся характСристикой ΠΏΡ€ΠΎΠ²ΠΎΠ΄Π½ΠΈΠΊΠ° ΠΈ называСтся Π΅Π³ΠΎ сопротивлСниСм. Π’ Ρ€Π΅Π·ΡƒΠ»ΡŒΡ‚Π°Ρ‚Π΅ Ρ„ΠΎΡ€ΠΌΡƒΠ»Π° (21.2) ΠΏΡ€ΠΈΠ½ΠΈΠΌΠ°Π΅Ρ‚ Π²ΠΈΠ΄

(21.3)

Π€ΠΎΡ€ΠΌΡƒΠ»Π° (21.3) называСтся Π·Π°ΠΊΠΎΠ½ΠΎΠΌ Ома для ΠΎΠ΄Π½ΠΎΡ€ΠΎΠ΄Π½ΠΎΠ³ΠΎ участка Ρ†Π΅ΠΏΠΈ, Π° сам участок Ρ†Π΅ΠΏΠΈ часто Π½Π°Π·Ρ‹Π²Π°ΡŽΡ‚ рСзистором (ΠΎΡ‚ английского слова resistance β€” сопротивлСниС).

Если ΠΏΡ€ΠΎΠ²ΠΎΠ΄Π½ΠΈΠΊ являСтся ΠΎΠ΄Π½ΠΎΡ€ΠΎΠ΄Π½Ρ‹ΠΌ ΠΈ ΠΈΠΌΠ΅Π΅Ρ‚ Ρ†ΠΈΠ»ΠΈΠ½Π΄Ρ€ΠΈΡ‡Π΅ΡΠΊΡƒΡŽ Ρ„ΠΎΡ€ΠΌΡƒ (ΠΏΡ€ΠΎΠ²ΠΎΠ΄), Ρ‚ΠΎ Π΅Π³ΠΎ сопротивлСниС ΠΏΡ€ΠΎΠΏΠΎΡ€Ρ†ΠΈΠΎΠ½Π°Π»ΡŒΠ½ΠΎ Π΄Π»ΠΈΠ½Π΅ ΠΈ ΠΎΠ±Ρ€Π°Ρ‚Π½ΠΎ ΠΏΡ€ΠΎΠΏΠΎΡ€Ρ†ΠΈΠΎΠ½Π°Π»ΡŒΠ½ΠΎ ΠΏΠ»ΠΎΡ‰Π°Π΄ΠΈ сСчСния

(21.4)

Π³Π΄Π΅ коэффициСнт ΠΏΡ€ΠΎΠΏΠΎΡ€Ρ†ΠΈΠΎΠ½Π°Π»ΡŒΠ½ΠΎΡΡ‚ΠΈ зависит Ρ‚ΠΎΠ»ΡŒΠΊΠΎ ΠΎΡ‚ ΠΌΠ°Ρ‚Π΅Ρ€ΠΈΠ°Π»Π° ΠΏΡ€ΠΎΠ²ΠΎΠ΄Π½ΠΈΠΊΠ° ΠΈ называСтся Π΅Π³ΠΎ ΡƒΠ΄Π΅Π»ΡŒΠ½Ρ‹ΠΌ сопротивлСниСм.

Если участок Ρ†Π΅ΠΏΠΈ прСдставляСт собой нСсколько ΠΏΠΎΡΠ»Π΅Π΄ΠΎΠ²Π°Ρ‚Π΅Π»ΡŒΠ½ΠΎ соСдинСнных ΠΎΠ΄Π½ΠΎΡ€ΠΎΠ΄Π½Ρ‹Ρ… ΠΏΡ€ΠΎΠ²ΠΎΠ΄Π½ΠΈΠΊΠΎΠ² с сопротивлСниями (см. рисунок), Ρ‚ΠΎ сила Ρ‚ΠΎΠΊΠ° Ρ‡Π΅Ρ€Π΅Π· ΠΊΠ°ΠΆΠ΄Ρ‹ΠΉ ΠΏΡ€ΠΎΠ²ΠΎΠ΄Π½ΠΈΠΊ Π±ΡƒΠ΄Π΅Ρ‚ ΠΎΠ΄ΠΈΠ½Π°ΠΊΠΎΠ²ΠΎΠΉ , элСктричСскоС напряТСниС Π½Π° всСм участкС Ρ†Π΅ΠΏΠΈ Ρ€Π°Π²Π½ΠΎ суммС напряТСний Π½Π° ΠΊΠ°ΠΆΠ΄ΠΎΠΌ ΠΏΡ€ΠΎΠ²ΠΎΠ΄Π½ΠΈΠΊΠ΅ , Π° эквивалСнтноС сопротивлСниС всСго участка Ρ€Π°Π²Π½ΠΎ суммС сопротивлСний ΠΎΡ‚Π΄Π΅Π»ΡŒΠ½Ρ‹Ρ… ΠΏΡ€ΠΎΠ²ΠΎΠ΄Π½ΠΈΠΊΠΎΠ²

(21.4)

Если участок Ρ†Π΅ΠΏΠΈ прСдставляСт собой нСсколько ΠΎΠ΄Π½ΠΎΡ€ΠΎΠ΄Π½Ρ‹Ρ… ΠΏΡ€ΠΎΠ²ΠΎΠ΄Π½ΠΈΠΊΠΎΠ² с сопротивлСниями , соСдинСнных ΠΏΠ°Ρ€Π°Π»Π»Π΅Π»ΡŒΠ½ΠΎ (см. рисунок), Ρ‚ΠΎ элСктричСскоС напряТСниС Π½Π° ΠΊΠ°ΠΆΠ΄ΠΎΠΌ ΠΏΡ€ΠΎΠ²ΠΎΠ΄Π½ΠΈΠΊΠ΅ Π±ΡƒΠ΄Π΅Ρ‚ ΠΎΠ΄ΠΈΠ½Π°ΠΊΠΎΠ²Ρ‹ΠΌ , Ρ‚ΠΎΠΊ Ρ‡Π΅Ρ€Π΅Π· участок Π±ΡƒΠ΄Π΅Ρ‚ Ρ€Π°Π²Π΅Π½ суммС Ρ‚ΠΎΠΊΠΎΠ², Ρ‚Π΅ΠΊΡƒΡ‰ΠΈΡ… Ρ‡Π΅Ρ€Π΅Π· ΠΊΠ°ΠΆΠ΄Ρ‹ΠΉ ΠΏΡ€ΠΎΠ²ΠΎΠ΄Π½ΠΈΠΊ , Π° Π²Π΅Π»ΠΈΡ‡ΠΈΠ½Π°, обратная эквивалСнтному ΡΠΎΠΏΡ€ΠΎΡ‚ΠΈΠ²Π»Π΅Π½ΠΈΡŽ всСго участка, Ρ€Π°Π²Π½ΠΎ суммС ΠΎΠ±Ρ€Π°Ρ‚Π½Ρ‹Ρ… сопротивлСний ΠΎΡ‚Π΄Π΅Π»ΡŒΠ½Ρ‹Ρ… ΠΏΡ€ΠΎΠ²ΠΎΠ΄Π½ΠΈΠΊΠΎΠ²

(21.5)

Рассмотрим Ρ‚Π΅ΠΏΠ΅Ρ€ΡŒ Π·Π°ΠΊΠΎΠ½ Ома для Π·Π°ΠΌΠΊΠ½ΡƒΡ‚ΠΎΠΉ элСктричСской Ρ†Π΅ΠΏΠΈ. ΠŸΡƒΡΡ‚ΡŒ имССтся замкнутая элСктричСская Ρ†Π΅ΠΏΡŒ, состоящая ΠΈΠ· источника сторонних сил с Π²Π½ΡƒΡ‚Ρ€Π΅Π½Π½ΠΈΠΌ сопротивлСниСм ΠΈ внСшнСго сопротивлСния . ΠŸΡƒΡΡ‚ΡŒ ΠΏΡ€ΠΈ ΠΏΡ€ΠΎΡ…ΠΎΠΆΠ΄Π΅Π½ΠΈΠΈ заряда Ρ‡Π΅Ρ€Π΅Π· источник сторонниС силы ΡΠΎΠ²Π΅Ρ€ΡˆΠ°ΡŽΡ‚ Ρ€Π°Π±ΠΎΡ‚Ρƒ . Π­Π»Π΅ΠΊΡ‚Ρ€ΠΎΠ΄Π²ΠΈΠΆΡƒΡ‰Π΅ΠΉ силой источника (часто ΠΈΡΠΏΠΎΠ»ΡŒΠ·ΡƒΠ΅Ρ‚ΡΡ Π°Π±Π±Ρ€Π΅Π²ΠΈΠ°Ρ‚ΡƒΡ€Π° Π­Π”Π‘) называСтся ΠΎΡ‚Π½ΠΎΡˆΠ΅Π½ΠΈΠ΅ Ρ€Π°Π±ΠΎΡ‚Ρ‹ сторонних сил ΠΊ заряду

(21.6)

Π’ этом случаС сила Ρ‚ΠΎΠΊΠ° Π² Ρ†Π΅ΠΏΠΈ Ρ€Π°Π²Π½Π°

(21.7)

Π€ΠΎΡ€ΠΌΡƒΠ»Π° (21.7) называСтся Π·Π°ΠΊΠΎΠ½ΠΎΠΌ Ома для Π·Π°ΠΌΠΊΠ½ΡƒΡ‚ΠΎΠΉ элСктричСской Ρ†Π΅ΠΏΠΈ.

ΠŸΡ€ΠΈ ΠΏΡ€ΠΎΡ…ΠΎΠΆΠ΄Π΅Π½ΠΈΠΈ элСктричСского Ρ‚ΠΎΠΊΠ° Ρ‡Π΅Ρ€Π΅Π· участок Ρ†Π΅ΠΏΠΈ элСктричСскоС ΠΏΠΎΠ»Π΅ ΡΠΎΠ²Π΅Ρ€ΡˆΠ°Π΅Ρ‚ Ρ€Π°Π±ΠΎΡ‚Ρƒ (часто эту Ρ€Π°Π±ΠΎΡ‚Ρƒ Π½Π°Π·Ρ‹Π²Π°ΡŽΡ‚ Ρ€Π°Π±ΠΎΡ‚ΠΎΠΉ Ρ‚ΠΎΠΊΠ°, хотя Ρ‚Π΅Ρ€ΠΌΠΈΠ½ этот Π½Π΅ ΠΎΡ‡Π΅Π½ΡŒ Ρ‚ΠΎΡ‡Π½Ρ‹ΠΉ). ΠžΡ‡Π΅Π²ΠΈΠ΄Π½ΠΎ, вся эта Ρ€Π°Π±ΠΎΡ‚Π° прСвращаСтся Π² Ρ‚Π΅ΠΏΠ»ΠΎ. ΠŸΠΎΡΡ‚ΠΎΠΌΡƒ Ссли Ρ‡Π΅Ρ€Π΅Π· участок Ρ†Π΅ΠΏΠΈ ΠΏΡ€ΠΎΡˆΠ΅Π» заряд , Π³Π΄Π΅ β€” сила Ρ‚ΠΎΠΊΠ° Π² Ρ†Π΅ΠΏΠΈ, β€” врСмя, Ρ‚ΠΎ количСство Π²Ρ‹Π΄Π΅Π»ΠΈΠ²ΡˆΠ΅ΠΉΡΡ Ρ‚Π΅ΠΏΠ»ΠΎΡ‚Ρ‹ Ρ€Π°Π²Π½ΠΎ

(21.8)

(для получСния послСднСго ΠΈ прСдпослСднСго равСнств использован Π·Π°ΠΊΠΎΠ½ Ома для участка Ρ†Π΅ΠΏΠΈ). Π€ΠΎΡ€ΠΌΡƒΠ»Ρ‹ (21.8) Π½Π°Π·Ρ‹Π²Π°ΡŽΡ‚ΡΡ Π·Π°ΠΊΠΎΠ½ΠΎΠΌ ДТоуля-Π›Π΅Π½Ρ†Π°. Из Ρ„ΠΎΡ€ΠΌΡƒΠ»Ρ‹ (21.8) слСдуСт, Ρ‡Ρ‚ΠΎ количСство Π²Ρ‹Π΄Π΅Π»ΠΈΠ²ΡˆΠ΅ΠΉΡΡ ΠΏΡ€ΠΈ ΠΏΡ€ΠΎΡ‚Π΅ΠΊΠ°Π½ΠΈΠΈ элСктричСского Ρ‚ΠΎΠΊΠ° Ρ‚Π΅ΠΏΠ»ΠΎΡ‚Ρ‹ Π»ΠΈΠ½Π΅ΠΉΠ½ΠΎ зависит ΠΎΡ‚ Π²Ρ€Π΅ΠΌΠ΅Π½ΠΈ наблюдСния. ΠŸΠΎΡΡ‚ΠΎΠΌΡƒ ΠΎΡ‚Π½ΠΎΡˆΠ΅Π½ΠΈΠ΅

(21.9)

ΠΊΠΎΡ‚ΠΎΡ€ΠΎΠ΅ называСтся ΠΌΠΎΡ‰Π½ΠΎΡΡ‚ΡŒΡŽ Ρ‚ΠΎΠΊΠ°, Π½Π΅ зависит ΠΎΡ‚ Π²Ρ€Π΅ΠΌΠ΅Π½ΠΈ наблюдСния. Π€ΠΎΡ€ΠΌΡƒΠ»Ρƒ (21.9) Ρ‚Π°ΠΊΠΆΠ΅ Π½Π°Π·Ρ‹Π²Π°ΡŽΡ‚ Π·Π°ΠΊΠΎΠ½ΠΎΠΌ ДТоуля-Π›Π΅Π½Ρ†Π°.

Рассмотрим Ρ‚Π΅ΠΏΠ΅Ρ€ΡŒ Π·Π°Π΄Π°Ρ‡ΠΈ.

Π‘Ρ‚Ρ€ΡƒΠΊΡ‚ΡƒΡ€Π° ΠΌΠ΅Ρ‚Π°Π»Π»Π° ΠΊΡ€Π°Ρ‚ΠΊΠΎ ΠΎΠ±ΡΡƒΠΆΠ΄Π°Π»Π°ΡΡŒ Π² Π³Π». 16: ΠΏΠΎΠ»ΠΎΠΆΠΈΡ‚Π΅Π»ΡŒΠ½ΠΎ заряТСнныС ΠΈΠΎΠ½Ρ‹ располоТСны Π² ΡƒΠ·Π»Π°Ρ… кристалличСской Ρ€Π΅ΡˆΠ΅Ρ‚ΠΊΠΈ, ΠΎΠ±Ρ€Π°Π·ΠΎΠ²Π°Π²ΡˆΠΈΠ΅ΡΡ Π² Ρ€Π΅Π·ΡƒΠ»ΡŒΡ‚Π°Ρ‚Π΅ диссоциации Π²Π°Π»Π΅Π½Ρ‚Π½Ρ‹Π΅ элСктроны ΠΌΠΎΠ³ΡƒΡ‚ свободно ΠΏΠ΅Ρ€Π΅ΠΌΠ΅Ρ‰Π°Ρ‚ΡŒΡΡ ΠΏΠΎ ΠΏΡ€ΠΎΠ²ΠΎΠ΄Π½ΠΈΠΊΡƒ (свободныС элСктроны). Они ΠΈ ΠΎΡΡƒΡ‰Π΅ΡΡ‚Π²Π»ΡΡŽΡ‚ ΠΏΡ€ΠΎΠ²ΠΎΠ΄ΠΈΠΌΠΎΡΡ‚ΡŒ ΠΌΠ΅Ρ‚Π°Π»Π»Π° (Π·Π°Π΄Π°Ρ‡Π° 21.1.1 β€” ΠΎΡ‚Π²Π΅Ρ‚ 2).

Богласно ΠΎΠΏΡ€Π΅Π΄Π΅Π»Π΅Π½ΠΈΡŽ (21.1) Π½Π°Ρ…ΠΎΠ΄ΠΈΠΌ ΡΡ€Π΅Π΄Π½ΡŽΡŽ силу Ρ‚ΠΎΠΊΠ° Π² ΠΊΠ°Π½Π°Π»Π΅ ΠΌΠΎΠ»Π½ΠΈΠΈ (Π·Π°Π΄Π°Ρ‡Π° 21.1.2)

(ΠΎΡ‚Π²Π΅Ρ‚ 2).

Если Π·Π° 1 ΠΌΠΈΠ½ Ρ‡Π΅Ρ€Π΅Π· сСчСниС ΠΏΡ€ΠΎΠ²ΠΎΠ΄Π½ΠΈΠΊΠ° ΠΏΡ€ΠΎΡ‚Π΅ΠΊΠ°Π΅Ρ‚ заряд 60 Кл (Π·Π°Π΄Π°Ρ‡Π° 21.1.3), Ρ‚ΠΎ сила Ρ‚ΠΎΠΊΠ° Π² этом ΠΏΡ€ΠΎΠ²ΠΎΠ΄Π½ΠΈΠΊΠ΅ Ρ€Π°Π²Π½Π° А. ΠŸΡ€ΠΈΠΌΠ΅Π½ΡΡ Π΄Π°Π»Π΅Π΅ ΠΊ этому ΠΏΡ€ΠΎΠ²ΠΎΠ΄Π½ΠΈΠΊΡƒ Π·Π°ΠΊΠΎΠ½ Ома для участка Ρ†Π΅ΠΏΠΈ, ΠΏΠΎΠ»ΡƒΡ‡Π°Π΅ΠΌ Π’ (ΠΎΡ‚Π²Π΅Ρ‚ 2).

По Π·Π°ΠΊΠΎΠ½Ρƒ Ома для участка Ρ†Π΅ΠΏΠΈ ΠΈΠΌΠ΅Π΅ΠΌ для силы Ρ‚ΠΎΠΊΠ° Ρ‡Π΅Ρ€Π΅Π· участок Ρ†Π΅ΠΏΠΈ послС измСнСния Π΅Π³ΠΎ сопротивлСния ΠΈ элСктричСского напряТСния Π½Π° Π½Π΅ΠΌ (Π·Π°Π΄Π°Ρ‡Π° 21.1.4)

Π’Π°ΠΊΠΈΠΌ ΠΎΠ±Ρ€Π°Π·ΠΎΠΌ, сила Ρ‚ΠΎΠΊΠ° ΡƒΠΌΠ΅Π½ΡŒΡˆΠΈΠ»Π°ΡΡŒ Π² 4 Ρ€Π°Π·Π° (ΠΎΡ‚Π²Π΅Ρ‚ 3).

Богласно Π·Π°ΠΊΠΎΠ½Ρƒ Ома для участка Ρ†Π΅ΠΏΠΈ сопротивлСниС β€” это коэффициСнт ΠΏΡ€ΠΎΠΏΠΎΡ€Ρ†ΠΈΠΎΠ½Π°Π»ΡŒΠ½ΠΎΡΡ‚ΠΈ ΠΌΠ΅ΠΆΠ΄Ρƒ напряТСниСм Π½Π° этом участкС ΠΈ силой Ρ‚ΠΎΠΊΠ° Π² Π½Π΅ΠΌ. ΠŸΠΎΡΡ‚ΠΎΠΌΡƒ Π² Π·Π°Π΄Π°Ρ‡Π΅ 21.1.5 ΠΈΠΌΠ΅Π΅ΠΌ, Π½Π°ΠΏΡ€ΠΈΠΌΠ΅Ρ€, ΠΈΡΠΏΠΎΠ»ΡŒΠ·ΡƒΡ ΠΊΡ€Π°ΠΉΠ½ΡŽΡŽ Ρ‚ΠΎΡ‡ΠΊΡƒ Π³Ρ€Π°Ρ„ΠΈΠΊΠ°

(ΠΎΡ‚Π²Π΅Ρ‚ 2). Из-Π·Π° Π»ΠΈΠ½Π΅ΠΉΠ½ΠΎΠΉ зависимости Ρ‚ΠΎΠΊΠ° ΠΎΡ‚ напряТСния вычислСния ΠΌΠΎΠΆΠ½ΠΎ Π±Ρ‹Π»ΠΎ Π²Ρ‹ΠΏΠΎΠ»Π½ΠΈΡ‚ΡŒ ΠΈ ΠΏΠΎ Π΄Ρ€ΡƒΠ³ΠΈΠΌ Ρ‚ΠΎΡ‡ΠΊΠ°ΠΌ Π³Ρ€Π°Ρ„ΠΈΠΊΠ°, ΠΎΡ‚Π²Π΅Ρ‚ Π±Ρ‹Π» Π±Ρ‹ Ρ‚Π°ΠΊΠΈΠΌ ΠΆΠ΅.

Богласно Ρ„ΠΎΡ€ΠΌΡƒΠ»Π΅ (21.4) ΠΈΠΌΠ΅Π΅ΠΌ для ΠΏΠ΅Ρ€Π²ΠΎΠΉ ΠΏΡ€ΠΎΠ²ΠΎΠ»ΠΎΠΊΠΈ Π² Π·Π°Π΄Π°Ρ‡Π΅ 21.1.6

Π³Π΄Π΅ β€” ΡƒΠ΄Π΅Π»ΡŒΠ½ΠΎΠ΅ сопротивлСниС ΠΌΠ΅Π΄ΠΈ, β€” Π΄Π»ΠΈΠ½Π° ΠΏΡ€ΠΎΠ²ΠΎΠ΄Π½ΠΈΠΊΠ°, β€” Π΅Π³ΠΎ радиус. Для ΠΌΠ΅Π΄Π½ΠΎΠΉ ΠΏΡ€ΠΎΠ²ΠΎΠ»ΠΎΠΊΠΈ с Π²Π΄Π²ΠΎΠ΅ большСй Π΄Π»ΠΈΠ½ΠΎΠΉ ΠΈ Π²Ρ‚Ρ€ΠΎΠ΅ бóльшим радиусом сСчСния ΠΈΠΌΠ΅Π΅ΠΌ

(ΠΎΡ‚Π²Π΅Ρ‚ 3).

Как слСдуСт ΠΈΠ· Ρ„ΠΎΡ€ΠΌΡƒΠ»Ρ‹ (21.4) ΠΏΡ€ΠΈ Π΄Π²ΡƒΠΊΡ€Π°Ρ‚Π½ΠΎΠΌ ΡƒΠΌΠ΅Π½ΡŒΡˆΠ΅Π½ΠΈΠΈ Π΄Π»ΠΈΠ½Ρ‹ ΠΏΡ€ΠΎΠ²ΠΎΠ΄Π½ΠΈΠΊΠ° Π²Π΄Π²ΠΎΠ΅ ΡƒΠΌΠ΅Π½ΡŒΡˆΠ°Π΅Ρ‚ΡΡ Π΅Π³ΠΎ сопротивлСниС. ΠŸΠΎΡΡ‚ΠΎΠΌΡƒ ΠΈΠ· Π·Π°ΠΊΠΎΠ½Π° Ома для участка Ρ†Π΅ΠΏΠΈ (21.3) Π·Π°ΠΊΠ»ΡŽΡ‡Π°Π΅ΠΌ, Ρ‡Ρ‚ΠΎ ΠΏΡ€ΠΈ Π΄Π²ΡƒΠΊΡ€Π°Ρ‚Π½ΠΎΠΌ ΡƒΠΌΠ΅Π½ΡŒΡˆΠ΅Π½ΠΈΠΈ напряТСния Π½Π° ΠΏΡ€ΠΎΠ²ΠΎΠ΄Π½ΠΈΠΊΠ΅ ΠΈ Π΄Π²ΡƒΠΊΡ€Π°Ρ‚Π½ΠΎΠΌ ΡƒΠΌΠ΅Π½ΡŒΡˆΠ΅Π½ΠΈΠΈ Π΅Π³ΠΎ Π΄Π»ΠΈΠ½Ρ‹ (Π·Π°Π΄Π°Ρ‡Π° 21.1.7) сила Ρ‚ΠΎΠΊΠ° Π² ΠΏΡ€ΠΎΠ²ΠΎΠ΄Π½ΠΈΠΊΠ΅ Π½Π΅ измСнится (ΠΎΡ‚Π²Π΅Ρ‚ 4).

Π’ Π·Π°Π΄Π°Ρ‡Π΅ 21.1.8 слСдуСт ΠΈΡΠΏΠΎΠ»ΡŒΠ·ΠΎΠ²Π°Ρ‚ΡŒ Π·Π°ΠΊΠΎΠ½ Ома для Π·Π°ΠΌΠΊΠ½ΡƒΡ‚ΠΎΠΉ элСктричСской Ρ†Π΅ΠΏΠΈ (21.7). ИмССм

Π³Π΄Π΅ β€” Π­Π”Π‘ источника, β€” сопротивлСнии Π΅ внСшнСй Ρ†Π΅ΠΏΠΈ, β€” сопротивлСниС источника (ΠΎΡ‚Π²Π΅Ρ‚ 1).

Π’ Π·Π°Π΄Π°Ρ‡Π΅ 21.1.9 слСдуСт ΠΏΡ€ΠΈΠΌΠ΅Π½ΠΈΡ‚ΡŒ Π·Π°ΠΊΠΎΠ½ Ома для Π·Π°ΠΌΠΊΠ½ΡƒΡ‚ΠΎΠΉ элСктричСской Ρ†Π΅ΠΏΠΈ (21.7) ΠΊ ΠΊΠ°ΠΊΠΎΠΌΡƒ-Π½ΠΈΠ±ΡƒΠ΄ΡŒ Π·Π½Π°Ρ‡Π΅Π½ΠΈΡŽ внСшнСго сопротивлСния, ΠΏΠΎ Π³Ρ€Π°Ρ„ΠΈΠΊΡƒ Π½Π°ΠΉΡ‚ΠΈ силу Ρ‚ΠΎΠΊΠ° Π² Ρ†Π΅ΠΏΠΈ, Π° Π·Π°Ρ‚Π΅ΠΌ ΠΈ Π­Π”Π‘ источника. ΠŸΡ€ΠΎΡ‰Π΅ всСго ΠΏΡ€ΠΈΠΌΠ΅Π½ΠΈΡ‚ΡŒ Π·Π°ΠΊΠΎΠ½ Ома ΠΊ ΡΠ»ΡƒΡ‡Π°ΡŽ . Из Π³Ρ€Π°Ρ„ΠΈΠΊΠ° Π½Π°Ρ…ΠΎΠ΄ΠΈΠΌ силу Ρ‚ΠΎΠΊΠ° . ΠŸΠΎΡΡ‚ΠΎΠΌΡƒ

Π³Π΄Π΅ β€” Π²Π½ΡƒΡ‚Ρ€Π΅Π½Π½Π΅Π΅ сопротивлСниС источника (ΠΎΡ‚Π²Π΅Ρ‚ 3).

Из Ρ„ΠΎΡ€ΠΌΡƒΠ»Ρ‹ (21.9) слСдуСт, Ρ‡Ρ‚ΠΎ ΠΏΡ€ΠΈ фиксированном сопротивлСнии участка Ρ†Π΅ΠΏΠΈ ΡƒΠ²Π΅Π»ΠΈΡ‡Π΅Π½ΠΈΠ΅ элСктричСского напряТСния Π² 2 Ρ€Π°Π·Π° (Π·Π°Π΄Π°Ρ‡Π° 21.1.10) ΠΏΡ€ΠΈΠ²Π΅Π΄Π΅Ρ‚ ΠΊ ΡƒΠ²Π΅Π»ΠΈΡ‡Π΅Π½ΠΈΡŽ мощности Ρ‚ΠΎΠΊΠ° Π² 4 Ρ€Π°Π·Π° (ΠΎΡ‚Π²Π΅Ρ‚ 2).

Π’ Π·Π°Π΄Π°Ρ‡Π΅ 21.2.1 ΡƒΠ΄ΠΎΠ±Π½ΠΎ ΠΈΡΠΏΠΎΠ»ΡŒΠ·ΠΎΠ²Π°Ρ‚ΡŒ Π²Ρ‚ΠΎΡ€ΡƒΡŽ ΠΈΠ· Ρ„ΠΎΡ€ΠΌΡƒΠ» (21.9) . ИмССм Π’Ρ‚ (ΠΎΡ‚Π²Π΅Ρ‚ 3).

Часто школьники Π½Π΅ ΠΌΠΎΠ³ΡƒΡ‚ ΠΎΡ‚Π²Π΅Ρ‚ΠΈΡ‚ΡŒ Π½Π° Ρ‚Π°ΠΊΠΎΠΉ вопрос: ΠΈΠ· Ρ„ΠΎΡ€ΠΌΡƒΠ»Ρ‹ для мощности Ρ‚ΠΎΠΊΠ° слСдуСт, Ρ‡Ρ‚ΠΎ ΠΌΠΎΡ‰Π½ΠΎΡΡ‚ΡŒ Π»ΠΈΠ½Π΅ΠΉΠ½ΠΎ растСт с ростом сопротивлСния, Π° ΠΈΠ· Ρ„ΠΎΡ€ΠΌΡƒΠ»Ρ‹ β€” ΡƒΠ±Ρ‹Π²Π°Π΅Ρ‚ с ростом сопротивлСния. А ΠΊΠ°ΠΊ ΠΆΠ΅ Π² Π΄Π΅ΠΉΡΡ‚Π²ΠΈΡ‚Π΅Π»ΡŒΠ½ΠΎΡΡ‚ΠΈ ΠΌΠΎΡ‰Π½ΠΎΡΡ‚ΡŒ зависит ΠΎΡ‚ сопротивлСния? Π”Π°Π²Π°ΠΉΡ‚Π΅ разбСрСмся Π² этом вопросС Π½Π° ΠΏΡ€ΠΈΠΌΠ΅Ρ€Π΅ Π·Π°Π΄Π°Ρ‡ΠΈ 21.2.2. ΠšΠΎΠ½Π΅Ρ‡Π½ΠΎ, ΠΎΠ±Π° ΠΏΡ€Π΅Π΄Π»ΠΎΠΆΠ΅Π½Π½Ρ‹Ρ… Β«Ρ€Π΅ΡˆΠ΅Π½ΠΈΡΒ» Π½Π΅ΠΏΡ€Π°Π²ΠΈΠ»ΡŒΠ½Ρ‹: Π² Π½ΠΈΡ… ΠΌΠΎΠ»Ρ‡Π°Π»ΠΈΠ²ΠΎ ΠΏΡ€Π΅Π΄ΠΏΠΎΠ»Π°Π³Π°Π»ΠΎΡΡŒ, Ρ‡Ρ‚ΠΎ сила Ρ‚ΠΎΠΊΠ°, Ρ‚Π΅ΠΊΡƒΡ‰Π΅Π³ΠΎ Ρ‡Π΅Ρ€Π΅Π· это сопротивлСниС, ΠΈΠ»ΠΈ напряТСниС Π½Π° этом сопротивлСнии Π½Π΅ зависят ΠΎΡ‚ Π΅Π³ΠΎ Π²Π΅Π»ΠΈΡ‡ΠΈΠ½Ρ‹. А Π½Π° самом Π΄Π΅Π»Π΅ эти Π²Π΅Π»ΠΈΡ‡ΠΈΠ½Ρ‹ ΠΎΡ‚ сопротивлСния зависят, ΠΏΡ€ΠΈΡ‡Π΅ΠΌ эти зависимости ΠΌΠΎΠ³ΡƒΡ‚ Π±Ρ‹Ρ‚ΡŒ Ρ€Π°Π·Π½Ρ‹ΠΌΠΈ для Ρ€Π°Π·Π½Ρ‹Ρ… источников Ρ‚ΠΎΠΊΠ°. Π’Π½ΡƒΡ‚Ρ€Π΅Π½Π½Π΅Π΅ сопротивлСниС Π±Ρ‹Ρ‚ΠΎΠ²Ρ‹Ρ… элСктричСских сСтСй ΠΎΡ‡Π΅Π½ΡŒ ΠΌΠ°Π»ΠΎ. Π’ этом случаС ΠΈΠ· Π·Π°ΠΊΠΎΠ½ΠΎΠ² Ома для Π·Π°ΠΌΠΊΠ½ΡƒΡ‚ΠΎΠΉ Ρ†Π΅ΠΏΠΈ ΠΈ участка Ρ†Π΅ΠΏΠΈ (21.7), (21.3) слСдуСт, Ρ‡Ρ‚ΠΎ напряТСниС Π½Π° любом элСмСнтС, Π²ΠΊΠ»ΡŽΡ‡Π΅Π½Π½ΠΎΠΌ Π² Ρ‚Π°ΠΊΡƒΡŽ ΡΠ΅Ρ‚ΡŒ, Π½Π΅ зависит ΠΎΡ‚ сопротивлСния этого элСмСнта ΠΈ Ρ€Π°Π²Π½ΠΎ Π½ΠΎΠΌΠΈΠ½Π°Π»ΡŒΠ½ΠΎΠΌΡƒ Π½Π°ΠΏΡ€ΡΠΆΠ΅Π½ΠΈΡŽ сСти . ΠŸΠΎΡΡ‚ΠΎΠΌΡƒ ΠΈΠ· Ρ„ΠΎΡ€ΠΌΡƒΠ»Ρ‹ Π·Π°ΠΊΠ»ΡŽΡ‡Π°Π΅ΠΌ, Ρ‡Ρ‚ΠΎ ΠΌΠΎΡ‰Π½ΠΎΡΡ‚ΡŒ, которая выдСляСтся Π½Π° Ρ‚Π°ΠΊΠΎΠΌ элСмСнтС ΠΎΠ±Ρ€Π°Ρ‚Π½ΠΎ ΠΏΡ€ΠΎΠΏΠΎΡ€Ρ†ΠΈΠΎΠ½Π°Π»ΡŒΠ½Π° Π΅Π³ΠΎ ΡΠΎΠΏΡ€ΠΎΡ‚ΠΈΠ²Π»Π΅Π½ΠΈΡŽ (ΠΎΡ‚Π²Π΅Ρ‚ 3). ΠžΡ‚ΠΌΠ΅Ρ‚ΠΈΠΌ, Ρ‡Ρ‚ΠΎ ΠΈΠ· ΠΏΡ€ΠΎΠ²Π΅Π΄Π΅Π½Π½ΠΎΠ³ΠΎ рассуТдСния слСдуСт, Ρ‡Ρ‚ΠΎ выдСляСмая ΠΌΠΎΡ‰Π½ΠΎΡΡ‚ΡŒ Π±ΡƒΠ΄Π΅Ρ‚ ΠΎΡ‡Π΅Π½ΡŒ большой (опасная Π² Π±Ρ‹Ρ‚Ρƒ ситуация!) для ΠΌΠ°Π»ΠΎΠ³ΠΎ сопротивлСния внСшнСго участка Ρ†Π΅ΠΏΠΈ, Ρ‚.Π΅. Π² случаС ΠΊΠΎΡ€ΠΎΡ‚ΠΊΠΎΠ³ΠΎ замыкания, ΠΊΠΎΡ‚ΠΎΡ€ΠΎΠ³ΠΎ, Ρ‚Π°ΠΊΠΈΠΌ ΠΎΠ±Ρ€Π°Π·ΠΎΠΌ, Π½Π΅ΠΎΠ±Ρ…ΠΎΠ΄ΠΈΠΌΠΎ ΠΈΠ·Π±Π΅Π³Π°Ρ‚ΡŒ.

Если Π±Ρ‹ Π²Π½ΡƒΡ‚Ρ€Π΅Π½Π½Π΅Π΅ сопротивлСниС источника Π±Ρ‹Π»ΠΎ Π±Ρ‹ ΠΌΠ½ΠΎΠ³ΠΎ большС внСшнСго сопротивлСния, Ρ‚ΠΎΠΊ Π² Ρ†Π΅ΠΏΠΈ опрСдСлялся Π±Ρ‹, Π³Π»Π°Π²Π½Ρ‹ΠΌ ΠΎΠ±Ρ€Π°Π·ΠΎΠΌ, Π²Π½ΡƒΡ‚Ρ€Π΅Π½Π½ΠΈΠΌ сопротивлСниСм источника, Π° ΠΎΡ‚ внСшнСго сопротивлСния зависСл Π±Ρ‹ слабо. Π’ этом случаС ΠΌΠΎΡ‰Π½ΠΎΡΡ‚ΡŒ Ρ‚ΠΎΠΊΠ° Π±Ρ‹Π»Π° Π±Ρ‹ прямо ΠΏΡ€ΠΎΠΏΠΎΡ€Ρ†ΠΈΠΎΠ½Π°Π»ΡŒΠ½Π° ΡΠΎΠΏΡ€ΠΎΡ‚ΠΈΠ²Π»Π΅Π½ΠΈΡŽ участка Ρ†Π΅ΠΏΠΈ.

Как ΠΎΠ±ΡΡƒΠΆΠ΄Π°Π»ΠΎΡΡŒ Π² Ρ€Π΅ΡˆΠ΅Π½ΠΈΠΈ ΠΏΡ€Π΅Π΄Ρ‹Π΄ΡƒΡ‰Π΅ΠΉ Π·Π°Π΄Π°Ρ‡ΠΈ, сопротивлСниС элСмСнта, Ρ€Π°Π±ΠΎΡ‚Π°ΡŽΡ‰Π΅Π³ΠΎ Π² Π±Ρ‹Ρ‚ΠΎΠ²ΠΎΠΉ элСктросСти Ρ€Π°Π²Π½ΠΎ , Π³Π΄Π΅ β€” номинальная ΠΌΠΎΡ‰Π½ΠΎΡΡ‚ΡŒ Π΄Π°Π½Π½ΠΎΠ³ΠΎ элСмСнта, β€” напряТСниС Π² сСти. ΠŸΠΎΡΡ‚ΠΎΠΌΡƒ ΠΎΡ‚Π½ΠΎΡˆΠ΅Π½ΠΈΠ΅ сопротивлСний Π»Π°ΠΌΠΏ ΠΌΠΎΡ‰Π½ΠΎΡΡ‚ΡŒΡŽ Π’Ρ‚ ΠΈ Π’Ρ‚, рассчитанных Π½Π° Ρ€Π°Π±ΠΎΡ‚Ρƒ Π² ΠΎΠ΄Π½ΠΎΠΉ ΠΈ Ρ‚ΠΎΠΉ ΠΆΠ΅ Π±Ρ‹Ρ‚ΠΎΠ²ΠΎΠΉ элСктричСской сСти (Π·Π°Π΄Π°Ρ‡Π° 21.2.3) Ρ€Π°Π²Π½ΠΎ

(ΠΎΡ‚Π²Π΅Ρ‚ 2).

ΠŸΠΎΡΠΊΠΎΠ»ΡŒΠΊΡƒ рСзисторы Π² Π·Π°Π΄Π°Ρ‡Π΅ 21.2.4 соСдинСны ΠΏΠΎΡΠ»Π΅Π΄ΠΎΠ²Π°Ρ‚Π΅Π»ΡŒΠ½ΠΎ, Ρ‚ΠΎ сила Ρ‚ΠΎΠΊΠ° Π² Π½ΠΈΡ… ΠΎΠ΄ΠΈΠ½Π°ΠΊΠΎΠ²Π°. ΠŸΠΎΡΡ‚ΠΎΠΌΡƒ ΠΈΠ· Π·Π°ΠΊΠΎΠ½Π° Ома для участка Ρ†Π΅ΠΏΠΈ Π·Π°ΠΊΠ»ΡŽΡ‡Π°Π΅ΠΌ, Ρ‡Ρ‚ΠΎ

(ΠΎΡ‚Π²Π΅Ρ‚ 2).

ΠŸΡ€ΠΈ ΠΏΠ°Ρ€Π°Π»Π»Π΅Π»ΡŒΠ½ΠΎΠΌ соСдинСнии Π»Π°ΠΌΠΏ (Π·Π°Π΄Π°Ρ‡Π° 21.2.5) напряТСниС Π½Π° Π½ΠΈΡ… ΠΎΠ΄ΠΈΠ½Π°ΠΊΠΎΠ²ΠΎ (см. Π²Π²Π΅Π΄Π΅Π½ΠΈΠ΅ ΠΊ настоящСй Π³Π»Π°Π²Π΅). ΠŸΠΎΡΡ‚ΠΎΠΌΡƒ ΠΈΠ· Π·Π°ΠΊΠΎΠ½Π° Ома для участка Ρ†Π΅ΠΏΠΈ слСдуСт, Ρ‡Ρ‚ΠΎ

(ΠΎΡ‚Π²Π΅Ρ‚ 1).

РассматриваСмый Π² Π·Π°Π΄Π°Ρ‡Π΅ 21.2.6 участок прСдставляСт собой Π΄Π²Π° ΠΏΠΎΡΠ»Π΅Π΄ΠΎΠ²Π°Ρ‚Π΅Π»ΡŒΠ½Ρ‹Ρ… соСдинСнных элСмСнта, ΠΎΠ΄ΠΈΠ½ ΠΈΠ· ΠΊΠΎΡ‚ΠΎΡ€Ρ‹Ρ… Π΅ΡΡ‚ΡŒ рСзистор 6 Ом, Π²Ρ‚ΠΎΡ€ΠΎΠΉ β€” Π΄Π²Π° Ρ‚Π°ΠΊΠΈΡ… ΠΆΠ΅ рСзистора, соСдинСнных ΠΏΠ°Ρ€Π°Π»Π»Π΅Π»ΡŒΠ½ΠΎ. По ΠΏΡ€Π°Π²ΠΈΠ»Π°ΠΌ слоТСния сопротивлСний Π½Π°Ρ…ΠΎΠ΄ΠΈΠΌ эквивалСнтноС сопротивлСниС Π²Ρ‚ΠΎΡ€ΠΎΠ³ΠΎ участка

Π° Π·Π°Ρ‚Π΅ΠΌ ΠΈ эквивалСнтноС сопротивлСниС всСй Ρ†Π΅ΠΏΠΈ

(ΠΎΡ‚Π²Π΅Ρ‚ 3).

ΠŸΡ€ΠΈ Ρ€Π°Π·ΠΎΠΌΠΊΠ½ΡƒΡ‚ΠΎΠΌ ΠΊΠ»ΡŽΡ‡Π΅ сопротивлСниС участка Ρ†Π΅ΠΏΠΈ, Π΄Π°Π½Π½ΠΎΠ³ΠΎ Π² Π·Π°Π΄Π°Ρ‡Π΅ 21.2.7, ΠΌΠΎΠΆΠ½ΠΎ Π½Π°ΠΉΡ‚ΠΈ ΠΊΠ°ΠΊ Π² ΠΏΡ€Π΅Π΄Ρ‹Π΄ΡƒΡ‰Π΅ΠΉ Π·Π°Π΄Π°Ρ‡Π΅ , Π³Π΄Π΅ β€” сопротивлСниС ΠΊΠ°ΠΆΠ΄ΠΎΠ³ΠΎ рСзистора. Если ΠΊΠ»ΡŽΡ‡ Π·Π°ΠΌΠΊΠ½ΡƒΡ‚, Ρ‚ΠΎ Ρ†Π΅ΠΏΡŒ сводится ΠΊ ΠΎΠ΄Π½ΠΎΠΌΡƒ рСзистору (Ρ‚.ΠΊ. ΠΏΠ°Ρ€Π°Π»Π»Π΅Π»ΡŒΠ½ΠΎ Π΄Π²ΡƒΠΌ рСзисторам Π²ΠΊΠ»ΡŽΡ‡Π°Π΅Ρ‚ΡΡ ΠΏΡ€ΠΎΠ²ΠΎΠ΄Π½ΠΈΠΊ с ΠΏΡ€Π΅Π½Π΅Π±Ρ€Π΅ΠΆΠΈΠΌΠΎ ΠΌΠ°Π»Ρ‹ΠΌ сопротивлСниСм). ΠŸΠΎΡΡ‚ΠΎΠΌΡƒ Π² этом случаС сопротивлСниС Ρ†Π΅ΠΏΠΈ Ρ€Π°Π²Π½ΠΎ . Π’Π°ΠΊΠΈΠΌ ΠΎΠ±Ρ€Π°Π·ΠΎΠΌ, сопротивлСниС Π²Ρ‚ΠΎΡ€ΠΎΠΉ Ρ†Π΅ΠΏΠΈ составляСт Π΄Π²Π΅ Ρ‚Ρ€Π΅Ρ‚ΠΈ ΠΎΡ‚ сопротивлСния ΠΏΠ΅Ρ€Π²ΠΎΠΉ (ΠΎΡ‚Π²Π΅Ρ‚ 1).

Как ΠΎΠ±ΡΡƒΠΆΠ΄Π°Π»ΠΎΡΡŒ Π² Ρ€Π΅ΡˆΠ΅Π½ΠΈΠΈ Π·Π°Π΄Π°Ρ‡ΠΈ 21.2.2, сопротивлСниС элСмСнта номинальной мощности , Ρ€Π°Π±ΠΎΡ‚Π°ΡŽΡ‰Π΅Π³ΠΎ Π² Π±Ρ‹Ρ‚ΠΎΠ²ΠΎΠΉ элСктросСти Ρ€Π°Π²Π½Π°

Π³Π΄Π΅ Π’ β€” напряТСниС сСти. Из этой Ρ„ΠΎΡ€ΠΌΡƒΠ»Ρ‹ слСдуСт, Ρ‡Ρ‚ΠΎ Ρ‡Π΅ΠΌ большС номинальная ΠΌΠΎΡ‰Π½ΠΎΡΡ‚ΡŒ элСмСнта, Ρ‚Π΅ΠΌ мСньшС Π΄ΠΎΠ»ΠΆΠ½ΠΎ Π±Ρ‹Ρ‚ΡŒ Π΅Π³ΠΎ сопротивлСниС. Если Π΄Π²Π΅ Π»Π°ΠΌΠΏΡ‹ накаливания Π²ΠΊΠ»ΡŽΡ‡Π΅Π½Ρ‹ ΠΏΠΎΡΠ»Π΅Π΄ΠΎΠ²Π°Ρ‚Π΅Π»ΡŒΠ½ΠΎ (Π·Π°Π΄Π°Ρ‡Π° 21.2.8), Ρ‚ΠΎ сила Ρ‚ΠΎΠΊΠ° Π² Π½ΠΈΡ… ΠΎΠ΄ΠΈΠ½Π°ΠΊΠΎΠ²Π° ΠΈ ΠΎΡ‚Π½ΠΎΡˆΠ΅Π½ΠΈΠ΅ мощностСй Ρ‚ΠΎΠΊΠ° Π² этих Π»Π°ΠΌΠΏΠ°Ρ… Ρ€Π°Π²Π½ΠΎ ΠΎΡ‚Π½ΠΎΡˆΠ΅Π½ΠΈΡŽ ΠΈΡ… сопротивлСний. ΠžΡ‚ΡΡŽΠ΄Π° слСдуСт, Ρ‡Ρ‚ΠΎ ΠΎΡ‚Π½ΠΎΡˆΠ΅Π½ΠΈΠ΅ Ρ€Π΅Π°Π»ΡŒΠ½ΠΎ выдСляСмых Π² Π»Π°ΠΌΠΏΠ°Ρ… мощностСй ΠΈ ΠΎΠ±Ρ€Π°Ρ‚Π½ΠΎ ΠΎΡ‚Π½ΠΎΡˆΠ΅Π½ΠΈΡŽ Π½ΠΎΠΌΠΈΠ½Π°Π»ΡŒΠ½Ρ‹Ρ… мощностСй этих Π»Π°ΠΌΠΏ:

(ΠΎΡ‚Π²Π΅Ρ‚ 2).

Π Π°Π±ΠΎΡ‚Π°, ΡΠΎΠ²Π΅Ρ€ΡˆΠ°Π΅ΠΌΠ°Ρ элСктричСским ΠΏΠΎΠ»Π΅ΠΌ Π² ΠΏΡ€ΠΎΠ²ΠΎΠ΄Π½ΠΈΠΊΠ΅ ΠΏΡ€ΠΈ ΠΏΡ€ΠΎΡ‚Π΅ΠΊΠ°Π½ΠΈΠΈ ΠΏΠΎ Π½Π΅ΠΌΡƒ элСктричСского Ρ‚ΠΎΠΊΠ°, прСвращаСтся Π² ΡΠ½Π΅Ρ€Π³ΠΈΡŽ Ρ‚ΠΎΠΊΠ°, которая Π·Π°Ρ‚Π΅ΠΌ прСвращаСтся Π² Ρ‚Π΅ΠΏΠ»ΠΎΠ²ΡƒΡŽ ΡΠ½Π΅Ρ€Π³ΠΈΡŽ. ΠŸΠΎΡΡ‚ΠΎΠΌΡƒ Ρ€Π°Π±ΠΎΡ‚Ρƒ поля ΠΌΠΎΠΆΠ½ΠΎ Π½Π°ΠΉΡ‚ΠΈ ΠΈΠ· Π·Π°ΠΊΠΎΠ½Π° ДТоуля-Π›Π΅Π½Ρ†Π°. Для Ρ€Π°Π±ΠΎΡ‚Ρ‹ поля Π·Π° врСмя ΠΏΠΎΠ»ΡƒΡ‡Π°Π΅ΠΌ . Из этой Ρ„ΠΎΡ€ΠΌΡƒΠ»Ρ‹ Π½Π°Ρ…ΠΎΠ΄ΠΈΠΌ сопротивлСниС ΠΏΡ€ΠΎΠ²ΠΎΠ΄Π½ΠΈΠΊΠ° Π² Π·Π°Π΄Π°Ρ‡Π΅ 21.2.9 β€”

(ΠΎΡ‚Π²Π΅Ρ‚ 1).

ΠŸΠΎΡΠΊΠΎΠ»ΡŒΠΊΡƒ ΠΏΡ€ΠΈ ΠΏΠΎΡΠ»Π΅Π΄ΠΎΠ²Π°Ρ‚Π΅Π»ΡŒΠ½ΠΎΠΌ соСдинСнии рСзисторов Ρ‚ΠΎΠΊ Ρ‡Π΅Ρ€Π΅Π· ΠΊΠ°ΠΆΠ΄Ρ‹ΠΉ ΠΈΠ· Π½ΠΈΡ… ΠΎΠ΄ΠΈΠ½Π°ΠΊΠΎΠ², ΠΈΠ· Π·Π°ΠΊΠΎΠ½Π° ДТоуля-Π›Π΅Π½Ρ†Π° (22.8) Π·Π°ΠΊΠ»ΡŽΡ‡Π°Π΅ΠΌ, Ρ‡Ρ‚ΠΎ ΠΈΠ· Π΄Π²ΡƒΡ… сопротивлСний ΠΈ (Π·Π°Π΄Π°Ρ‡Π° 21.2.10; см. рисунок) наибольшСй Π±ΡƒΠ΄Π΅Ρ‚ ΠΌΠΎΡ‰Π½ΠΎΡΡ‚ΡŒ Ρ‚ΠΎΠΊΠ° Π½Π° сопротивлСнии , ΠΈΠ· Π΄Π²ΡƒΡ… сопротивлСний ΠΈ β€” Π½Π° сопротивлСнии . Π‘Ρ€Π°Π²Π½ΠΈΠΌ мощности Ρ‚ΠΎΠΊΠ° Π½Π° этих сопротивлСниях. Учитывая, Ρ‡Ρ‚ΠΎ ΠΏΡ€ΠΈ ΠΏΠ°Ρ€Π°Π»Π»Π΅Π»ΡŒΠ½ΠΎΠΌ соСдинСнии элСмСнтов элСктричСскоС напряТСниС Π½Π° ΠΊΠ°ΠΆΠ΄ΠΎΠΌ элСмСнтС ΠΎΠ΄ΠΈΠ½Π°ΠΊΠΎΠ²ΠΎΠ΅, Π° ΠΏΡ€ΠΈ ΠΏΠΎΡΠ»Π΅Π΄ΠΎΠ²Π°Ρ‚Π΅Π»ΡŒΠ½ΠΎΠΌ β€” ΡΠΊΠ»Π°Π΄Ρ‹Π²Π°ΡŽΡ‚ΡΡ значСния сопротивлСний, ΠΏΠΎΠ»ΡƒΡ‡ΠΈΠΌ ΠΈΠ· Π·Π°ΠΊΠΎΠ½ΠΎΠ² Ома для Π²Π΅Ρ€Ρ…Π½Π΅Π³ΠΎ ΠΈ Π½ΠΈΠΆΠ½Π΅Π³ΠΎ участков Ρ†Π΅ΠΏΠΈ ΠΈ Π·Π°ΠΊΠΎΠ½Π° ДТоуля-Π›Π΅Π½Ρ†Π°

Π³Π΄Π΅ β€” элСктричСскоС напряТСниС, ΠΏΡ€ΠΈΠ»ΠΎΠΆΠ΅Π½Π½ΠΎΠ΅ ΠΊΠΎ всСй Ρ†Π΅ΠΏΠΈ. ΠŸΠΎΡΠΊΠΎΠ»ΡŒΠΊΡƒ Ρ‚ΠΎ Π² прСдставлСнной схСмС наибольшая ΠΌΠΎΡ‰Π½ΠΎΡΡ‚ΡŒ Π±ΡƒΠ΄Π΅Ρ‚ Π²Ρ‹Π΄Π΅Π»ΡΡ‚ΡŒΡΡ Π½Π° сопротивлСнии (ΠΎΡ‚Π²Π΅Ρ‚ 2).

Ρ‡Π΅ΠΌΡƒ Ρ€Π°Π²Π½ΠΎ ΠΏΡ€ΠΈ ΠΏΠ°Ρ€Π°Π»Π»Π΅Π»ΡŒΠ½ΠΎΠΌ ΠΈ ΠΏΠΎΡΠ»Π΅Π΄ΠΎΠ²Π°Ρ‚Π΅Π»ΡŒΠ½ΠΎΠΌ соСдинСнии, Π·Π°ΠΊΠΎΠ½ Ома

Π§Ρ‚ΠΎ Ρ‚Π°ΠΊΠΎΠ΅ элСктричСскоС сопротивлСниС участка Ρ†Π΅ΠΏΠΈ

ЭлСктричСскоС сопротивлСниС для участка Ρ†Π΅ΠΏΠΈ опрСдСляСтся ΠΏΡ€ΠΈ ΠΏΠΎΠΌΠΎΡ‰ΠΈ Π·Π°ΠΊΠΎΠ½Π° Ома. Для Ρ‚ΠΎΠ³ΠΎ, Ρ‡Ρ‚ΠΎΠ±Ρ‹ ΠΏΠΎΠ½ΡΡ‚ΡŒ процСссы, происходящиС Π² элСмСнтах элСктричСской Ρ†Π΅ΠΏΠΈ постоянного Ρ‚ΠΎΠΊΠ°, Π½Π΅ΠΎΠ±Ρ…ΠΎΠ΄ΠΈΠΌΠΎ Π΄Π°Ρ‚ΡŒ ΠΎΠ±Ρ‰Π΅Π΅ ΠΎΠΏΡ€Π΅Π΄Π΅Π»Π΅Π½ΠΈΠ΅ Π·Π°ΠΊΠΎΠ½Π° Ома.

ΠžΠΏΡ€Π΅Π΄Π΅Π»Π΅Π½ΠΈΠ΅

Π—Π°ΠΊΠΎΠ½ Ома: сила Ρ‚ΠΎΠΊΠ° Π² участкС Ρ†Π΅ΠΏΠΈ прямо ΠΏΡ€ΠΎΠΏΠΎΡ€Ρ†ΠΈΠΎΠ½Π°Π»ΡŒΠ½Π° Π½Π°ΠΏΡ€ΡΠΆΠ΅Π½ΠΈΡŽ Π½Π° ΠΊΠΎΠ½Ρ†Π°Ρ… этого участка ΠΈ ΠΎΠ±Ρ€Π°Ρ‚Π½ΠΎ ΠΏΡ€ΠΎΠΏΠΎΡ€Ρ†ΠΈΠΎΠ½Π°Π»ΡŒΠ½Π° Π΅Π³ΠΎ ΡΠΎΠΏΡ€ΠΎΡ‚ΠΈΠ²Π»Π΅Π½ΠΈΡŽ.

I β€” сила Ρ‚ΠΎΠΊΠ°Β (Π΅Π΄ΠΈΠ½ΠΈΡ†Π° измСрСния β€” Π°ΠΌΠΏΠ΅Ρ€).

ΠžΡΡ‚ΠΎΡ€ΠΎΠΆΠ½ΠΎ! Если ΠΏΡ€Π΅ΠΏΠΎΠ΄Π°Π²Π°Ρ‚Π΅Π»ΡŒ ΠΎΠ±Π½Π°Ρ€ΡƒΠΆΠΈΡ‚ ΠΏΠ»Π°Π³ΠΈΠ°Ρ‚ Π² Ρ€Π°Π±ΠΎΡ‚Π΅, Π½Π΅ ΠΈΠ·Π±Π΅ΠΆΠ°Ρ‚ΡŒ ΠΊΡ€ΡƒΠΏΠ½Ρ‹Ρ… ΠΏΡ€ΠΎΠ±Π»Π΅ΠΌ (Π²ΠΏΠ»ΠΎΡ‚ΡŒ Π΄ΠΎ отчислСния). Если Π½Π΅Ρ‚ возмоТности Π½Π°ΠΏΠΈΡΠ°Ρ‚ΡŒ самому, Π·Π°ΠΊΠ°ΠΆΠΈΡ‚Π΅ Ρ‚ΡƒΡ‚.

\( I=\frac{U}{R}I=RU\)

U β€” напряТСниС (измСряСтся Π² Π²ΠΎΠ»ΡŒΡ‚Π°Ρ…). ПадСниС напряТСния Π½Π° участкС ΠΏΡ€ΠΎΠ²ΠΎΠ΄Π½ΠΈΠΊΠ° равняСтся ΠΏΡ€ΠΎΠΈΠ·Π²Π΅Π΄Π΅Π½ΠΈΡŽ силы Ρ‚ΠΎΠΊΠ° ΠΈ сопротивлСния Π½Π° участкС.

\(U=IRU=IR\)

RΒ β€” элСктричСскоС сопротивлСниС (измСряСтся Π² ΠΎΠΌΠ°Ρ…). ЭлСктричСскоС сопротивлСниС R это ΠΎΡ‚Π½ΠΎΡˆΠ΅Π½ΠΈΠ΅ напряТСния Π½Π° ΠΊΠΎΠ½Ρ†Π°Ρ… ΠΏΡ€ΠΎΠ²ΠΎΠ΄Π½ΠΈΠΊΠ° ΠΊ силС Ρ‚ΠΎΠΊΠ°.

\(R=\frac{U}{I}R=IU\)​

Как Π·Π°ΠΏΠΎΠΌΠ½ΠΈΡ‚ΡŒ Ρ„ΠΎΡ€ΠΌΡƒΠ»Ρ‹ Π·Π°ΠΊΠΎΠ½Π°

Для запоминания Π·Π°ΠΊΠΎΠ½Π°, Π΄Π΅ΠΉΡΡ‚Π²ΡƒΡŽΡ‰Π΅Π³ΠΎ Π½Π° участкС Ρ†Π΅ΠΏΠΈ, ΠΏΡ€ΠΈΠΌΠ΅Π½ΡΡŽΡ‚ Ρ‚Ρ€Π΅ΡƒΠ³ΠΎΠ»ΡŒΠ½ΠΈΠΊ Ома. Π‘ ΠΏΠΎΠΌΠΎΡ‰ΡŒΡŽ Π½Π΅Π³ΠΎ ΠΌΠΎΠΆΠ½ΠΎ ΠΈΠ·ΠΎΠ±Ρ€Π°Π·ΠΈΡ‚ΡŒ ΠΎΡ‚Π½ΠΎΡˆΠ΅Π½ΠΈΠ΅ Π²Π΅Π»ΠΈΡ‡ΠΈΠ½ Π΄Ρ€ΡƒΠ³ ΠΊ Π΄Ρ€ΡƒΠ³Ρƒ.

Β 

Если Π·Π°ΠΊΡ€Ρ‹Ρ‚ΡŒ ΠΎΠ΄ΠΈΠ½ ΠΈΠ· ΠΊΠΎΠΌΠΏΠΎΠ½Π΅Π½Ρ‚ΠΎΠ², ΠΎΡΡ‚Π°Π²ΡˆΠΈΠ΅ΡΡ ΠΏΠΎΠΊΠ°ΠΆΡƒΡ‚ способ Π΅Π΅ нахоТдСния. НапримСр, Π·Π°ΠΊΡ€Ρ‹Π² U, ΡƒΠ²ΠΈΠ΄ΠΈΠΌ I ΠΈ R, ΠΏΡ€ΠΎΠΈΠ·Π²Π΅Π΄Π΅Π½ΠΈΠ΅ ΠΊΠΎΡ‚ΠΎΡ€Ρ‹Ρ… ΠΏΠΎΠΊΠ°Π·Ρ‹Π²Π°Π΅Ρ‚ Π²Π΅Π»ΠΈΡ‡ΠΈΠ½Ρƒ напряТСния. Π’Π°ΠΊ ΠΆΠ΅ ΠΌΠΎΠΆΠ½ΠΎ ΠΏΠΎΡΡ‚ΡƒΠΏΠΈΡ‚ΡŒ ΠΈ с Π΄Ρ€ΡƒΠ³ΠΈΠΌΠΈ элСмСнтами Ρ‚Ρ€Π΅ΡƒΠ³ΠΎΠ»ΡŒΠ½ΠΈΠΊΠ°.

Π‘ΠΎΠΏΡ€ΠΎΡ‚ΠΈΠ²Π»Π΅Π½ΠΈΠ΅ ΠΏΡ€ΠΈ ΠΏΠ°Ρ€Π°Π»Π»Π΅Π»ΡŒΠ½ΠΎΠΌ ΠΈ ΠΏΠΎΡΠ»Π΅Π΄ΠΎΠ²Π°Ρ‚Π΅Π»ΡŒΠ½ΠΎΠΌ соСдинСнии

ΠŸΡ€ΠΈ ΠΏΠ°Ρ€Π°Π»Π»Π΅Π»ΡŒΠ½ΠΎΠΌ ΠΈ ΠΏΠΎΡΠ»Π΅Π΄ΠΎΠ²Π°Ρ‚Π΅Π»ΡŒΠ½ΠΎΠΌ соСдинСнии рСзисторов Π²Π΅Π»ΠΈΡ‡ΠΈΠ½Π° сопротивлСния Π±ΡƒΠ΄Π΅Ρ‚ Ρ€Π°Π·Π»ΠΈΡ‡Π°Ρ‚ΡŒΡΡ.

ΠŸΠΎΡΠ»Π΅Π΄ΠΎΠ²Π°Ρ‚Π΅Π»ΡŒΠ½ΠΎΠ΅ соСдинСниС прСдставляСт собой Ρ†Π΅ΠΏΡŒ, ΡΠΎΡΡ‚ΠΎΡΡ‰ΡƒΡŽ ΠΈΠ· ΠΏΠΎΠ΄ΠΊΠ»ΡŽΡ‡Π΅Π½Π½Ρ‹Ρ… ΠΎΠ΄Π½ΠΎΠ³ΠΎ Π·Π° Π΄Ρ€ΡƒΠ³ΠΈΠΌ ΠΏΡ€ΠΎΠ²ΠΎΠ΄Π½ΠΈΠΊΠΎΠ². ΠšΠΎΠ»ΠΈΡ‡Π΅ΡΡ‚Π²ΠΎ звСньСв Π½Π΅ ΠΎΠ³Ρ€Π°Π½ΠΈΡ‡Π΅Π½ΠΎ. На ΠΊΠ°ΠΆΠ΄ΠΎΠΌ участкС Ρ†Π΅ΠΏΠΈ ΠΌΠΎΠΆΠ½ΠΎ ΠΎΠΏΡ€Π΅Π΄Π΅Π»ΠΈΡ‚ΡŒ Π²Π½ΡƒΡ‚Ρ€Π΅Π½Π½Π΅Π΅ сопротивлСниС. Π‘ΠΎΠΏΡ€ΠΎΡ‚ΠΈΠ²Π»Π΅Π½ΠΈΠ΅ всСй Ρ†Π΅ΠΏΠΈ Π±ΡƒΠ΄Π΅Ρ‚ Ρ€Π°Π²Π½ΡΡ‚ΡŒΡΡ ΠΈΡ… суммС ΠΈ Ρ€Π°ΡΡΡ‡ΠΈΡ‚Ρ‹Π²Π°Ρ‚ΡŒΡΡ ΠΏΠΎ Π΄Π°Π½Π½ΠΎΠΉ Ρ„ΠΎΡ€ΠΌΡƒΠ»Π΅:

\(R_{AB}=R_A+R_B\)

ΠŸΠ°Ρ€Π°Π»Π»Π΅Π»ΡŒΠ½ΠΎΠ΅ соСдинСниС ΠΎΠ·Π½Π°Ρ‡Π°Π΅Ρ‚, Ρ‡Ρ‚ΠΎ всС участки Ρ†Π΅ΠΏΠΈ ΠΏΠΎΠ΄ΠΊΠ»ΡŽΡ‡Π΅Π½Ρ‹ ΠΊ ΠΎΠ΄Π½ΠΎΠΌΡƒ ΡƒΠ·Π»Ρƒ. НапряТСниС Π½Π° всСх элСмСнтах ΠΎΠ΄ΠΈΠ½Π°ΠΊΠΎΠ²ΠΎ, Π° Ρ‚ΠΎΠΊ развСтвляСтся ΠΈ Ρ€Π°Π²Π΅Π½ суммС Ρ‚ΠΎΠΊΠΎΠ² Π½Π° всСх участках. ΠŸΠΎΡΡ‚ΠΎΠΌΡƒ ΠΎΠ±Ρ‰Π΅Π΅ сопротивлСниС ΠΌΠΎΠΆΠ½ΠΎ Π²Ρ‹Ρ‡ΠΈΡΠ»ΠΈΡ‚ΡŒ ΠΏΠΎ Ρ„ΠΎΡ€ΠΌΡƒΠ»Π΅:Β 

\(R_ {AB}=\frac{R_1\times R_2}{R_1+R_2}.\)

Π—Π°ΠΊΠΎΠ½ ΠΎΠΌΠ° для участка Ρ†Π΅ΠΏΠΈ: Ρ„ΠΎΡ€ΠΌΡƒΠ»Π°, объяснСниС простыми словами

Основой ΠΏΡ€ΠΈΠΊΠ»Π°Π΄Π½ΠΎΠΉ элСктротСхники бСзусловно являСтся Π·Π°ΠΊΠΎΠ½ Ома для участка Ρ†Π΅ΠΏΠΈ. НС зная Π΅Π³ΠΎ основных ΠΏΠΎΠ»ΠΎΠΆΠ΅Π½ΠΈΠΉ, ΠΌΠΎΠΆΠ½ΠΎ Π΄ΠΎΠΏΡƒΡΡ‚ΠΈΡ‚ΡŒ ΡΠ΅Ρ€ΡŒΠ΅Π·Π½Ρ‹Π΅ ошибки Π² практичСской Ρ€Π°Π±ΠΎΡ‚Π΅. О самом физичСском явлСнии всСм извСстно Π΅Ρ‰Π΅ со ΡˆΠΊΠΎΠ»Ρ‹. Но с ΠΏΠΎΠΌΠΎΡ‰ΡŒΡŽ Π±Π°Π·ΠΎΠ²Ρ‹Ρ… ΠΏΠΎΠ»ΠΎΠΆΠ΅Π½ΠΈΠΉ ΠΌΠΎΠΆΠ΅Ρ‚ Ρ„ΠΎΡ€ΠΌΡƒΠ»ΠΈΡ€ΠΎΠ²Π°Ρ‚ΡŒΡΡ ΠΌΠ½ΠΎΠ³ΠΎ Π΄Ρ€ΡƒΠ³ΠΈΡ… Π²Π°Ρ€ΠΈΠ°Π½Ρ‚ΠΎΠ², Ρ‚Π°ΠΊ ΠΈΠ»ΠΈ ΠΈΠ½Π°Ρ‡Π΅ Π·Π°Ρ‚Ρ€Π°Π³ΠΈΠ²Π°ΡŽΡ‰ΠΈΡ… особСнности примСнСния Π·Π°ΠΊΠΎΠ½Π° Π² Ρ€Π΅Π°Π»ΡŒΠ½Ρ‹Ρ… условиях. ИмСнно здСсь Π²ΠΎΠ·Π½ΠΈΠΊΠ°ΡŽΡ‚ слоТности, Ρ‚Ρ€Π΅Π±ΡƒΡŽΡ‰ΠΈΠ΅ Ρ‚ΠΎΡ‡Π½Ρ‹Ρ… Π·Π½Π°Π½ΠΈΠΉ ΠΈ Π½Π°Π²Ρ‹ΠΊΠΎΠ² опСрирования физичСскими Π²Π΅Π»ΠΈΡ‡ΠΈΠ½Π°ΠΌΠΈ.

Как Π·Π²ΡƒΡ‡ΠΈΡ‚ Π·Π°ΠΊΠΎΠ½ Ома для участка Ρ†Π΅ΠΏΠΈ

Π’ΠΎΠΊ Π² ΠΏΡ€ΠΎΠ²ΠΎΠ΄Π½ΠΈΠΊΠ΅ Π²ΠΎΠ·Π½ΠΈΠΊΠ°Π΅Ρ‚ Π² элСктричСском ΠΏΠΎΠ»Π΅, ΠΊΠΎΡ‚ΠΎΡ€ΠΎΠ΅, Π² свою ΠΎΡ‡Π΅Ρ€Π΅Π΄ΡŒ, появляСтся ΠΏΡ€ΠΈ Π½Π°Π»ΠΈΡ‡ΠΈΠΈ разности ΠΏΠΎΡ‚Π΅Π½Ρ†ΠΈΠ°Π»ΠΎΠ² ΠΈΠ»ΠΈ напряТСния. Π”Π²ΠΈΠΆΠ΅Π½ΠΈΠ΅ Ρ‚ΠΎΠΊΠ° Π½Π°ΠΏΡ€Π°Π²Π»Π΅Π½ΠΎ Π² сторону мСньшСго ΠΏΠΎΡ‚Π΅Π½Ρ†ΠΈΠ°Π»Π°. Условно считаСтся, Ρ‡Ρ‚ΠΎ Π² этом Π½Π°ΠΏΡ€Π°Π²Π»Π΅Π½ΠΈΠΈ Π΄Π²ΠΈΠ³Π°ΡŽΡ‚ΡΡ ΠΏΠΎΠ»ΠΎΠΆΠΈΡ‚Π΅Π»ΡŒΠ½Ρ‹Π΅ заряды, Π° Π² ΠΎΠ±Ρ€Π°Ρ‚Π½ΡƒΡŽ сторону происходит Π΄Π²ΠΈΠΆΠ΅Π½ΠΈΠ΅ свободных элСктронов.

На участкС мСталличСского ΠΏΡ€ΠΎΠ²ΠΎΠ΄Π½ΠΈΠΊΠ° Π΄Π°Π½Π½Ρ‹ΠΉ процСсс Π±ΡƒΠ΄Π΅Ρ‚ Π²Ρ‹Π³Π»ΡΠ΄Π΅Ρ‚ΡŒ ΡΠ»Π΅Π΄ΡƒΡŽΡ‰ΠΈΠΌ ΠΎΠ±Ρ€Π°Π·ΠΎΠΌ. На ΠΊΠ°ΠΆΠ΄ΠΎΠΌ ΠΊΠΎΠ½Ρ†Π΅ присутствуСт ΠΏΠΎΡ‚Π΅Π½Ρ†ΠΈΠ°Π» – Ο•1 ΠΈ Ο•2, ΠΏΡ€ΠΈ этом Ο•1 > Ο•2. Π‘Π»Π΅Π΄ΠΎΠ²Π°Ρ‚Π΅Π»ΡŒΠ½ΠΎ, напряТСниС Π² этом мСстС Ρ€Π°Π²Π½ΠΎ U = Ο•1 – Ο•2. НСмСцкий ΡƒΡ‡Π΅Π½Ρ‹ΠΉ Ом практичСски установил Π·Π°Π²ΠΈΡΠΈΠΌΠΎΡΡ‚ΡŒ, ΠΏΡ€ΠΈ ΠΊΠΎΡ‚ΠΎΡ€ΠΎΠΉ с ΡƒΠ²Π΅Π»ΠΈΡ‡Π΅Π½ΠΈΠ΅ΠΌ напряТСния, возрастаСт ΠΈ сила Ρ‚ΠΎΠΊΠ°, ΠΏΡ€ΠΎΡ‚Π΅ΠΊΠ°ΡŽΡ‰Π΅Π³ΠΎ Ρ‡Π΅Ρ€Π΅Π· Π½Π΅ΠΏΠΎΠ»Π½Ρ‹ΠΉ участок.

Для ΠΊΠ°ΠΆΠ΄ΠΎΠ³ΠΎ ΠΈΠ· ΠΏΡ€ΠΎΠ²ΠΎΠ΄Π½ΠΈΠΊΠΎΠ², ΠΎΡ‚Π»ΠΈΡ‡Π°ΡŽΡ‰ΠΈΡ…ΡΡ ΠΌΠ°Ρ‚Π΅Ρ€ΠΈΠ°Π»Π°ΠΌΠΈ, Π±Ρ‹Π» построСн свой Π³Ρ€Π°Ρ„ΠΈΠΊ, ΠΎΡ‚Ρ€Π°ΠΆΠ°ΡŽΡ‰ΠΈΠΉ Π·Π°Π²ΠΈΡΠΈΠΌΠΎΡΡ‚ΡŒ силы Ρ‚ΠΎΠΊΠ° ΠΎΡ‚ напряТСния. Π’ дальнСйшСм, эти Π³Ρ€Π°Ρ„ΠΈΠΊΠΈ стали извСстны, ΠΊΠ°ΠΊ Π²ΠΎΠ»ΡŒΡ‚-Π°ΠΌΠΏΠ΅Ρ€Π½Ρ‹Π΅ характСристики. Π’ Ρ€Π΅Π·ΡƒΠ»ΡŒΡ‚Π°Ρ‚Π΅, Π±Ρ‹Π»ΠΎ установлСно Π½Π°Π»ΠΈΡ‡ΠΈΠ΅ Π»ΠΈΠ½Π΅ΠΉΠ½ΠΎΠΉ связи ΠΌΠ΅ΠΆΠ΄Ρƒ ΠΎΠ±Π΅ΠΈΠΌΠΈ Π²Π΅Π»ΠΈΡ‡ΠΈΠ½Π°ΠΌΠΈ – силой Ρ‚ΠΎΠΊΠ° ΠΈ напряТСниСм. Π’ΠΎ Π΅ΡΡ‚ΡŒ, ΠΎΠ½ΠΈ находятся Π² прямой ΠΏΡ€ΠΎΠΏΠΎΡ€Ρ†ΠΈΠΎΠ½Π°Π»ΡŒΠ½ΠΎΠΉ зависимости.

Но, ΠΊΠ°ΠΊ ΠΏΠΎΠΊΠ°Π·Ρ‹Π²Π°ΡŽΡ‚ Π³Ρ€Π°Ρ„ΠΈΠΊΠΈ, всС ΠΏΡ€ΠΎΠ²ΠΎΠ΄Π½ΠΈΠΊΠΈ ΠΎΠ±Π»Π°Π΄Π°ΡŽΡ‚ Ρ€Π°Π·Π½Ρ‹ΠΌΠΈ коэффициСнтами ΠΏΡ€ΠΎΠΏΠΎΡ€Ρ†ΠΈΠΎΠ½Π°Π»ΡŒΠ½ΠΎΡΡ‚ΠΈ. Π‘Π»Π΅Π΄ΠΎΠ²Π°Ρ‚Π΅Π»ΡŒΠ½ΠΎ, Ρƒ Π½ΠΈΡ… разная ΡΡ‚Π΅ΠΏΠ΅Π½ΡŒ проводимости, ΠΏΠΎΠ»ΡƒΡ‡ΠΈΠ²ΡˆΠ°Ρ Π½Π°Π·Π²Π°Π½ΠΈΠ΅ элСктричСского сопротивлСния (R). ΠŸΠΎΡΡ‚ΠΎΠΌΡƒ, Ρ‡Π΅ΠΌ Π½ΠΈΠΆΠ΅ Π±ΡƒΠ΄Π΅Ρ‚ сопротивлСниС ΠΏΡ€ΠΎΠ²ΠΎΠ΄Π½ΠΈΠΊΠ°, Ρ‚Π΅ΠΌ Π²Ρ‹ΡˆΠ΅ сила Ρ‚ΠΎΠΊΠ°, проходящСго Ρ‡Π΅Ρ€Π΅Π· Π½Π΅Π³ΠΎ. ΠŸΡ€ΠΈ Ρ‚ΠΎΠΌ, Ρ‡Ρ‚ΠΎ напряТСниС для всСх ΠΏΡ€ΠΎΠ²ΠΎΠ΄Π½ΠΈΠΊΠΎΠ² Π±ΡƒΠ΄Π΅Ρ‚ ΠΎΠ΄ΠΈΠ½Π°ΠΊΠΎΠ²Ρ‹ΠΌ.

ПослС всСх ΠΎΠΏΡ‹Ρ‚ΠΎΠ² ΡƒΡ‡Π΅Π½Ρ‹ΠΉ смог ΠΎΠΊΠΎΠ½Ρ‡Π°Ρ‚Π΅Π»ΡŒΠ½ΠΎ ΡΡ„ΠΎΡ€ΠΌΡƒΠ»ΠΈΡ€ΠΎΠ²Π°Ρ‚ΡŒ свой Π·Π°ΠΊΠΎΠ½ для участка Ρ†Π΅ΠΏΠΈ:

Π‘ΠΈΠ»Π° Ρ‚ΠΎΠΊΠ° Π² ΠΎΠ΄Π½ΠΎΡ€ΠΎΠ΄Π½ΠΎΠΌ ΠΏΡ€ΠΎΠ²ΠΎΠ΄Π½ΠΈΠΊΠ΅ Π½Π° ΠΎΡ‚Π΄Π΅Π»ΡŒΠ½ΠΎΠΌ участкС, находится Π² прямой ΠΏΡ€ΠΎΠΏΠΎΡ€Ρ†ΠΈΠΈ с напряТСниСм Π½Π° этом ΠΆΠ΅ участкС ΠΈ Π² ΠΎΠ±Ρ€Π°Ρ‚Π½ΠΎΠΉ ΠΏΡ€ΠΎΠΏΠΎΡ€Ρ†ΠΈΠΎΠ½Π°Π»ΡŒΠ½ΠΎΠΉ зависимости с сопротивлСниСм Π΄Π°Π½Π½ΠΎΠ³ΠΎ ΠΏΡ€ΠΎΠ²ΠΎΠ΄Π½ΠΈΠΊΠ°.

ΠŸΡ€ΠΈΠ½ΡΡ‚Ρ‹Π΅ Π΅Π΄ΠΈΠ½ΠΈΡ†Ρ‹ измСрСния

ΠŸΡ€ΠΈ использовании Π·Π°ΠΊΠΎΠ½Π° Ома для практичСских расчСтов всС матСматичСскиС вычислСния Π²Ρ‹ΠΏΠΎΠ»Π½ΡΡŽΡ‚ΡΡ Π² установлСнных Π΅Π΄ΠΈΠ½ΠΈΡ†Π°Ρ… ΠΈΠ·ΠΌΠ΅Ρ€Π΅Π½ΠΈΠΉ для всСх 3-Ρ… Π²Π΅Π»ΠΈΡ‡ΠΈΠ½:

  • Π‘ΠΈΠ»Π° Ρ‚ΠΎΠΊΠ° – Π² Π°ΠΌΠΏΠ΅Ρ€Π°Ρ… (А).
  • НапряТСниС – Π² Π²ΠΎΠ»ΡŒΡ‚Π°Ρ… (Π’/V).
  • Π‘ΠΎΠΏΡ€ΠΎΡ‚ΠΈΠ²Π»Π΅Π½ΠΈΠ΅ – Π² ΠΎΠΌΠ°Ρ… (Ом).

Π˜ΡΡ…ΠΎΠ΄Π½Ρ‹Π΅ Π΄Π°Π½Π½Ρ‹Π΅ ΠΈ Π΄Ρ€ΡƒΠ³ΠΈΠ΅ ΠΏΠ°Ρ€Π°ΠΌΠ΅Ρ‚Ρ€Ρ‹, прСдставлСнныС Π² Π΅Π΄ΠΈΠ½ΠΈΡ†Π°Ρ…, Π΄ΠΎΠ»ΠΆΠ½Ρ‹ ΠΏΠ΅Ρ€Π΅Π²ΠΎΠ΄ΠΈΡ‚ΡŒΡΡ Π² общСпринятыС значСния.

ДСйствиС основных Π΅Π΄ΠΈΠ½ΠΈΡ† ΠΈ физичСскоС соблюдСниС Π·Π°ΠΊΠΎΠ½Π° Ома Π½Π΅Π²ΠΎΠ·ΠΌΠΎΠΆΠ½ΠΎ Π² ΡΠ»Π΅Π΄ΡƒΡŽΡ‰ΠΈΡ… ситуациях:

  • НаличиС высоких частот, ΠΏΡ€ΠΈ ΠΊΠΎΡ‚ΠΎΡ€Ρ‹Ρ… элСктричСскоС ΠΏΠΎΠ»Π΅ измСняСтся с большой ΡΠΊΠΎΡ€ΠΎΡΡ‚ΡŒΡŽ.
  • НизкотСмпСратурный Ρ€Π΅ΠΆΠΈΠΌ ΠΈ ΡΠ²Π΅Ρ€Ρ…ΠΏΡ€ΠΎΠ²ΠΎΠ΄ΠΈΠΌΠΎΡΡ‚ΡŒ.
  • Бильно Ρ€Π°Π·ΠΎΠ³Ρ€Π΅Ρ‚Ρ‹Π΅ спирали Π»Π°ΠΌΠΏ накаливания, ΠΊΠΎΠ³Π΄Π° отсутствуСт Π»ΠΈΠ½Π΅ΠΉΠ½ΠΎΡΡ‚ΡŒ напряТСния.
  • ΠŸΡ€ΠΎΠ±ΠΎΠΉ ΠΏΡ€ΠΎΠ²ΠΎΠ΄Π½ΠΈΠΊΠ° ΠΈΠ»ΠΈ диэлСктрика, Π²Ρ‹Π·Π²Π°Π½Π½Ρ‹ΠΉ высоким напряТСниСм.
  • Π­Π»Π΅ΠΊΡ‚Ρ€ΠΎΠ½Π½Ρ‹Π΅ ΠΈ Π²Π°ΠΊΡƒΡƒΠΌΠ½Ρ‹Π΅ Π»Π°ΠΌΠΏΡ‹, Π·Π°ΠΏΠΎΠ»Π½Π΅Π½Π½Ρ‹Π΅ Π³Π°Π·Π°ΠΌΠΈ.
  • ΠŸΠΎΠ»ΡƒΠΏΡ€ΠΎΠ²ΠΎΠ΄Π½ΠΈΠΊΠΈ с Ρ€-ΠΏ-ΠΏΠ΅Ρ€Π΅Ρ…ΠΎΠ΄Π°ΠΌΠΈ, Π² Ρ‚ΠΎΠΌ числС, Π΄ΠΈΠΎΠ΄Ρ‹ ΠΈ транзисторы.

Π‘ΠΈΠ»Π° Ρ‚ΠΎΠΊΠ°

Π‘ΠΈΠ»Π° Ρ‚ΠΎΠΊΠ° Π²ΠΎΠ·Π½ΠΈΠΊΠ°Π΅Ρ‚ ΠΏΡ€ΠΈ Π½Π°Π»ΠΈΡ‡ΠΈΠΈ частиц со свободными зарядами. Они ΠΏΠ΅Ρ€Π΅ΠΌΠ΅Ρ‰Π°ΡŽΡ‚ΡΡ Ρ‡Π΅Ρ€Π΅Π· ΠΏΠΎΠΏΠ΅Ρ€Π΅Ρ‡Π½ΠΎΠ΅ сСчСниС ΠΏΡ€ΠΎΠ²ΠΎΠ΄Π½ΠΈΠΊΠ° ΠΈΠ· ΠΎΠ΄Π½ΠΎΠΉ Ρ‚ΠΎΡ‡ΠΊΠΈ Π² Π΄Ρ€ΡƒΠ³ΡƒΡŽ. Π˜ΡΡ‚ΠΎΡ‡Π½ΠΈΠΊ питания создаСт элСктричСскоС ΠΏΠΎΠ»Π΅, ΠΏΠΎΠ΄ дСйствиСм ΠΊΠΎΡ‚ΠΎΡ€ΠΎΠ³ΠΎ элСктроны Π½Π°Ρ‡ΠΈΠ½Π°ΡŽΡ‚ Π΄Π²ΠΈΠ³Π°Ρ‚ΡŒΡΡ упорядочСнно.

Π’Π°ΠΊΠΈΠΌ ΠΎΠ±Ρ€Π°Π·ΠΎΠΌ, сила Ρ‚ΠΎΠΊΠ° являСтся количСством элСктричСства, проходящСго Ρ‡Π΅Ρ€Π΅Π· ΠΎΠΏΡ€Π΅Π΄Π΅Π»Π΅Π½Π½ΠΎΠ΅ сСчСниС Π·Π° Π΅Π΄ΠΈΠ½ΠΈΡ†Ρƒ Π²Ρ€Π΅ΠΌΠ΅Π½ΠΈ. Π£Π²Π΅Π»ΠΈΡ‡ΠΈΡ‚ΡŒ этот ΠΏΠΎΠΊΠ°Π·Π°Ρ‚Π΅Π»ΡŒ ΠΌΠΎΠΆΠ½ΠΎ ΠΏΡƒΡ‚Π΅ΠΌ увСличСния мощности источника Ρ‚ΠΎΠΊΠ° ΠΈΠ»ΠΈ ΠΈΠ·ΡŠΡΡ‚ΠΈΡ ΠΈΠ· Ρ†Π΅ΠΏΠΈ рСзистивных элСмСнтов.

ΠœΠ΅ΠΆΠ΄ΡƒΠ½Π°Ρ€ΠΎΠ΄Π½Π°Ρ Π΅Π΄ΠΈΠ½ΠΈΡ†Π° БИ для Ρ‚ΠΎΠΊΠ° – Π°ΠΌΠΏΠ΅Ρ€. Π­Ρ‚ΠΎ довольно большая Π²Π΅Π»ΠΈΡ‡ΠΈΠ½Π°, ΠΏΠΎΡΠΊΠΎΠ»ΡŒΠΊΡƒ для Ρ‡Π΅Π»ΠΎΠ²Π΅ΠΊΠ° ΡΠΌΠ΅Ρ€Ρ‚Π΅Π»ΡŒΠ½ΠΎ опасными ΡΡ‡ΠΈΡ‚Π°ΡŽΡ‚ΡΡ всСго 0,1 А. Π’ элСктротСхникС ΠΌΠ°Π»Ρ‹Π΅ Π²Π΅Π»ΠΈΡ‡ΠΈΠ½Ρ‹ ΠΌΠΎΠ³ΡƒΡ‚ Π²Ρ‹Ρ€Π°ΠΆΠ°Ρ‚ΡŒΡΡ Π² ΠΌΠΈΠΊΡ€ΠΎ- ΠΈ ΠΌΠΈΠ»Π»ΠΈΠ°ΠΌΠΏΠ΅Ρ€Π°Ρ….

ΠžΠΏΡ€Π΅Π΄Π΅Π»Π΅Π½ΠΈΠ΅ силы Ρ‚ΠΎΠΊΠ° ΠΌΠΎΠΆΠ½ΠΎ ΠΎΠΊΠΎΠ½Ρ‡Π°Ρ‚Π΅Π»ΡŒΠ½ΠΎ ΡΡ„ΠΎΡ€ΠΌΠΈΡ€ΠΎΠ²Π°Ρ‚ΡŒ Π² Π²ΠΈΠ΄Π΅ Ρ„ΠΎΡ€ΠΌΡƒΠ»Ρ‹ I = q/t, Π² ΠΊΠΎΡ‚ΠΎΡ€ΠΎΠΉ q являСтся зарядом, проходящим Ρ‡Π΅Ρ€Π΅Π· сСчСниС, t – ΠΎΡ‚Ρ€Π΅Π·ΠΎΠΊ Π²Ρ€Π΅ΠΌΠ΅Π½ΠΈ, Π·Π°Ρ‚Ρ€Π°Ρ‡Π΅Π½Π½Ρ‹ΠΉ Π½Π° ΠΏΠ΅Ρ€Π΅ΠΌΠ΅Ρ‰Π΅Π½ΠΈΠ΅ этого заряда.

ΠšΡ€ΠΎΠΌΠ΅ Ρ‚ΠΎΠ³ΠΎ, сила Ρ‚ΠΎΠΊΠ° ΠΌΠΎΠΆΠ΅Ρ‚ Π·Π°ΠΏΠΈΡΡ‹Π²Π°Ρ‚ΡŒΡΡ с ΠΏΠΎΠΌΠΎΡ‰ΡŒΡŽ основной Ρ„ΠΎΡ€ΠΌΡƒΠ»Ρ‹, ΠΊΠΎΠ³Π΄Π° извСстны значСния напряТСния ΠΈ сопротивлСния. Π’ числом Π²ΠΈΠ΄Π΅ ΠΎΠ½Π° Π±ΡƒΠ΄Π΅Ρ‚ Π³Π»Π°ΡΠΈΡ‚ΡŒ ΡΠ»Π΅Π΄ΡƒΡŽΡ‰Π΅Π΅:

Π‘ΠΎΠΏΡ€ΠΎΡ‚ΠΈΠ²Π»Π΅Π½ΠΈΠ΅

Рассматривая Π·Π°ΠΊΠΎΠ½ ΠΎΠΌΠ° для участка Ρ†Π΅ΠΏΠΈ, нСльзя Π·Π°Π±Ρ‹Π²Π°Ρ‚ΡŒ ΠΎ Ρ‚Π°ΠΊΠΎΠΌ понятии, ΠΊΠ°ΠΊ сопротивлСниС. Данная Π²Π΅Π»ΠΈΡ‡ΠΈΠ½Π° считаСтся основной характСристикой ΠΏΡ€ΠΎΠ²ΠΎΠ΄Π½ΠΈΠΊΠ°, ΠΏΠΎΡΠΊΠΎΠ»ΡŒΠΊΡƒ ΠΈΠΌΠ΅Π½Π½ΠΎ сопротивлСниС влияСт Π½Π° качСство проводимости. Π Π°Π·Π½Ρ‹Π΅ ΠΌΠ°Ρ‚Π΅Ρ€ΠΈΠ°Π»Ρ‹ проводят Ρ‚ΠΎΠΊ Π»ΡƒΡ‡ΡˆΠ΅ ΠΈΠ»ΠΈ Ρ…ΡƒΠΆΠ΅. Π­Ρ‚ΠΎ ΠΎΠ±ΡŠΡΡΠ½ΡΠ΅Ρ‚ΡΡ Π½Π΅ΠΎΠ΄Π½ΠΎΡ€ΠΎΠ΄Π½ΠΎΡΡ‚ΡŒΡŽ ΠΈΡ… структуры, различиями Π² кристалличСских Ρ€Π΅ΡˆΠ΅Ρ‚ΠΊΠ°Ρ…. ΠŸΠΎΡΡ‚ΠΎΠΌΡƒ Π² ΠΎΠ΄Π½ΠΈΡ… случаях элСктроны двиТутся с большСй ΡΠΊΠΎΡ€ΠΎΡΡ‚ΡŒΡŽ, Π° Π² Π΄Ρ€ΡƒΠ³ΠΈΡ… – с мСньшСй.

БобствСнным элСктричСским сопротивлСниСм ΠΎΠ±Π»Π°Π΄Π°ΡŽΡ‚ всС ΠΏΡ€ΠΎΠ²ΠΎΠ΄Π½ΠΈΠΊΠΈ, находящиСся Π² Ρ‚Π²Π΅Ρ€Π΄ΠΎΠΌ, ΠΆΠΈΠ΄ΠΊΠΎΠΌ, Π³Π°Π·ΠΎΠΎΠ±Ρ€Π°Π·Π½ΠΎΠΌ ΠΈ ΠΏΠ»Π°Π·ΠΌΠ΅Π½Π½ΠΎΠΌ состоянии. Π£ ΠΊΠ°ΠΆΠ΄ΠΎΠ³ΠΎ ΠΈΠ· Π½ΠΈΡ… своя характСристика, называСмая ΡƒΠ΄Π΅Π»ΡŒΠ½Ρ‹ΠΌ сопротивлСниСм. Данная Π²Π΅Π»ΠΈΡ‡ΠΈΠ½Π° ΠΎΡ‚Ρ€Π°ΠΆΠ°Π΅Ρ‚ ΡΠΏΠΎΡΠΎΠ±Π½ΠΎΡΡ‚ΡŒ ΠΊΠ°ΠΆΠ΄ΠΎΠ³ΠΎ ΠΌΠ°Ρ‚Π΅Ρ€ΠΈΠ°Π»Π° ΠΊ ΡΠΎΠΏΡ€ΠΎΡ‚ΠΈΠ²Π»Π΅Π½ΠΈΡŽ. Π—Π° эталон принимаСтся ΠΏΡ€ΠΎΠ²ΠΎΠ΄Π½ΠΈΠΊ Π΄Π»ΠΈΠ½ΠΎΠΉ 1 ΠΌ с ΠΏΠΎΠΏΠ΅Ρ€Π΅Ρ‡Π½Ρ‹ΠΌ сСчСниСм 1 ΠΌΒ².

Π§Ρ‚ΠΎΠ±Ρ‹ Π½Π°ΠΉΡ‚ΠΈ сопротивлСниС ΠΏΡ€ΠΎΠ²ΠΎΠ΄Π½ΠΈΠΊΠ° ΠΈΠ· Π΄Π°Π½Π½ΠΎΠ³ΠΎ ΠΌΠ°Ρ‚Π΅Ρ€ΠΈΠ°Π»Π° Π½ΡƒΠΆΠ½ΠΎ Π²ΠΎΡΠΏΠΎΠ»ΡŒΠ·ΠΎΠ²Π°Ρ‚ΡŒΡΡ Ρ„ΠΎΡ€ΠΌΡƒΠ»ΠΎΠΉ: R = ρ x (l/S). Π’ Π½Π΅ΠΉ l являСтся Π΄Π»ΠΈΠ½ΠΎΠΉ ΠΏΡ€ΠΎΠ²ΠΎΠ΄Π½ΠΈΠΊΠ°, S – ΠΏΠ»ΠΎΡ‰Π°Π΄ΡŒΡŽ Π΅Π³ΠΎ ΠΏΠΎΠΏΠ΅Ρ€Π΅Ρ‡Π½ΠΎΠ³ΠΎ сСчСния, ρ – ΡƒΠ΄Π΅Π»ΡŒΠ½Ρ‹ΠΌ сопротивлСниСм.

По Π·Π°ΠΊΠΎΠ½Ρƒ Ома Π½Π° участкС Ρ†Π΅ΠΏΠΈ эта Π²Π΅Π»ΠΈΡ‡ΠΈΠ½Π° опрСдСляСтся: R = U/I.

НапряТСниС

НапряТСниС относится ΠΊ Π²Π°ΠΆΠ½Ρ‹ΠΌ характСристикам элСктричСского Ρ‚ΠΎΠΊΠ°, ΠΏΡ€ΠΎΡ‚Π΅ΠΊΠ°ΡŽΡ‰Π΅Π³ΠΎ Π² ΠΏΡ€ΠΎΠ²ΠΎΠ΄Π½ΠΈΠΊΠ΅. Π‘ физичСской Ρ‚ΠΎΡ‡ΠΊΠΈ зрСния, это Ρ€Π°Π±ΠΎΡ‚Π° элСктричСского поля, ΠΊΠΎΡ‚ΠΎΡ€ΠΎΠ΅ ΠΏΠ΅Ρ€Π΅ΠΌΠ΅Ρ‰Π°Π΅Ρ‚ заряд Π½Π° ΠΊΠ°ΠΊΠΎΠ΅-Ρ‚ΠΎ расстояниС. Π’ элСктротСхникС напряТСниСм считаСтся Ρ€Π°Π·Π½ΠΎΡΡ‚ΡŒ ΠΏΠΎΡ‚Π΅Π½Ρ†ΠΈΠ°Π»ΠΎΠ² ΠΌΠ΅ΠΆΠ΄Ρƒ двумя Ρ‚ΠΎΡ‡ΠΊΠ°ΠΌΠΈ участка Ρ†Π΅ΠΏΠΈ. На ΠΏΡ€Π°ΠΊΡ‚ΠΈΠΊΠ΅ эта Π²Π΅Π»ΠΈΡ‡ΠΈΠ½Π° слуТит для опрСдСлСния возмоТности ΠΏΠΎΠ΄ΠΊΠ»ΡŽΡ‡Π΅Π½ΠΈΡ ΠΊ сСти ΠΏΠΎΡ‚Ρ€Π΅Π±ΠΈΡ‚Π΅Π»Π΅ΠΉ элСктроэнСргии, ΠΏΡ€ΠΎΠ΄ΠΎΠ»ΠΆΠΈΡ‚Π΅Π»ΡŒΠ½ΠΎΡΡ‚ΡŒ ΠΈΡ… Ρ€Π°Π±ΠΎΡ‚Ρ‹ Π² этом состоянии.

Π’ элСктричСской Ρ†Π΅ΠΏΠΈ напряТСниС Π²ΠΎΠ·Π½ΠΈΠΊΠ°Π΅Ρ‚ ΡΠ»Π΅Π΄ΡƒΡŽΡ‰ΠΈΠΌ ΠΎΠ±Ρ€Π°Π·ΠΎΠΌ:

  • Π’Π½Π°Ρ‡Π°Π»Π΅ Ρ†Π΅ΠΏΡŒ ΠΏΠΎΠ΄ΠΊΠ»ΡŽΡ‡Π°Π΅Ρ‚ΡΡ ΠΊ источнику Ρ‚ΠΎΠΊΠ° ΠΏΡƒΡ‚Π΅ΠΌ соСдинСния с двумя полюсами. Π­Ρ‚ΠΎ ΠΌΠΎΠΆΠ΅Ρ‚ Π±Ρ‹Ρ‚ΡŒ Π³Π΅Π½Π΅Ρ€Π°Ρ‚ΠΎΡ€ ΠΈΠ»ΠΈ батарСя.
  • На ΠΎΠ΄Π½ΠΎΠΌ полюсС ΠΈΠ»ΠΈ ΠΊΠ»Π΅ΠΌΠΌΠ΅ – ΠΈΠ·Π±Ρ‹Ρ‚ΠΎΡ‡Π½ΠΎΠ΅ количСство элСктроном, Π° Π½Π° Π΄Ρ€ΡƒΠ³ΠΎΠΌ – ΠΈΡ… нСдостаСт. ΠŸΠ΅Ρ€Π²Ρ‹ΠΉ условно считаСтся ΠΏΠΎΠ»ΠΎΠΆΠΈΡ‚Π΅Π»ΡŒΠ½Ρ‹ΠΌ, Π²Ρ‚ΠΎΡ€ΠΎΠΉ – ΠΎΡ‚Ρ€ΠΈΡ†Π°Ρ‚Π΅Π»ΡŒΠ½Ρ‹ΠΌ.
  • ЭлСктричСскоС ΠΏΠΎΠ»Π΅ источника энСргии Π²ΠΎΠ·Π΄Π΅ΠΉΡΡ‚Π²ΡƒΡŽΡ‚ Π½Π° элСктроны ΠΏΠΎΠ»ΠΎΠΆΠΈΡ‚Π΅Π»ΡŒΠ½ΠΎΠ³ΠΎ полюса ΠΈ самого ΠΏΡ€ΠΎΠ²ΠΎΠ΄Π½ΠΈΠΊΠ°, заставляя ΠΈΡ… Π΄Π²ΠΈΠ³Π°Ρ‚ΡŒΡΡ Π² сторону ΠΎΡ‚Ρ€ΠΈΡ†Π°Ρ‚Π΅Π»ΡŒΠ½ΠΎΠ³ΠΎ полюса ΠΈ ΠΏΡ€ΠΈΡ‚ΡΠ³ΠΈΠ²Π°Ρ‚ΡŒΡΡ ΠΊ Π½Π΅ΠΌΡƒ. Π’Π°ΠΊΠΎΠ΅ притяТСниС происходит ΠΈΠ·-Π·Π° ΠΏΠΎΠ»ΠΎΠΆΠΈΡ‚Π΅Π»ΡŒΠ½ΠΎΠ³ΠΎ заряда Π½Π° этом полюсС, ΠΏΠΎΡΠΊΠΎΠ»ΡŒΠΊΡƒ элСктроны здСсь ΠΎΡ‚ΡΡƒΡ‚ΡΡ‚Π²ΡƒΡŽΡ‚.
  • ΠœΠ΅ΠΆΠ΄Ρƒ ΠΎΠ±Π΅ΠΈΠΌΠΈ ΠΊΠ»Π΅ΠΌΠΌΠ°ΠΌΠΈ Π²ΠΎΠ·Π½ΠΈΠΊΠ°Π΅Ρ‚ Ρ€Π°Π·Π½ΠΎΡΡ‚ΡŒ ΠΏΠΎΡ‚Π΅Π½Ρ†ΠΈΠ°Π»ΠΎΠ² с ΠΎΠΏΡ€Π΅Π΄Π΅Π»Π΅Π½Π½Ρ‹ΠΌ Π·Π½Π°Ρ‡Π΅Π½ΠΈΠ΅ΠΌ, Ρ‡Ρ‚ΠΎ ΠΏΡ€ΠΈΠ²ΠΎΠ΄ΠΈΡ‚ ΠΊ упорядочСнному двиТСнию элСктронов Π² ΠΏΡ€ΠΎΠ²ΠΎΠ΄Π½ΠΈΠΊΠ°Ρ… ΠΈ ΠΏΠΎΠ΄ΠΊΠ»ΡŽΡ‡Π΅Π½Π½Ρ‹Ρ… Π½Π°Π³Ρ€ΡƒΠ·ΠΊΠ°Ρ…. ΠŸΠΎΡΡ‚Π΅ΠΏΠ΅Π½Π½ΠΎ ΠΈΠ·Π±Ρ‹Ρ‚ΠΎΠΊ элСктронов ΠΏΠΎΠ»ΠΎΠΆΠΈΡ‚Π΅Π»ΡŒΠ½ΠΎΠ³ΠΎ полюса ΡƒΠΌΠ΅Π½ΡŒΡˆΠ°Π΅Ρ‚ΡΡ, соотвСтствСнно, сниТаСтся ΠΈ ΠΏΠΎΡ‚Π΅Π½Ρ†ΠΈΠ°Π». Π₯Π°Ρ€Π°ΠΊΡ‚Π΅Ρ€Π½Ρ‹ΠΌ ΠΏΡ€ΠΈΠΌΠ΅Ρ€ΠΎΠΌ слуТит аккумуляторная батарСя. ΠŸΡ€ΠΈ ΠΏΠΎΠ΄ΠΊΠ»ΡŽΡ‡Π΅Π½ΠΈΠΈ Π½Π°Π³Ρ€ΡƒΠ·ΠΊΠΈ, Π΅Π΅ ΠΏΠΎΡ‚Π΅Π½Ρ†ΠΈΠ°Π» Π±ΡƒΠ΄Π΅Ρ‚ ΠΏΠ°Π΄Π°Ρ‚ΡŒ, Π²ΠΏΠ»ΠΎΡ‚ΡŒ Π΄ΠΎ ΠΏΠΎΠ»Π½ΠΎΠΉ разрядки. Для восстановлСния ΠΏΠ΅Ρ€Π²ΠΎΠ½Π°Ρ‡Π°Π»ΡŒΠ½Ρ‹Ρ… свойств, потрСбуСтся подзарядка ΠΎΡ‚ постороннСго источника Ρ‚ΠΎΠΊΠ°.

ΠŸΡ€ΠΈ Π½Π΅ΠΈΠ·ΠΌΠ΅Π½Π½ΠΎΠΉ мощности источника энСргии, Π·Π½Π°Ρ‡Π΅Π½ΠΈΠ΅ напряТСния ΠΌΠΎΠΆΠ΅Ρ‚ Π±Ρ‹Ρ‚ΡŒ Ρ€Π°Π·Π½Ρ‹ΠΌ ΠΏΠΎΠ΄ дСйствиСм ΡΠ»Π΅Π΄ΡƒΡŽΡ‰ΠΈΡ… Ρ„Π°ΠΊΡ‚ΠΎΡ€ΠΎΠ²:

  1. ΠœΠ°Ρ‚Π΅Ρ€ΠΈΠ°Π» ΡΠΎΠ΅Π΄ΠΈΠ½ΠΈΡ‚Π΅Π»ΡŒΠ½Ρ‹Ρ… ΠΏΡ€ΠΎΠ²ΠΎΠ΄Π½ΠΈΠΊΠΎΠ². Π£ ΠΊΠ°ΠΆΠ΄ΠΎΠ³ΠΎ свой Π²ΠΎΠ»ΡŒΡ‚Π°ΠΌΠΏΠ΅Ρ€Π½Ρ‹ΠΉ Π³Ρ€Π°Ρ„ΠΈΠΊ.
  2. ΠšΠΎΠ»ΠΈΡ‡Π΅ΡΡ‚Π²ΠΎ ΠΏΠΎΡ‚Ρ€Π΅Π±ΠΈΡ‚Π΅Π»Π΅ΠΉ, ΠΏΠΎΠ΄ΠΊΠ»ΡŽΡ‡Π΅Π½Π½Ρ‹Ρ… ΠΊ сСти.
  3. Π’Π΅ΠΌΠΏΠ΅Ρ€Π°Ρ‚ΡƒΡ€Π° ΠΎΠΊΡ€ΡƒΠΆΠ°ΡŽΡ‰Π΅ΠΉ срСды.
  4. ΠšΠ°Ρ‡Π΅ΡΡ‚Π²ΠΎ ΠΌΠΎΠ½Ρ‚Π°ΠΆΠ° самой сСти.

Π—Π°ΠΊΠΎΠ½ Ома для участка Ρ†Π΅ΠΏΠΈ – расчСт Ρ†Π΅ΠΏΠ΅ΠΉ

ΠŸΡ€ΠΎΡΡ‚Π΅ΠΉΡˆΠΈΠΉ Π²Π°Ρ€ΠΈΠ°Π½Ρ‚ наглядно прСдставлСн Π½Π° рисункС. Π­Ρ‚ΠΎ ΠΎΠ΄Π½ΠΎΡ€ΠΎΠ΄Π½Ρ‹ΠΉ участок Ρ†Π΅ΠΏΠΈ ΠΎΡ‚ΠΊΡ€Ρ‹Ρ‚ΠΎΠ³ΠΎ Ρ‚ΠΈΠΏΠ°.

Для Π΅Π³ΠΎ описания примСняСтся извСстная Ρ„ΠΎΡ€ΠΌΡƒΠ»Π°, которая Π±ΡƒΠ΄Π΅Ρ‚ ΠΈΠΌΠ΅Ρ‚ΡŒ ΡΠ»Π΅Π΄ΡƒΡŽΡ‰ΡƒΡŽ Ρ„ΠΎΡ€ΠΌΡƒ:

  • I = U/R, Π³Π΄Π΅ I являСтся силой Ρ‚ΠΎΠΊΠ°, U – напряТСниСм, R – сопротивлСниСм.

Данная Ρ„ΠΎΡ€ΠΌΡƒΠ»Π° являСтся ΠΈΠ½Ρ‚Π΅Π³Ρ€Π°Π»ΡŒΠ½ΠΎΠΉ. Π‘ Π΅Π΅ ΠΏΠΎΠΌΠΎΡ‰ΡŒΡŽ Ρ…ΠΎΡ€ΠΎΡˆΠΎ Π²ΠΈΠ΄Π½ΠΎ, ΠΊΠ°ΠΊ ΠΏΡ€ΠΈ возрастании напряТСния, увСличиваСтся ΠΈ сила Ρ‚ΠΎΠΊΠ°. Но, Ссли ΡƒΠ²Π΅Π»ΠΈΡ‡ΠΈΡ‚ΡŒ сопротивлСниС, Ρ‚ΠΎ сила Ρ‚ΠΎΠΊΠ°, Π½Π°ΠΎΠ±ΠΎΡ€ΠΎΡ‚, Π±ΡƒΠ΄Π΅Ρ‚ ΠΏΠΎΠ½ΠΈΠΆΠ°Ρ‚ΡŒΡΡ.

На схСмС ΠΈΠ·ΠΎΠ±Ρ€Π°ΠΆΠ΅Π½ всСго ΠΎΠ΄ΠΈΠ½ элСмСнт, ΠΎΠ±Π»Π°Π΄Π°ΡŽΡ‰ΠΈΠΉ сопротивлСниСм. На ΠΏΡ€Π°ΠΊΡ‚ΠΈΠΊΠ΅, ΠΈΡ… ΠΌΠΎΠΆΠ΅Ρ‚ Π±Ρ‹Ρ‚ΡŒ любоС количСство. Они ΠΌΠΎΠ³ΡƒΡ‚ ΡΠΎΠ΅Π΄ΠΈΠ½ΡΡ‚ΡŒΡΡ ΠΏΠΎΡΠ»Π΅Π΄ΠΎΠ²Π°Ρ‚Π΅Π»ΡŒΠ½ΠΎ, ΠΏΠ°Ρ€Π°Π»Π»Π΅Π»ΡŒΠ½ΠΎ ΠΈ ΡΠΌΠ΅ΡˆΠ°Π½Π½Ρ‹ΠΌ способом.

НСоднородный участок Ρ†Π΅ΠΏΠΈ постоянного Ρ‚ΠΎΠΊΠ°

ΠΠ΅ΠΎΠ΄Π½ΠΎΡ€ΠΎΠ΄Π½ΡƒΡŽ структуру ΠΈΠΌΠ΅Π΅Ρ‚ Ρ‚Π°ΠΊΠΎΠΉ участок Ρ†Π΅ΠΏΠΈ, Π³Π΄Π΅ ΠΏΠΎΠΌΠΈΠΌΠΎ ΠΏΡ€ΠΎΠ²ΠΎΠ΄Π½ΠΈΠΊΠΎΠ² ΠΈ элСмСнтов, присутствуСт источник Ρ‚ΠΎΠΊΠ°. Π•Π³ΠΎ Π­Π”Π‘ Π½Π΅ΠΎΠ±Ρ…ΠΎΠ΄ΠΈΠΌΠΎ ΡƒΡ‡ΠΈΡ‚Ρ‹Π²Π°Ρ‚ΡŒ ΠΏΡ€ΠΈ расчСтах ΠΎΠ±Ρ‰Π΅ΠΉ силы Ρ‚ΠΎΠΊΠ° Π½Π° Π΄Π°Π½Π½ΠΎΠΌ участкС.

БущСствуСт Ρ„ΠΎΡ€ΠΌΡƒΠ»Π°, которая Π΄Π°Π΅Ρ‚ ΠΎΠΏΡ€Π΅Π΄Π΅Π»Π΅Π½ΠΈΠ΅ основным ΠΏΠ°Ρ€Π°ΠΌΠ΅Ρ‚Ρ€Π°ΠΌ ΠΈ процСссам Π½Π΅ΠΎΠ΄Π½ΠΎΡ€ΠΎΠ΄Π½ΠΎΠ³ΠΎ участка: q = q0 x n x V. Π•Π΅ ΠΏΠΎΠΊΠ°Π·Π°Ρ‚Π΅Π»ΠΈ Ρ…Π°Ρ€Π°ΠΊΡ‚Π΅Ρ€ΠΈΠ·ΡƒΡŽΡ‚ΡΡ ΡΠ»Π΅Π΄ΡƒΡŽΡ‰ΠΈΠΌ ΠΎΠ±Ρ€Π°Π·ΠΎΠΌ:

  • Π’ процСссС пСрСмСщСния зарядов (q) ΠΎΠ½ΠΈ ΠΏΡ€ΠΈΠΎΠ±Ρ€Π΅Ρ‚Π°ΡŽΡ‚ ΠΎΠΏΡ€Π΅Π΄Π΅Π»Π΅Π½Π½ΡƒΡŽ ΠΏΠ»ΠΎΡ‚Π½ΠΎΡΡ‚ΡŒ. Π•Π΅ ΠΏΠΎΠΊΠ°Π·Π°Ρ‚Π΅Π»ΠΈ зависят ΠΎΡ‚ силы Ρ‚ΠΎΠΊΠ° ΠΈ ΠΏΠ»ΠΎΡ‰Π°Π΄ΠΈ ΠΏΠΎΠΏΠ΅Ρ€Π΅Ρ‡Π½ΠΎΠ³ΠΎ сСчСния ΠΏΡ€ΠΎΠ²ΠΎΠ΄Π½ΠΈΠΊΠ° (S).
  • Π’ условиях ΠΎΠΏΡ€Π΅Π΄Π΅Π»Π΅Π½Π½ΠΎΠΉ ΠΊΠΎΠ½Ρ†Π΅Π½Ρ‚Ρ€Π°Ρ†ΠΈΠΈ (n) ΠΌΠΎΠΆΠ½ΠΎ Ρ‚ΠΎΡ‡Π½ΠΎ ΡƒΠΊΠ°Π·Π°Ρ‚ΡŒ Ρ‡ΠΈΡΠ»Π΅Π½Π½ΠΎΡΡ‚ΡŒ Π΅Π΄ΠΈΠ½ΠΈΡ‡Π½Ρ‹Ρ… зарядов (q0), ΠΊΠΎΡ‚ΠΎΡ€Ρ‹Π΅ Π±Ρ‹Π»ΠΈ ΠΏΠ΅Ρ€Π΅ΠΌΠ΅Ρ‰Π΅Π½Ρ‹ Π·Π° Π΅Π΄ΠΈΠ½ΠΈΡ‡Π½Ρ‹ΠΉ ΠΎΡ‚Ρ€Π΅Π·ΠΎΠΊ Π²Ρ€Π΅ΠΌΠ΅Π½ΠΈ.
  • Для расчСтов ΠΏΡ€ΠΎΠ²ΠΎΠ΄Π½ΠΈΠΊ условно считаСтся цилиндричСским участком, ΠΈΠΌΠ΅ΡŽΡ‰ΠΈΠΌ ΠΊΠ°ΠΊΠΎΠΉ-Ρ‚ΠΎ объСм (V).

ΠŸΡ€ΠΈ ΠΏΠΎΠ΄ΠΊΠ»ΡŽΡ‡Π΅Π½ΠΈΠΈ ΠΏΡ€ΠΎΠ²ΠΎΠ΄Π½ΠΈΠΊΠ° ΠΊ аккумулятору, послСдний Ρ‡Π΅Ρ€Π΅Π· Π½Π΅ΠΊΠΎΡ‚ΠΎΡ€ΠΎΠ΅ врСмя Π±ΡƒΠ΄Π΅Ρ‚ разряТСн. Π’ΠΎ Π΅ΡΡ‚ΡŒ, Π΄Π²ΠΈΠΆΠ΅Π½ΠΈΠ΅ элСктронов постСпСнно замСдляСтся ΠΈ, Π² ΠΊΠΎΠ½Ρ†Π΅ ΠΊΠΎΠ½Ρ†ΠΎΠ², прСкратится совсСм. Π­Ρ‚ΠΎΠΌΡƒ способствуСт молСкулярная Ρ€Π΅ΡˆΠ΅Ρ‚ΠΊΠ° ΠΏΡ€ΠΎΠ²ΠΎΠ΄Π½ΠΈΠΊΠ°, ΠΎΠΊΠ°Π·Ρ‹Π²Π°ΡŽΡ‰Π°Ρ противодСйствиС, столкновСния элСктронов ΠΌΠ΅ΠΆΠ΄Ρƒ собой ΠΈ Π΄Ρ€ΡƒΠ³ΠΈΠ΅ Ρ„Π°ΠΊΡ‚ΠΎΡ€Ρ‹. Для прСодолСния Ρ‚Π°ΠΊΠΎΠ³ΠΎ сопротивлСния слСдуСт Π΄ΠΎΠΏΠΎΠ»Π½ΠΈΡ‚Π΅Π»ΡŒΠ½ΠΎ ΠΏΡ€ΠΈΠ»ΠΎΠΆΠΈΡ‚ΡŒ ΠΎΠΏΡ€Π΅Π΄Π΅Π»Π΅Π½Π½Ρ‹Π΅ сторонниС силы.

Π’ΠΎ врСмя расчСтов эти силы ΡΡƒΠΌΠΌΠΈΡ€ΡƒΡŽΡ‚ΡΡ с кулоновскими. ΠšΡ€ΠΎΠΌΠ΅ Ρ‚ΠΎΠ³ΠΎ, для пСрСнСсСния Π΅Π΄ΠΈΠ½ΠΈΡ‡Π½ΠΎΠ³ΠΎ заряда q ΠΈΠ· 1-ΠΉ Ρ‚ΠΎΡ‡ΠΊΠΈ Π²ΠΎ 2-ю потрСбуСтся Π²Ρ‹ΠΏΠΎΠ»Π½Π΅Π½ΠΈΠ΅ Ρ€Π°Π±ΠΎΡ‚Ρ‹ А1-2 ΠΈΠ»ΠΈ просто А12. Π‘ этой Ρ†Π΅Π»ΡŒΡŽ создаСтся Ρ€Π°Π·Π½ΠΈΡ†Π° ΠΏΠΎΡ‚Π΅Π½Ρ†ΠΈΠ°Π»ΠΎΠ² (Ο•1 – Ο•2). Под дСйствиСм источника постоянного Ρ‚ΠΎΠΊΠ° Π²ΠΎΠ·Π½ΠΈΠΊΠ°Π΅Ρ‚ Π­Π”Π‘, ΠΏΠ΅Ρ€Π΅ΠΌΠ΅Ρ‰Π°ΡŽΡ‰Π°Ρ заряды ΠΏΠΎ Ρ†Π΅ΠΏΠΈ. Π’Π΅Π»ΠΈΡ‡ΠΈΠ½Π° ΠΎΠ±Ρ‰Π΅Π³ΠΎ напряТСния Π±ΡƒΠ΄Π΅Ρ‚ ΡΠΎΡΡ‚ΠΎΡΡ‚ΡŒ ΠΈΠ· всСх сил, ΠΎΡ‚ΠΌΠ΅Ρ‡Π΅Π½Π½Ρ‹Ρ… Π²Ρ‹ΡˆΠ΅.

ΠŸΠΎΠ»ΡΡ€Π½ΠΎΡΡ‚ΡŒ ΠΏΠΎΠ΄ΠΊΠ»ΡŽΡ‡Π΅Π½ΠΈΡ ΠΊ источнику постоянного Ρ‚ΠΎΠΊΠ° Π½ΡƒΠΆΠ½ΠΎ ΡƒΡ‡ΠΈΡ‚Ρ‹Π²Π°Ρ‚ΡŒ Π² расчСтах. ΠŸΡ€ΠΈ ΠΈΠ·ΠΌΠ΅Π½Π΅Π½ΠΈΠΈ ΠΊΠ»Π΅ΠΌΠΌ Π±ΡƒΠ΄Π΅Ρ‚ ΠΌΠ΅Π½ΡΡ‚ΡŒΡΡ ΠΈ Π­Π”Π‘, ΡƒΡΠΊΠΎΡ€ΡΡŽΡ‰Π°Ρ ΠΈΠ»ΠΈ Π·Π°ΠΌΠ΅Π΄Π»ΡΡŽΡ‰Π°Ρ ΠΏΠ΅Ρ€Π΅ΠΌΠ΅Ρ‰Π΅Π½ΠΈΠ΅ зарядов.

Π€ΠΎΡ€ΠΌΡƒΠ»ΠΈΡ€ΠΎΠ²ΠΊΠ° Π·Π°ΠΊΠΎΠ½Π° Ома для ΠΏΠΎΠ»Π½ΠΎΠΉ Ρ†Π΅ΠΏΠΈ

Π—Π°ΠΊΠΎΠ½ Ома для ΠΏΠΎΠ»Π½ΠΎΠΉ Ρ†Π΅ΠΏΠΈ выраТаСтся ΠΏΠΎΠ²Π΅Π΄Π΅Π½ΠΈΠ΅ΠΌ основных Π²Π΅Π»ΠΈΡ‡ΠΈΠ½, Π±Ρ‹Π» Π²Ρ‹Π²Π΅Π΄Π΅Π½ ΠΎΠΏΡ‹Ρ‚Π½Ρ‹ΠΌ ΡΠΊΡΠΏΠ΅Ρ€ΠΈΠΌΠ΅Π½Ρ‚Π°Π»ΡŒΠ½Ρ‹ΠΌ ΠΏΡƒΡ‚Π΅ΠΌ. Π Π΅Π·ΡƒΠ»ΡŒΡ‚Π°Ρ‚ΠΎΠΌ стало выявлСниС связСй, ΠΎΠ±ΡŠΠ΅Π΄ΠΈΠ½ΡΡŽΡ‰ΠΈΡ… силу Ρ‚ΠΎΠΊΠ° (I), ΡΠ»Π΅ΠΊΡ‚Ρ€ΠΎΠ΄Π²ΠΈΠΆΡƒΡ‰ΡƒΡŽ силу – Π­Π”Π‘ (Π•), внСшнСС (R) ΠΈ Π²Π½ΡƒΡ‚Ρ€Π΅Π½Π½Π΅Π΅ (r) сопротивлСния Π² Ρ†Π΅ΠΏΠΈ.

Π’ тСорСтичСских расчСтах с Ρ‚ΠΎΡ‡ΠΊΠΈ зрСния чистой Ρ„ΠΈΠ·ΠΈΠΊΠΈ, Π² цСпях прСдполагался Ρ‚Π°ΠΊ Π½Π°Π·Ρ‹Π²Π°Π΅ΠΌΡ‹ΠΉ ΠΈΠ΄Π΅Π°Π»ΡŒΠ½Ρ‹ΠΉ источник постоянного Ρ‚ΠΎΠΊΠ°. ПослС Ρ‚ΠΎΠ³ΠΎ, ΠΊΠ°ΠΊ Π±Ρ‹Π»ΠΈ ΠΏΡ€ΠΎΠ²Π΅Π΄Π΅Π½Ρ‹ Ρ€Π΅Π°Π»ΡŒΠ½Ρ‹Π΅ исслСдования, Π²Ρ‹ΡΡΠ½ΠΈΠ»ΠΎΡΡŒ, Ρ‡Ρ‚ΠΎ источник Ρ‚ΠΎΠΊΠ° ΠΎΠ±Π»Π°Π΄Π°Π΅Ρ‚ собствСнным сопротивлСниСм.

Π€ΠΎΡ€ΠΌΡƒΠ»ΠΈΡ€ΠΎΠ²ΠΊΠ° Π·Π°ΠΊΠΎΠ½Π° Ома для ΠΏΠΎΠ»Π½ΠΎΠΉ Ρ†Π΅ΠΏΠΈ ΠΏΡ€ΠΈΠΎΠ±Ρ€Π΅Π»Π° ΡΠ»Π΅Π΄ΡƒΡŽΡ‰ΠΈΠΉ Π²ΠΈΠ΄: Π‘ΠΈΠ»Π° Ρ‚ΠΎΠΊΠ° находится Π² прямой ΠΏΡ€ΠΎΠΏΠΎΡ€Ρ†ΠΈΠΈ с суммой Π­Π”Π‘ Ρ†Π΅ΠΏΠΈ, ΠΈ Π² ΠΎΠ±Ρ€Π°Ρ‚Π½ΠΎΠΉ ΠΏΡ€ΠΎΠΏΠΎΡ€Ρ†ΠΈΠΈ с суммой сопротивлСний самой Ρ†Π΅ΠΏΠΈ ΠΈ источника Ρ‚ΠΎΠΊΠ°.

Π‘Π»Π΅Π΄ΡƒΠ΅Ρ‚ сразу ΠΆΠ΅ Π²Ρ‹ΡΡΠ½ΠΈΡ‚ΡŒ, Ρ‡Ρ‚ΠΎ Ρ‚Π°ΠΊΠΎΠ΅ элСктродвиТущая сила. По сути, ΠΎΠ½Π° являСтся физичСской Π²Π΅Π»ΠΈΡ‡ΠΈΠ½ΠΎΠΉ, Ρ…Π°Ρ€Π°ΠΊΡ‚Π΅Ρ€ΠΈΠ·ΡƒΡŽΡ‰Π΅ΠΉ дСйствиС Π²Π½Π΅ΡˆΠ½ΠΈΡ… сил источника Π­Π”Π‘. НапримСр, Π² простой Π±Π°Ρ‚Π°Ρ€Π΅ΠΉΠΊΠ΅ ΠΏΠ΅Ρ€Π΅ΠΌΠ΅Ρ‰Π΅Π½ΠΈΠ΅ зарядов происходит Π² Ρ€Π΅Π·ΡƒΠ»ΡŒΡ‚Π°Ρ‚Π΅ химичСской Ρ€Π΅Π°ΠΊΡ†ΠΈΠΈ. Π’ΠΎ Π΅ΡΡ‚ΡŒ, данная сила Π΄Π²ΠΈΠ³Π°Π΅Ρ‚ заряд, обСспСчивая ΠΎΠ±Ρ‰Π΅Π΅ Ρ‚Π΅Ρ‡Π΅Π½ΠΈΠ΅ элСктричСского Ρ‚ΠΎΠΊΠ°.

Π€ΠΎΡ€ΠΌΡƒΠ»Ρƒ Π·Π°ΠΊΠΎΠ½Π° Ома ΠΌΠΎΠΆΠ½ΠΎ Π·Π°ΠΏΠΈΡΠ°Ρ‚ΡŒ ΠΏΠΎ-Π΄Ρ€ΡƒΠ³ΠΎΠΌΡƒ. Π­Π”Π‘ источника Ρ‚ΠΎΠΊΠ° ΠΏΠΎΠ»Π½ΠΎΠΉ Ρ†Π΅ΠΏΠΈ прСдставляСт собой суммарныС падСния напряТСний Ρƒ самого источника ΠΈ Π²ΠΎ внСшнСй Ρ†Π΅ΠΏΠΈ:

ΠŸΠ΅Ρ€Π΅ΠΌΠ΅Π½Π½Ρ‹ΠΉ Ρ‚ΠΎΠΊ

Π’ ΠΎΡ‚Π»ΠΈΡ‡ΠΈΠ΅ ΠΎΡ‚ Ρ†Π΅ΠΏΠ΅ΠΉ, ΠΏΠΎ ΠΊΠΎΡ‚ΠΎΡ€Ρ‹ΠΌ Ρ‚Π΅Ρ‡Π΅Ρ‚ постоянный Ρ‚ΠΎΠΊ, Π² Ρ†Π΅ΠΏΠΈ ΠΏΠ΅Ρ€Π΅ΠΌΠ΅Π½Π½ΠΎΠ³ΠΎ Ρ‚ΠΎΠΊΠ° ΠΊΡ€ΠΎΠΌΠ΅ Π°ΠΊΡ‚ΠΈΠ²Π½ΠΎΠΉ Π½Π°Π³Ρ€ΡƒΠ·ΠΊΠΈ Π² Π²ΠΈΠ΄Π΅ ΠΏΠΎΡ‚Ρ€Π΅Π±ΠΈΡ‚Π΅Π»Π΅ΠΉ, входят элСмСнты с Ρ€Π΅Π°ΠΊΡ‚ΠΈΠ²Π½Ρ‹ΠΌ сопротивлСниСм. Π­Ρ‚ΠΎ Ρ€Π°Π·Π»ΠΈΡ‡Π½Ρ‹Π΅ Ρ‚ΠΈΠΏΡ‹ ΠΊΠ°Ρ‚ΡƒΡˆΠ΅ΠΊ ΠΈ кондСнсаторов, ΠΎΠ±Π»Π°Π΄Π°ΡŽΡ‰ΠΈΡ… ΠΈΠ½Π΄ΡƒΠΊΡ‚ΠΈΠ²Π½ΠΎΡΡ‚ΡŒΡŽ ΠΈ Π΅ΠΌΠΊΠΎΡΡ‚ΡŒΡŽ.

Π‘ ΡƒΠ²Π΅Π»ΠΈΡ‡Π΅Π½ΠΈΠ΅ΠΌ напряТСния Π±ΡƒΠ΄Π΅Ρ‚ расти ΠΈ сила Ρ‚ΠΎΠΊΠ°. Однако, ΠΊ Π°ΠΊΡ‚ΠΈΠ²Π½ΠΎΠΌΡƒ ΡΠΎΠΏΡ€ΠΎΡ‚ΠΈΠ²Π»Π΅Π½ΠΈΡŽ здСсь Π΄ΠΎΠ±Π°Π²Π»ΡΡŽΡ‚ΡΡ Ρ€Π΅Π°ΠΊΡ‚ΠΈΠ²Π½Ρ‹Π΅. Π‘ связи с этим, ΠΏΠΎΠ»Π½Ρ‹ΠΉ расклад для Ρ‚Π°ΠΊΠΎΠΉ Ρ†Π΅ΠΏΠΈ Π±ΡƒΠ΄Π΅Ρ‚ Π²Ρ‹Π³Π»ΡΠ΄Π΅Ρ‚ΡŒ Ρ‚Π°ΠΊ:

  • I = U/Z, Π³Π΄Π΅ I ΠΈ U – это сила Ρ‚ΠΎΠΊΠ° ΠΈ напряТСниС, Π° Z – являСтся ΠΏΠΎΠ»Π½Ρ‹ΠΌ сопротивлСниСм Ρ†Π΅ΠΏΠΈ.

ΠŸΠΎΠΊΠ°Π·Π°Ρ‚Π΅Π»ΡŒ Z слСдуСт Ρ€Π°ΡΡΠΌΠΎΡ‚Ρ€Π΅Ρ‚ΡŒ Π±ΠΎΠ»Π΅Π΅ ΠΏΠΎΠ΄Ρ€ΠΎΠ±Π½ΠΎ. ΠŸΡ€Π΅ΠΆΠ΄Π΅ всСго, это сумма, Π²ΠΊΠ»ΡŽΡ‡Π°ΡŽΡ‰Π°Ρ Π°ΠΊΡ‚ΠΈΠ²Π½ΠΎΠ΅, ΠΈΠ½Π΄ΡƒΠΊΡ‚ΠΈΠ²Π½ΠΎΠ΅ ΠΈ СмкостноС сопротивлСния. Π’ΠΎ Π΅ΡΡ‚ΡŒ, Π½Π° элСктричСский Ρ‚ΠΎΠΊ ΠΎΠΊΠ°Π·Ρ‹Π²Π°Π΅Ρ‚ влияниС Π½Π΅ Ρ‚ΠΎΠ»ΡŒΠΊΠΎ обычная омичСская Π½Π°Π³Ρ€ΡƒΠ·ΠΊΠ°, Π½ΠΎ Ρ‚Π°ΠΊΠΆΠ΅ Π΅ΠΌΠΊΠΎΡΡ‚ΡŒ (Π‘) ΠΈ ΠΈΠ½Π΄ΡƒΠΊΡ‚ΠΈΠ²Π½ΠΎΡΡ‚ΡŒ (L).

Π’ Ρ€Π΅Π·ΡƒΠ»ΡŒΡ‚Π°Ρ‚Π΅, краткая Ρ„ΠΎΡ€ΠΌΡƒΠ»Π° ΠΏΠΎΠ»Π½ΠΎΠ³ΠΎ сопротивлСния ΠΏΡ€ΠΈΠΌΠ΅Ρ‚ ΡΠ»Π΅Π΄ΡƒΡŽΡ‰ΠΈΠΉ Π²ΠΈΠ΄:

ΠžΠΏΡ‹Ρ‚Π½Ρ‹ΠΌ ΠΏΡƒΡ‚Π΅ΠΌ Π±Ρ‹Π»ΠΎ установлСно, Ρ‡Ρ‚ΠΎ Π² цСпях ΠΏΠ΅Ρ€Π΅ΠΌΠ΅Π½Π½ΠΎΠ³ΠΎ Ρ‚ΠΎΠΊΠ° Π½Π°Π±Π»ΡŽΠ΄Π°Π΅Ρ‚ΡΡ нСсовпадСниС ΠΏΠΎ Ρ„Π°Π·Π΅ ΠΊΠΎΠ»Π΅Π±Π°Π½ΠΈΠΉ Ρ‚ΠΎΠΊΠ° ΠΈ напряТСния. Π’Π΅Π»ΠΈΡ‡ΠΈΠ½Π° этих нСсовпадСний ΠΎΠ½Π° ΠΆΠ΅ Ρ€Π°Π·Π½ΠΈΡ†Π° Ρ„Π°Π· находится ΠΏΠΎΠ΄ нСпосрСдствСнным влияниСм индуктивности ΠΈ Смкости.

ИспользованиС Π½Π° ΠΏΡ€Π°ΠΊΡ‚ΠΈΠΊΠ΅

Π—Π°ΠΊΠΎΠ½ Ома Π»Π΅ΠΆΠΈΡ‚ Π² основС всСх расчСтов ΠΏΡ€ΠΎΠΈΠ·Π²ΠΎΠ΄ΠΈΠΌΡ‹Ρ… Π² элСктроникС ΠΈ элСктротСхникС. Π‘ΡƒΠ΄ΡƒΡ‰ΠΈΡ… спСциалистов с ΠΏΠ΅Ρ€Π²Ρ‹Ρ… Π΄Π½Π΅ΠΉ ΡƒΡ‡Π°Ρ‚, ΠΊΠ°ΠΊ ΠΈΡΠΏΠΎΠ»ΡŒΠ·ΠΎΠ²Π°Ρ‚ΡŒ Ρ‚Π°ΠΊ Π½Π°Π·Ρ‹Π²Π°Π΅ΠΌΡ‹ΠΉ Ρ‚Ρ€Π΅ΡƒΠ³ΠΎΠ»ΡŒΠ½ΠΈΠΊ. Π§Ρ‚ΠΎΠ±Ρ‹ Π½Π°ΠΉΡ‚ΠΈ ΠΊΠ°ΠΊΡƒΡŽ-Ρ‚ΠΎ ΠΈΡΠΊΠΎΠΌΡƒΡŽ Π²Π΅Π»ΠΈΡ‡ΠΈΠ½Ρƒ, Π΄ΠΎΠ»ΠΆΠ½Ρ‹ Π²Ρ‹ΠΏΠΎΠ»Π½ΡΡ‚ΡŒΡΡ простыС арифмСтичСскиС дСйствия. Если Π΄Π²Π° ΠΎΡΡ‚Π°Π²ΡˆΠΈΡ…ΡΡ ΠΏΠ°Ρ€Π°ΠΌΠ΅Ρ‚Ρ€Π° находятся Π² ΠΎΠ΄Π½ΠΎΠΉ строкС – ΠΎΠ½ΠΈ ΠΏΠ΅Ρ€Π΅ΠΌΠ½ΠΎΠΆΠ°ΡŽΡ‚ΡΡ. Если Π½Π° Ρ€Π°Π·Π½Ρ‹Ρ… уровнях, Ρ‚ΠΎ Π²Π΅Ρ€Ρ…Π½ΠΈΠΉ всСгда дСлится Π½Π° Π½ΠΈΠΆΠ½ΠΈΠΉ.

ΠŸΡ€Π°ΠΊΡ‚ΠΈΡ‡Π΅ΡΠΊΠΈ данная схСма выглядит Ρ‚Π°ΠΊ:

  • U = I x R, I = U/R, R = U/I.

Π‘Π°ΠΌΡ‹Π΅ простыС вычислСния производятся Π½Π° основС Π΄Π°Π½Π½Ρ‹Ρ… ΠΈΠ·ΠΌΠ΅Ρ€ΠΈΡ‚Π΅Π»ΡŒΠ½Ρ‹Ρ… ΠΏΡ€ΠΈΠ±ΠΎΡ€ΠΎΠ². На участкС Ρ†Π΅ΠΏΠΈ ΠΈΠ·ΠΌΠ΅Ρ€Π΅Π½ΠΈΠ΅ Ρ‚ΠΎΠΊΠ° выполняСтся Π°ΠΌΠΏΠ΅Ρ€ΠΌΠ΅Ρ‚Ρ€ΠΎΠΌ, Π° напряТСния – Π²ΠΎΠ»ΡŒΡ‚ΠΌΠ΅Ρ‚Ρ€ΠΎΠΌ. ПослС этого Π½Π°ΠΉΡ‚ΠΈ сопротивлСниС матСматичСским ΠΏΡƒΡ‚Π΅ΠΌ Π½Π΅ составит Ρ‚Ρ€ΡƒΠ΄Π°.

Для Π·Π°ΠΌΠ΅Ρ€ΠΎΠ² сопротивлСния Ρ‚ΠΎΠΆΠ΅ Π΅ΡΡ‚ΡŒ ΠΏΡ€ΠΈΠ±ΠΎΡ€ – ΠΎΠΌΠΌΠ΅Ρ‚Ρ€. ΠŸΠΎΠ»ΡƒΡ‡Π΅Π½Π½ΠΎΠ΅ Π²Ρ‹Ρ€Π°ΠΆΠ΅Π½ΠΈΠ΅, подставляСтся Π² ΠΎΠ΄Π½Ρƒ ΠΈΠ· Ρ„ΠΎΡ€ΠΌΡƒΠ», послС Ρ‡Π΅Π³ΠΎ находятся Π²Π΅Π»ΠΈΡ‡ΠΈΠ½Ρ‹ силы Ρ‚ΠΎΠΊΠ° ΠΈΠ»ΠΈ напряТСния. Π’ΠΎΡ‡Π½ΠΎΡΡ‚ΡŒ ΠΎΠΌΠΌΠ΅Ρ‚Ρ€Π° зависит ΠΎΡ‚ ΡΡ‚Π°Π±ΠΈΠ»ΡŒΠ½ΠΎΡΡ‚ΠΈ напряТСния, ΠΏΠΎΠ΄Π°Π²Π°Π΅ΠΌΠΎΠ³ΠΎ источником Ρ‚ΠΎΠΊΠ°. Бтабилизация проводится ΠΏΡƒΡ‚Π΅ΠΌ добавлСния рСзистора, Π²Ρ‹ΠΏΠΎΠ»Π½ΡΡŽΡ‰Π΅Π³ΠΎ Ρ„ΡƒΠ½ΠΊΡ†ΠΈΡŽ рСгулятора.

Иногда трСбуСтся ΠΈΡΠΊΠ»ΡŽΡ‡ΠΈΡ‚ΡŒ ΠΈΠ· схСмы ΠΊΠ°ΠΊΠΎΠΉ-Π½ΠΈΠ±ΡƒΠ΄ΡŒ элСмСнт Π±Π΅Π· Π΄Π΅ΠΌΠΎΠ½Ρ‚Π°ΠΆΠ°. Π‘ этой Ρ†Π΅Π»ΡŒΡŽ проводится ΡˆΡƒΠ½Ρ‚ΠΈΡ€ΠΎΠ²Π°Π½ΠΈΠ΅, ΠΊΠΎΠ³Π΄Π° приходится ΡƒΡΡ‚Π°Π½Π°Π²Π»ΠΈΠ²Π°Ρ‚ΡŒ ΠΏΡ€ΠΎΠ²ΠΎΠ΄Π½ΠΈΠΊ Π½Π° Π²Ρ…ΠΎΠ΄Π½Ρ‹Ρ… ΠΊΠ»Π΅ΠΌΠΌΠ°Ρ… Π½Π΅Π½ΡƒΠΆΠ½ΠΎΠ³ΠΎ рСзистора. Π’ΠΎΠΊ Π½Π°Ρ‡ΠΈΠ½Π°Π΅Ρ‚ ΠΈΠ΄Ρ‚ΠΈ Ρ‡Π΅Ρ€Π΅Π· ΡˆΡƒΠ½Ρ‚ с мСньшим сопротивлСниСм, Π° напряТСниС Π½Π° рСзисторС ΠΏΠ°Π΄Π°Π΅Ρ‚ Π΄ΠΎ нуля.

Π—Π°ΠΊΠΎΠ½ Ома ΠΈΡΠΏΠΎΠ»ΡŒΠ·ΡƒΠ΅Ρ‚ΡΡ Π² Π·Π°Ρ‰ΠΈΡ‚Π½Ρ‹Ρ… систСмах. Π­Ρ‚ΠΎ дСлаСтся с ΠΏΠΎΠΌΠΎΡ‰ΡŒΡŽ уставок, ΠΎΠ±Π΅ΡΠΏΠ΅Ρ‡ΠΈΠ²Π°ΡŽΡ‰ΠΈΡ… Π½ΠΎΡ€ΠΌΠ°Π»ΡŒΠ½ΡƒΡŽ Ρ€Π°Π±ΠΎΡ‚Ρƒ ΠΈ ΠΎΡ‚ΠΊΠ»ΡŽΡ‡Π°ΡŽΡ‰ΠΈΡ… ΠΏΠΈΡ‚Π°Π½ΠΈΠ΅ лишь Π² Π°Π²Π°Ρ€ΠΈΠΉΠ½Ρ‹Ρ… ситуациях.

Π€ΠΎΡ€ΠΌΡƒΠ»Ρ‹ для Π·Π°ΠΊΠΎΠ½Π° Ома

ΠŸΡ€Π΅Π΄ΡΡ‚Π°Π²Π»Π΅Π½Π½Ρ‹Π΅ Π½Π° рисункС Ρ„ΠΎΡ€ΠΌΡƒΠ»Ρ‹, Π½Π°Ρ‡Π°Π»ΠΈ Ρ„ΠΎΡ€ΠΌΠΈΡ€ΠΎΠ²Π°Ρ‚ΡŒΡΡ ΠΈΠ· основных Ρ„ΠΎΡ€ΠΌΡƒΠ» для ΠΏΠΎΠ»Π½ΠΎΠΉ Ρ†Π΅ΠΏΠΈ ΠΈ ΠΎΡ‚Π΄Π΅Π»ΡŒΠ½ΠΎΠ³ΠΎ участка. Π‘ ΠΈΡ… ΠΏΠΎΠΌΠΎΡ‰ΡŒΡŽ ΠΌΠΎΠΆΠ½ΠΎ Π²Ρ‹ΠΏΠΎΠ»Π½ΡΡ‚ΡŒ всС основныС расчСты, ΠΏΡ€ΠΈ составлСнии ΠΏΡ€ΠΎΠ΅ΠΊΡ‚ΠΎΠ² ΠΈ Π² Π΄Ρ€ΡƒΠ³ΠΈΡ… ситуациях. Π€ΠΎΡ€ΠΌΡƒΠ»Ρ‹ ΠΏΠΎΠ»Π½ΠΎΡΡ‚ΡŒΡŽ ΠΏΡ€ΠΈΠ³ΠΎΠ΄Π½Ρ‹ для Ρ€Π°Π±ΠΎΡ‚Ρ‹ с цСпями ΠΊΠ°ΠΊ постоянного, Ρ‚Π°ΠΊ ΠΈ ΠΏΠ΅Ρ€Π΅ΠΌΠ΅Π½Π½ΠΎΠ³ΠΎ Ρ‚ΠΎΠΊΠ°.

ВидСоинструкция

Π‘ΠΎΠΏΡ€ΠΎΡ‚ΠΈΠ²Π»Π΅Π½ΠΈΠ΅

— Π­Π»Π΅ΠΌΠ΅Π½Ρ‚Ρ‹ схСмы — Π‘ΠΎΠ΄Π΅Ρ€ΠΆΠ°Π½ΠΈΠ΅ MCAT

Π’ соотвСтствии с Π·Π°ΠΊΠΎΠ½ΠΎΠΌ Ома, ΠΏΠ°Π΄Π΅Π½ΠΈΠ΅ напряТСния , Π’ Π½Π° рСзисторС, ΠΊΠΎΠ³Π΄Π° Ρ‡Π΅Ρ€Π΅Π· Π½Π΅Π³ΠΎ ΠΏΡ€ΠΎΡ‚Π΅ΠΊΠ°Π΅Ρ‚ Ρ‚ΠΎΠΊ, рассчитываСтся ΠΏΠΎ Ρ„ΠΎΡ€ΠΌΡƒΠ»Π΅ V = IR, Π³Π΄Π΅ I — Ρ‚ΠΎΠΊ Π² Π°ΠΌΠΏΠ΅Ρ€Π°Ρ… (А), Π° R — сопротивлСниС Π² Ом (Ом).

Π’ΠΎΠΊ, ΠΏΡ€ΠΎΡ‚Π΅ΠΊΠ°ΡŽΡ‰ΠΈΠΉ Ρ‡Π΅Ρ€Π΅Π· Π±ΠΎΠ»ΡŒΡˆΠΈΠ½ΡΡ‚Π²ΠΎ вСщСств, прямо ΠΏΡ€ΠΎΠΏΠΎΡ€Ρ†ΠΈΠΎΠ½Π°Π»Π΅Π½ ΠΏΡ€ΠΈΠ»ΠΎΠΆΠ΅Π½Π½ΠΎΠΌΡƒ ΠΊ Π½Π΅ΠΌΡƒ Π½Π°ΠΏΡ€ΡΠΆΠ΅Π½ΠΈΡŽ Π’. . НСмСцкий Ρ„ΠΈΠ·ΠΈΠΊ Π“Π΅ΠΎΡ€Π³ Π‘ΠΈΠΌΠΎΠ½ Ом (1787–1854) Π±Ρ‹Π» ΠΏΠ΅Ρ€Π²Ρ‹ΠΌ, ΠΊΡ‚ΠΎ ΡΠΊΡΠΏΠ΅Ρ€ΠΈΠΌΠ΅Π½Ρ‚Π°Π»ΡŒΠ½ΠΎ продСмонстрировал, Ρ‡Ρ‚ΠΎ Ρ‚ΠΎΠΊ Π² мСталличСской ΠΏΡ€ΠΎΠ²ΠΎΠ»ΠΎΠΊΠ΅ прямо ΠΏΡ€ΠΎΠΏΠΎΡ€Ρ†ΠΈΠΎΠ½Π°Π»Π΅Π½ ΠΏΡ€ΠΈΠ»ΠΎΠΆΠ΅Π½Π½ΠΎΠΌΡƒ Π½Π°ΠΏΡ€ΡΠΆΠ΅Π½ΠΈΡŽ.МногиС вСщСства, для ΠΊΠΎΡ‚ΠΎΡ€Ρ‹Ρ… дСйствуСт Π·Π°ΠΊΠΎΠ½ Ома, Π½Π°Π·Ρ‹Π²Π°ΡŽΡ‚ΡΡ омичСскими. К Π½ΠΈΠΌ относятся Ρ…ΠΎΡ€ΠΎΡˆΠΈΠ΅ ΠΏΡ€ΠΎΠ²ΠΎΠ΄Π½ΠΈΠΊΠΈ, Ρ‚Π°ΠΊΠΈΠ΅ ΠΊΠ°ΠΊ мСдь ΠΈ алюминий, ΠΈ Π½Π΅ΠΊΠΎΡ‚ΠΎΡ€Ρ‹Π΅ ΠΏΠ»ΠΎΡ…ΠΈΠ΅ ΠΏΡ€ΠΎΠ²ΠΎΠ΄Π½ΠΈΠΊΠΈ ΠΏΡ€ΠΈ ΠΎΠΏΡ€Π΅Π΄Π΅Π»Π΅Π½Π½Ρ‹Ρ… ΠΎΠ±ΡΡ‚ΠΎΡΡ‚Π΅Π»ΡŒΡΡ‚Π²Π°Ρ…. ΠžΠΌΠΈΡ‡Π΅ΡΠΊΠΈΠ΅ ΠΌΠ°Ρ‚Π΅Ρ€ΠΈΠ°Π»Ρ‹ ΠΈΠΌΠ΅ΡŽΡ‚ сопротивлСниС R, ΠΊΠΎΡ‚ΠΎΡ€ΠΎΠ΅ Π½Π΅ зависит ΠΎΡ‚ напряТСния V ΠΈ Ρ‚ΠΎΠΊΠ° I. ΠžΠ±ΡŠΠ΅ΠΊΡ‚ с простым сопротивлСниСм называСтся рСзистором, Π΄Π°ΠΆΠ΅ Ссли Π΅Π³ΠΎ сопротивлСниС Π½Π΅Π²Π΅Π»ΠΈΠΊΠΎ.

РСзисторы ΠΏΠΎΡΠ»Π΅Π΄ΠΎΠ²Π°Ρ‚Π΅Π»ΡŒΠ½ΠΎ

РСзисторы

Π²ΠΊΠ»ΡŽΡ‡Π΅Π½Ρ‹ ΠΏΠΎΡΠ»Π΅Π΄ΠΎΠ²Π°Ρ‚Π΅Π»ΡŒΠ½ΠΎ всякий Ρ€Π°Π·, ΠΊΠΎΠ³Π΄Π° ΠΏΠΎΡ‚ΠΎΠΊ заряда ΠΈΠ»ΠΈ Ρ‚ΠΎΠΊ Π΄ΠΎΠ»ΠΆΠ΅Π½ ΠΏΡ€ΠΎΡ…ΠΎΠ΄ΠΈΡ‚ΡŒ Ρ‡Π΅Ρ€Π΅Π· ΠΊΠΎΠΌΠΏΠΎΠ½Π΅Π½Ρ‚Ρ‹ ΠΏΠΎΡΠ»Π΅Π΄ΠΎΠ²Π°Ρ‚Π΅Π»ΡŒΠ½ΠΎ.

ΠžΠ±Ρ‰Π΅Π΅ сопротивлСниС Π² Ρ†Π΅ΠΏΠΈ Ρ€Π°Π²Π½ΠΎ суммС ΠΎΡ‚Π΄Π΅Π»ΡŒΠ½Ρ‹Ρ… сопротивлСний.

ΠŸΠ°Ρ€Π°Π»Π»Π΅Π»ΡŒΠ½Ρ‹Π΅ рСзисторы

РСзисторы

Π²ΠΊΠ»ΡŽΡ‡Π΅Π½Ρ‹ ΠΏΠ°Ρ€Π°Π»Π»Π΅Π»ΡŒΠ½ΠΎ, ΠΊΠΎΠ³Π΄Π° ΠΊΠ°ΠΆΠ΄Ρ‹ΠΉ рСзистор ΠΏΠΎΠ΄ΠΊΠ»ΡŽΡ‡Π΅Π½ нСпосрСдствСнно ΠΊ источнику напряТСния ΠΏΡƒΡ‚Π΅ΠΌ соСдинСния ΠΏΡ€ΠΎΠ²ΠΎΠ΄ΠΎΠ² с Π½Π΅Π·Π½Π°Ρ‡ΠΈΡ‚Π΅Π»ΡŒΠ½Ρ‹ΠΌ сопротивлСниСм. Π’Π°ΠΊΠΈΠΌ ΠΎΠ±Ρ€Π°Π·ΠΎΠΌ, ΠΊ ΠΊΠ°ΠΆΠ΄ΠΎΠΌΡƒ рСзистору ΠΏΡ€ΠΈΠ»ΠΎΠΆΠ΅Π½ΠΎ ΠΏΠΎΠ»Π½ΠΎΠ΅ напряТСниС источника.

На ΠΊΠ°ΠΆΠ΄Ρ‹ΠΉ рСзистор Π² Ρ†Π΅ΠΏΠΈ подаСтся ΠΏΠΎΠ»Π½ΠΎΠ΅ напряТСниС. По Π·Π°ΠΊΠΎΠ½Ρƒ Ома Ρ‚ΠΎΠΊΠΈ, ΠΏΡ€ΠΎΡ‚Π΅ΠΊΠ°ΡŽΡ‰ΠΈΠ΅ Ρ‡Π΅Ρ€Π΅Π· ΠΎΡ‚Π΄Π΅Π»ΡŒΠ½Ρ‹Π΅ рСзисторы, Ρ€Π°Π²Π½Ρ‹ I1 = VR1

.

ΠžΠ±Ρ‰Π΅Π΅ сопротивлСниС Π² ΠΏΠ°Ρ€Π°Π»Π»Π΅Π»ΡŒΠ½ΠΎΠΉ Ρ†Π΅ΠΏΠΈ Ρ€Π°Π²Π½ΠΎ суммС ΠΎΠ±Ρ€Π°Ρ‚Π½Ρ‹Ρ… Π²Π΅Π»ΠΈΡ‡ΠΈΠ½ ΠΊΠ°ΠΆΠ΄ΠΎΠ³ΠΎ ΠΎΡ‚Π΄Π΅Π»ΡŒΠ½ΠΎΠ³ΠΎ сопротивлСния.

УдСльноС сопротивлСниС — это свойство ΠΌΠ°Ρ‚Π΅Ρ€ΠΈΠ°Π»Π°, ΠΊΠΎΡ‚ΠΎΡ€ΠΎΠ΅ количСствСнно опрСдСляСт, насколько сильно ΠΎΠ½ сопротивляСтся ΠΈΠ»ΠΈ ΠΏΡ€ΠΎΠ²ΠΎΠ΄ΠΈΡ‚ элСктричСский Ρ‚ΠΎΠΊ. НизкоС ΡƒΠ΄Π΅Π»ΡŒΠ½ΠΎΠ΅ сопротивлСниС ΡƒΠΊΠ°Π·Ρ‹Π²Π°Π΅Ρ‚ Π½Π° ΠΌΠ°Ρ‚Π΅Ρ€ΠΈΠ°Π», ΠΊΠΎΡ‚ΠΎΡ€Ρ‹ΠΉ Π»Π΅Π³ΠΊΠΎ пропускаСт элСктричСский Ρ‚ΠΎΠΊ, ΠΈ Π½Π°ΠΎΠ±ΠΎΡ€ΠΎΡ‚. РассчитываСтся ΠΊΠ°ΠΊ:

ρ = R β€’ A / L

R — элСктричСскоС сопротивлСниС ΠΎΠ΄Π½ΠΎΡ€ΠΎΠ΄Π½ΠΎΠ³ΠΎ ΠΎΠ±Ρ€Π°Π·Ρ†Π° ΠΌΠ°Ρ‚Π΅Ρ€ΠΈΠ°Π»Π°

Π» — Π΄Π»ΠΈΠ½Π° экзСмпляра

А — ΠΏΠ»ΠΎΡ‰Π°Π΄ΡŒ ΠΏΠΎΠΏΠ΅Ρ€Π΅Ρ‡Π½ΠΎΠ³ΠΎ сСчСния ΠΎΠ±Ρ€Π°Π·Ρ†Π°


ΠŸΡ€Π°ΠΊΡ‚ΠΈΡ‡Π΅ΡΠΊΠΈΠ΅ вопросы

Π₯анская акадСмия

Анализ сигналов напряТСния элСктрокардиограммы


ΠžΡ„ΠΈΡ†ΠΈΠ°Π»ΡŒΠ½Π°Ρ ΠΏΠΎΠ΄Π³ΠΎΡ‚ΠΎΠ²ΠΊΠ° MCAT (AAMC)

Π€ΠΈΠ·ΠΈΠΊΠ° — ΠΊΠ°Ρ€Ρ‚ΠΎΡ‡ΠΊΠΈ ΠΎΠ½Π»Π°ΠΉΠ½ Вопрос 1

Physics Question Pack ΠžΡ‚Ρ€Ρ‹Π²ΠΎΠΊ 9, вопрос 54

Physics Question Pack ΠžΡ‚Ρ€Ρ‹Π²ΠΎΠΊ 9, вопрос 56

ΠŸΠ°ΠΊΠ΅Ρ‚ вопросов ΠΏΠΎ Ρ„ΠΈΠ·ΠΈΠΊΠ΅, вопрос 117

БСкция Π±Π°Π½ΠΊΠ° C / P Вопрос 15 сСкции

БСкция Π±Π°Π½ΠΊΠ° C / P Вопрос 17 сСкции

ΠžΠ±Ρ€Π°Π·Π΅Ρ† тСста C / P Π Π°Π·Π΄Π΅Π» ΠžΡ‚Ρ€Ρ‹Π²ΠΎΠΊ 7 Вопрос 35

ΠŸΡ€Π°ΠΊΡ‚ΠΈΡ‡Π΅ΡΠΊΠΈΠΉ экзамСн 1 БСкция C / P ΠžΡ‚Ρ€Ρ‹Π²ΠΎΠΊ 10 Вопрос 52

ΠŸΡ€Π°ΠΊΡ‚ΠΈΡ‡Π΅ΡΠΊΠΈΠΉ экзамСн 2 Π Π°Π·Π΄Π΅Π» C / P, вопрос 59

ΠŸΡ€Π°ΠΊΡ‚ΠΈΡ‡Π΅ΡΠΊΠΈΠΉ экзамСн 3 Π Π°Π·Π΄Π΅Π» C / P ΠžΡ‚Ρ€Ρ‹Π²ΠΎΠΊ 7 Вопрос 39


ΠšΠ»ΡŽΡ‡Π΅Π²Ρ‹Π΅ Ρ‚ΠΎΡ‡ΠΊΠΈ

β€’ ПадСниС напряТСния V Π½Π° рСзисторС ΠΏΡ€ΠΈ ΠΏΡ€ΠΎΡ‚Π΅ΠΊΠ°Π½ΠΈΠΈ Ρ‡Π΅Ρ€Π΅Π· Π½Π΅Π³ΠΎ Ρ‚ΠΎΠΊΠ° рассчитываСтся ΠΏΠΎ Ρ„ΠΎΡ€ΠΌΡƒΠ»Π΅ V = IR

β€’ ΠžΠ±Ρ‰Π΅Π΅ сопротивлСниС Π² Ρ†Π΅ΠΏΠΈ Ρ€Π°Π²Π½ΠΎ суммС ΠΎΡ‚Π΄Π΅Π»ΡŒΠ½Ρ‹Ρ… сопротивлСний.

β€’ ΠžΠ±Ρ‰Π΅Π΅ сопротивлСниС Π² ΠΏΠ°Ρ€Π°Π»Π»Π΅Π»ΡŒΠ½ΠΎΠΉ Ρ†Π΅ΠΏΠΈ Ρ€Π°Π²Π½ΠΎ суммС ΠΎΠ±Ρ€Π°Ρ‚Π½Ρ‹Ρ… Π²Π΅Π»ΠΈΡ‡ΠΈΠ½ ΠΊΠ°ΠΆΠ΄ΠΎΠ³ΠΎ ΠΎΡ‚Π΄Π΅Π»ΡŒΠ½ΠΎΠ³ΠΎ сопротивлСния.

β€’ УдСльноС сопротивлСниС измСряСт, насколько сильно ΠΌΠ°Ρ‚Π΅Ρ€ΠΈΠ°Π» сопротивляСтся ΠΈΠ»ΠΈ ΠΏΡ€ΠΎΠ²ΠΎΠ΄ΠΈΡ‚ элСктричСский Ρ‚ΠΎΠΊ.


ΠšΠ»ΡŽΡ‡Π΅Π²Ρ‹Π΅ Ρ‚Π΅Ρ€ΠΌΠΈΠ½Ρ‹

Ρ‚ΠΎΠΊ : количСство заряда, ΠΏΠ΅Ρ€Π΅ΠΌΠ΅Ρ‰Π°ΡŽΡ‰Π΅Π³ΠΎΡΡ Ρ‡Π΅Ρ€Π΅Π· ΠΏΠΎΠΏΠ΅Ρ€Π΅Ρ‡Π½ΠΎΠ΅ сСчСниС Π·Π° ΠΏΠ΅Ρ€ΠΈΠΎΠ΄ Π²Ρ€Π΅ΠΌΠ΅Π½ΠΈ.

напряТСниС : Π Π°Π·Π½ΠΎΡΡ‚ΡŒ элСктричСских ΠΏΠΎΡ‚Π΅Π½Ρ†ΠΈΠ°Π»ΠΎΠ², выраТСнная Π² Π²ΠΎΠ»ΡŒΡ‚Π°Ρ…

сопротивлСниС : сопротивлСниС — это ΠΌΠ΅Ρ€Π° сопротивлСния Ρ‚ΠΎΠΊΡƒ, ΠΏΡ€ΠΎΡ‚Π΅ΠΊΠ°ΡŽΡ‰Π΅ΠΌΡƒ Π² элСктричСской Ρ†Π΅ΠΏΠΈ.

ΡƒΠ΄Π΅Π»ΡŒΠ½ΠΎΠ΅ сопротивлСниС: свойство ΠΌΠ°Ρ‚Π΅Ρ€ΠΈΠ°Π»Π°, ΠΊΠΎΡ‚ΠΎΡ€ΠΎΠ΅ количСствСнно опрСдСляСт, насколько сильно ΠΎΠ½ сопротивляСтся ΠΈΠ»ΠΈ ΠΏΡ€ΠΎΠ²ΠΎΠ΄ΠΈΡ‚ элСктричСский Ρ‚ΠΎΠΊ

ΠšΠ°Π»ΡŒΠΊΡƒΠ»ΡΡ‚ΠΎΡ€ Π·Π°ΠΊΠΎΠ½Π°

Ома | Ecomsa

Π”Ρ€ΡƒΠ³ΠΈΠΌΠΈ словами, Β«RΒ» — это постоянная ΠΈ нСзависимая Π΅Π΄ΠΈΠ½ΠΈΡ†Π° измСрСния Ρ‚ΠΎΠΊΠ°; Β«VΒ» ΠΈΠΌΠ΅Π΅Ρ‚ Π½Π΅Π±ΠΎΠ»ΡŒΡˆΡƒΡŽ ΠΏΠ»Π°Π²Π½ΠΎΡΡ‚ΡŒ Ρ‚ΠΎΠΊΠ° ΠΏΡ€ΠΈ Π±ΠΎΠ»Π΅Π΅ высоком сопротивлСнии, Π° Β«IΒ» прямо ΠΏΡ€ΠΎΠΏΠΎΡ€Ρ†ΠΈΠΎΠ½Π°Π»Π΅Π½ ΠΏΡ€ΠΈΠ»ΠΎΠΆΠ΅Π½Π½ΠΎΠΌΡƒ Π½Π°ΠΏΡ€ΡΠΆΠ΅Π½ΠΈΡŽ ΠΈ ΠΎΠ±Ρ€Π°Ρ‚Π½ΠΎ ΠΏΡ€ΠΎΠΏΠΎΡ€Ρ†ΠΈΠΎΠ½Π°Π»Π΅Π½ Π΅Π³ΠΎ ΡΠΎΠΏΡ€ΠΎΡ‚ΠΈΠ²Π»Π΅Π½ΠΈΡŽ. Π‘Π»Π΅Π΄ΠΎΠ²Π°Ρ‚Π΅Π»ΡŒΠ½ΠΎ, Π·Π°ΠΊΠΎΠ½ Ома фокусируСтся Π½Π° свойствах Π½Π΅ΠΊΠΎΡ‚ΠΎΡ€Ρ‹Ρ… ΠΌΠ°Ρ‚Π΅Ρ€ΠΈΠ°Π»ΠΎΠ².Однако это Π½Π΅ элСктромагнитный Π·Π°ΠΊΠΎΠ½, ΠΊΠ°ΠΊ Π·Π°ΠΊΠΎΠ½ Гаусса. На языкС ΠΌΠ°Ρ‚Π΅ΠΌΠ°Ρ‚ΠΈΠΊΠΈ это пСрСводится ΠΊΠ°ΠΊ V = IR

Π˜ΡΡ‚ΠΎΡ€ΠΈΡ Π·Π°ΠΊΠΎΠ½Π° Ома

Π”ΠΎ создания любого ΠΊΠ°Π»ΡŒΠΊΡƒΠ»ΡΡ‚ΠΎΡ€Π° сопротивлСния Π·Π°ΠΊΠΎΠ½ Ома Π±Ρ‹Π» Ρ€ΠΎΠΆΠ΄Π΅Π½ Π² 1827 Π³ΠΎΠ΄Ρƒ Π½Π΅ΠΌΠ΅Ρ†ΠΊΠΈΠΌ Ρ„ΠΈΠ·ΠΈΠΊΠΎΠΌ Π“Π΅ΠΎΡ€Π³ΠΎΠΌ Π‘ΠΈΠΌΠΎΠ½ΠΎΠΌ Омом. Он ΠΏΡ€ΠΎΠ²Π΅Π» ΠΎΠ±ΡˆΠΈΡ€Π½ΠΎΠ΅ исслСдованиС Π² области Π³Π°Π»ΡŒΠ²Π°Π½ΠΈΡ‡Π΅ΡΠΊΠΈΡ… ΠΏΠΎΡΠ»Π΅Π΄ΠΎΠ²Π°Ρ‚Π΅Π»ΡŒΠ½ΠΎΡΡ‚Π΅ΠΉ, ΠΎΠ±Π½Π°Ρ€ΡƒΠΆΠΈΠ² Π½Π΅ΠΊΠΎΡ‚ΠΎΡ€Ρ‹Π΅ значСния напряТСния ΠΈ Ρ‚ΠΎΠΊΠ°, ΠΏΡ€ΠΎΡ‚Π΅ΠΊΠ°ΡŽΡ‰ΠΈΠ΅ Ρ‡Π΅Ρ€Π΅Π· простыС элСктричСскиС Ρ†Π΅ΠΏΠΈ. Π’ настоящСС врСмя это расслСдованиС ΠΏΡ€ΠΈΠ²Π΅Π»ΠΎ ΠΊ ΠΏΡ€ΠΈΠ½ΡΡ‚ΠΈΡŽ Π·Π°ΠΊΠΎΠ½Π°, носящСго Π΅Π³ΠΎ имя.

Π’ связи с этим Ом ΠΏΠΎΠ»ΡƒΡ‡ΠΈΠ» мноТСство ΠΏΡ€ΠΈΠ·Π½Π°Π½ΠΈΠΉ ΠΈ Π½Π°Π³Ρ€Π°Π΄: Π² 1849 Π³ΠΎΠ΄Ρƒ ΠœΡŽΠ½Ρ…Π΅Π½ΡΠΊΠΈΠΉ унивСрситСт присвоил Π΅ΠΌΡƒ ΠΊΠ°Ρ„Π΅Π΄Ρ€Ρƒ профСссора Ρ„ΠΈΠ·ΠΈΠΊΠΈ, Π° Π² 1941 Π³ΠΎΠ΄Ρƒ ЛондонскоС ΠšΠΎΡ€ΠΎΠ»Π΅Π²ΡΠΊΠΎΠ΅ общСство Π½Π°Π³Ρ€Π°Π΄ΠΈΠ»ΠΎ Π΅Π³ΠΎ мСдалью Копли. Π΅Π΄ΠΈΠ½ΠΈΡ†Π° элСктричСского сопротивлСния послС Π½Π΅Π³ΠΎ, Ом.

ΠžΡΠΎΠ±Π΅Π½Π½ΠΎΡΡ‚ΠΈ Π·Π°ΠΊΠΎΠ½Π° Ома

  • ЭлСктричСскоС сопротивлСниС : это противодСйствиС ΠΈΠ»ΠΈ Π·Π°Ρ‚Ρ€ΡƒΠ΄Π½Π΅Π½ΠΈΠ΅, ΠΎΠ±Π½Π°Ρ€ΡƒΠΆΠΈΠ²Π°Π΅ΠΌΠΎΠ΅ Ρ‚ΠΎΠΊΠΎΠΌ Π² Π·Π°ΠΌΠΊΠ½ΡƒΡ‚ΠΎΠΉ Ρ†Π΅ΠΏΠΈ, ΠΊΠΎΡ‚ΠΎΡ€ΠΎΠ΅ ΡƒΠΌΠ΅Π½ΡŒΡˆΠ°Π΅Ρ‚ свободный ΠΏΠΎΡ‚ΠΎΠΊ элСктронов.Π•Π΄ΠΈΠ½ΠΈΡ†Π΅ΠΉ измСрСния сопротивлСния являСтся ΠΎΠΌ (R o Ξ©), Ρ‡Ρ‚ΠΎ ΠΎΠ·Π½Π°Ρ‡Π°Π΅Ρ‚, Ρ‡Ρ‚ΠΎ сопротивлСниС, обСспСчиваСмоС ΠΏΡ€ΠΎΠ²ΠΎΠ΄Π½ΠΈΠΊΠΎΠΌ, ΠΊΠΎΠ³Π΄Π° Ρ‡Π΅Ρ€Π΅Π· Π½Π΅Π³ΠΎ ΠΈ ΠΌΠ΅ΠΆΠ΄Ρƒ Π΅Π³ΠΎ ΠΊΡ€Π°ΠΉΠ½ΠΈΠΌΠΈ значСниями Ρ†ΠΈΡ€ΠΊΡƒΠ»ΠΈΡ€ΡƒΠ΅Ρ‚ Π°ΠΌΠΏΠ΅Ρ€ (сила), Π΄Π°Π΅Ρ‚ Ρ€Π°Π·Π½ΠΎΡΡ‚ΡŒ ΠΏΠΎΡ‚Π΅Π½Ρ†ΠΈΠ°Π»ΠΎΠ² (напряТСниС) Π² ΠΎΠ΄ΠΈΠ½ Π²ΠΎΠ»ΡŒΡ‚.
  • Ом : это Π΅Π΄ΠΈΠ½ΠΈΡ†Π° элСктричСского сопротивлСния, ΠΈ ΠΎΠ΄ΠΈΠ½ ΠΎΠΌ Ρ€Π°Π²Π΅Π½ ΠΎΠ΄Π½ΠΎΠΌΡƒ Π°ΠΌΠΏΠ΅Ρ€Ρƒ Ρ‚ΠΎΠΊΠ°, ΠΏΡ€ΠΎΡ‚Π΅ΠΊΠ°ΡŽΡ‰Π΅Π³ΠΎ ΠΏΡ€ΠΈ ΠΏΡ€ΠΈΠ»ΠΎΠΆΠ΅Π½ΠΈΠΈ напряТСния Π² ΠΎΠ΄ΠΈΠ½ Π²ΠΎΠ»ΡŒΡ‚. ВсС Ρ†Π΅ΠΏΠΈ ΠΈΠΌΠ΅ΡŽΡ‚ ΠΎΠΏΡ€Π΅Π΄Π΅Π»Π΅Π½Π½ΡƒΡŽ ΡΡ‚Π΅ΠΏΠ΅Π½ΡŒ сопротивлСния (ΠΈΠ»ΠΈ сопротивлСния) Ρ‚ΠΎΠΊΡƒ, Π² Ρ€Π΅Π·ΡƒΠ»ΡŒΡ‚Π°Ρ‚Π΅ Ρ‡Π΅Π³ΠΎ Ρ„ΠΎΡ€ΠΌΡƒΠ»Π° Ома R = V / I.Π”Ρ€ΡƒΠ³ΠΈΠΌΠΈ словами, ΡƒΠ²Π΅Π»ΠΈΡ‡Π΅Π½ΠΈΠ΅ Ρ‚ΠΎΠΊΠ° ΠΏΡ€ΠΈ Ρ‚ΠΎΠΌ ΠΆΠ΅ напряТСнии ΡƒΠΌΠ΅Π½ΡŒΡˆΠΈΡ‚ сопротивлСниС.
  • Π’ΠΎΠ»ΡŒΡ‚ : это Π΅Π΄ΠΈΠ½ΠΈΡ†Π° элСктродвиТущСй силы ΠΈΠ»ΠΈ элСктричСского давлСния (B), рСгулярно прикладываСмая ΠΊ Ρ†Π΅ΠΏΠΈ, ΠΈΠΌΠ΅ΡŽΡ‰Π΅ΠΉ сопротивлСниС Π² ΠΎΠ΄ΠΈΠ½ Ом, которая ΠΏΡ€ΠΎΠΈΠ·Π²ΠΎΠ΄ΠΈΡ‚ Ρ‚ΠΎΠΊ Π² ΠΎΠ΄ΠΈΠ½ Π°ΠΌΠΏΠ΅Ρ€. Π’ Π΄Π²ΡƒΡ… словах, Π²ΠΎΠ΄Ρƒ, Ρ‚Π΅ΠΊΡƒΡ‰ΡƒΡŽ ΠΏΠΎ ΠΌΠ΅Π΄Π½ΠΎΠΉ Ρ‚Ρ€ΡƒΠ±ΠΊΠ΅, ΠΌΠΎΠΆΠ½ΠΎ ΡΡ‡ΠΈΡ‚Π°Ρ‚ΡŒ Ρ€Π°Π²Π½ΠΎΠΉ Π½Π°ΠΏΡ€ΡΠΆΠ΅Π½ΠΈΡŽ, ΠΏΡ€ΠΎΡ‚Π΅ΠΊΠ°ΡŽΡ‰Π΅ΠΌΡƒ ΠΏΠΎ элСктричСскому кабСлю; ΠΏΠΎΡ‚ΠΎΠΌΡƒ Ρ‡Ρ‚ΠΎ для Π΅Π³ΠΎ двиТСния трСбуСтся сила, Π° сопротивлСниС этому ΠΏΠΎΡ‚ΠΎΠΊΡƒ измСряСтся Π² Π°ΠΌΠΏΠ΅Ρ€Π°Ρ….

АмпСр : это стандартная Π΅Π΄ΠΈΠ½ΠΈΡ†Π° измСрСния элСктричСского Ρ‚ΠΎΠΊΠ°, которая создаСтся Π΄Π°Π²Π»Π΅Π½ΠΈΠ΅ΠΌ Π² ΠΎΠ΄ΠΈΠ½ Π²ΠΎΠ»ΡŒΡ‚ Π² Ρ†Π΅ΠΏΠΈ с сопротивлСниСм Π² ΠΎΠ΄ΠΈΠ½ ΠΎΠΌ.

Π€ΠΎΡ€ΠΌΡƒΠ»Π° Π’Π°Ρ‚Ρ‚Π°, Π€ΠΎΡ€ΠΌΡƒΠ»Π° Ома ΠΈ Π€ΠΎΡ€ΠΌΡƒΠ»Π° АмпСра — ПониманиС Π—Π°ΠΊΠΎΠ½Π° Ома

Из-Π·Π° наличия ΠΌΠ°Ρ‚Π΅Ρ€ΠΈΠ°Π»ΠΎΠ² ΡƒΠΌΠ΅Π½ΡŒΡˆΠΈΡ‚Π΅ элСктричСский Ρ‚ΠΎΠΊ, ΠΏΡ€ΠΎΡ‚Π΅ΠΊΠ°ΡŽΡ‰ΠΈΠΉ Ρ‡Π΅Ρ€Π΅Π· Π½ΠΈΡ…, Π° ΠΏΡ€ΠΈ ΠΈΠ·ΠΌΠ΅Π½Π΅Π½ΠΈΠΈ ΠΈΡ… сопротивлСния Π·Π½Π°Ρ‡Π΅Π½ΠΈΠ΅ силы Ρ‚ΠΎΠΊΠ° Π² Π°ΠΌΠΏΠ΅Ρ€Π°Ρ… Ρ‚Π°ΠΊΠΆΠ΅ измСняСтся ΠΎΠ±Ρ€Π°Ρ‚Π½ΠΎ ΠΏΡ€ΠΎΠΏΠΎΡ€Ρ†ΠΈΠΎΠ½Π°Π»ΡŒΠ½ΠΎ.По ΠΌΠ΅Ρ€Π΅ увСличСния сопротивлСния Ρ‚ΠΎΠΊ ΡƒΠΌΠ΅Π½ΡŒΡˆΠ°Π΅Ρ‚ΡΡ, Π° ΠΏΠΎ ΠΌΠ΅Ρ€Π΅ ΡƒΠΌΠ΅Π½ΡŒΡˆΠ΅Π½ΠΈΡ сопротивлСния Ρ‚ΠΎΠΊ увСличиваСтся. Π’ ΠΎΠ±ΠΎΠΈΡ… случаях Π·Π½Π°Ρ‡Π΅Π½ΠΈΠ΅ напряТСния Ρ‚Ρ€Π΅Π±ΡƒΠ΅Ρ‚ постоянного поддСрТания.

Π‘Π»Π΅Π΄ΠΎΠ²Π°Ρ‚Π΅Π»ΡŒΠ½ΠΎ, Π·Π°ΠΊΠΎΠ½ Ома Ρ€Π°Π±ΠΎΡ‚Π°Π΅Ρ‚ для Ρ†Π΅ΠΏΠ΅ΠΉ ΠΈ пассивных участков Ρ†Π΅ΠΏΠΈ, ΠΊΠΎΡ‚ΠΎΡ€Ρ‹Π΅ Π°) ΠΈΠΌΠ΅ΡŽΡ‚ ΠΈΡΠΊΠ»ΡŽΡ‡ΠΈΡ‚Π΅Π»ΡŒΠ½ΠΎ рСзистивныС Π½Π°Π³Ρ€ΡƒΠ·ΠΊΠΈ (Π½ΠΎ Π½Π΅ ΠΈΠ½Π΄ΡƒΠΊΡ‚ΠΈΠ²Π½Ρ‹Π΅ ΠΈΠ»ΠΈ СмкостныС) ΠΈΠ»ΠΈ Π±) ΠΈΠΌΠ΅ΡŽΡ‚ постоянный Ρ€Π΅ΠΆΠΈΠΌ. Π’ ΠΎΠ±ΠΎΠΈΡ… случаях Π½Π° Π·Π½Π°Ρ‡Π΅Π½ΠΈΠ΅ сопротивлСния ΠΏΡ€ΠΎΠ²ΠΎΠ΄Π½ΠΈΠΊΠ° ΠΌΠΎΠΆΠ΅Ρ‚ Π²Π»ΠΈΡΡ‚ΡŒ Ρ‚Π΅ΠΌΠΏΠ΅Ρ€Π°Ρ‚ΡƒΡ€Π°. Π‘Π»Π΅Π΄ΠΎΠ²Π°Ρ‚Π΅Π»ΡŒΠ½ΠΎ, с Ρ‚ΠΎΡ‡ΠΊΠΈ зрСния Ρ„ΠΈΠ·ΠΈΠΊΠΈ, любоС устройство ΠΈΠ»ΠΈ ΠΌΠ°Ρ‚Π΅Ρ€ΠΈΠ°Π», вставлСнныС Π² ΡΠ»Π΅ΠΊΡ‚Ρ€ΠΈΡ‡Π΅ΡΠΊΡƒΡŽ Ρ†Π΅ΠΏΡŒ, Π²Ρ‹Π·Ρ‹Π²Π°ΡŽΡ‚ сопротивлСниС Π² Ρ‚ΠΎΠΊΠ΅.Π­Ρ‚ΠΎ сопротивлСниС ΠΌΠΎΠΆΠ΅Ρ‚ Π±Ρ‹Ρ‚ΡŒ ΡƒΠ²Π΅Π»ΠΈΡ‡Π΅Π½ΠΎ ΠΈΠ»ΠΈ ΡƒΠΌΠ΅Π½ΡŒΡˆΠ΅Π½ΠΎ Π² зависимости ΠΎΡ‚ ΠΈΡΠΏΠΎΠ»ΡŒΠ·ΡƒΠ΅ΠΌΠΎΠ³ΠΎ ΠΌΠ°Ρ‚Π΅Ρ€ΠΈΠ°Π»Π°.

Π§Ρ‚ΠΎΠ±Ρ‹ Ρ€Π°ΡΡΡ‡ΠΈΡ‚Π°Ρ‚ΡŒ сопротивлСниС ΠΌΠ°Ρ‚Π΅Ρ€ΠΈΠ°Π»Π° ΠΎΠΏΡ€Π΅Π΄Π΅Π»Π΅Π½Π½ΠΎΠΉ Π΄Π»ΠΈΠ½Ρ‹ ΠΈ Ρ‚ΠΎΠ»Ρ‰ΠΈΠ½Ρ‹, ΠΌΡ‹ Π΄ΠΎΠ»ΠΆΠ½Ρ‹ ΠΏΡ€ΠΈΠΌΠ΅Π½ΠΈΡ‚ΡŒ Ρ„ΠΎΡ€ΠΌΡƒΠ»Ρƒ Ома:

Π­Ρ‚ΠΎ ΠΎΠ·Π½Π°Ρ‡Π°Π΅Ρ‚, Ρ‡Ρ‚ΠΎ R Ρ€Π°Π²Π½ΠΎ rho (ρ), ΡƒΠΌΠ½ΠΎΠΆΠ΅Π½Π½ΠΎΠΌΡƒ Π½Π° Π΄Π»ΠΈΠ½Ρƒ ΠΏΡ€ΠΎΠ²ΠΎΠ΄Π½ΠΈΠΊΠ° (L) ΠΈ Ρ€Π°Π·Π΄Π΅Π»Π΅Π½Π½ΠΎΠΌΡƒ Π½Π° ΠΏΡ€ΠΎΠ²ΠΎΠ΄Π½ΠΈΠΊ. сСчСниС ΠΈΠ»ΠΈ Ρ‚ΠΎΠ»Ρ‰ΠΈΠ½Ρƒ (ΠΎΠ±Π»Π°ΡΡ‚ΡŒ S). Π“Π΄Π΅ rho (ρ) — постоянная Π²Π΅Π»ΠΈΡ‡ΠΈΠ½Π°, называСмая ΡƒΠ΄Π΅Π»ΡŒΠ½Ρ‹ΠΌ сопротивлСниСм; L — Π΄Π»ΠΈΠ½Π° ΠΆΠΈΠ»Ρ‹ кабСля Π² ΠΌΠ΅Ρ‚Ρ€Π°Ρ…, Π° S — сСчСниС ΠΈΠ»ΠΈ Ρ‚ΠΎΠ»Ρ‰ΠΈΠ½Π° ΠΆΠΈΠ»Ρ‹ кабСля Π² ΠΌΠΌ2.

Для получСния Π΄ΠΎΠΏΠΎΠ»Π½ΠΈΡ‚Π΅Π»ΡŒΠ½ΠΎΠΉ ΠΈΠ½Ρ„ΠΎΡ€ΠΌΠ°Ρ†ΠΈΠΈ ΠΌΡ‹ подСлимся Ρ‚Π°Π±Π»ΠΈΡ†Π΅ΠΉ с Π½Π΅ΠΊΠΎΡ‚ΠΎΡ€Ρ‹ΠΌΠΈ значСниями rho (ρ) Π² зависимости ΠΎΡ‚ Ρ‚ΠΈΠΏΠ° проводящСго ΠΌΠ°Ρ‚Π΅Ρ€ΠΈΠ°Π»Π°:

Для расчСта Π·Π½Π°Ρ‡Π΅Π½ΠΈΠΉ рСзистора Π½Π°ΠΌ ΡƒΠΆΠ΅ извСстна постоянная ΡƒΠ΄Π΅Π»ΡŒΠ½ΠΎΠ³ΠΎ сопротивлСния (ρ), поэтому ΠΌΡ‹ Π΄ΠΎΠ»ΠΆΠ½Ρ‹ ΠΎΠΏΡ€Π΅Π΄Π΅Π»ΠΈΡ‚ΡŒ Π΄Π»ΠΈΠ½Ρƒ ΠΏΡ€ΠΎΠ²ΠΎΠ΄Π½ΠΈΠΊΠ° (L) ΠΈ сСчСниС (S). Π—Π½Π°Ρ‡Π΅Π½ΠΈΠ΅:

  • Π§Π΅ΠΌ большС Π΄Π»ΠΈΠ½Π°, Ρ‚Π΅ΠΌ Π²Ρ‹ΡˆΠ΅ сопротивлСниС.
  • Π§Π΅ΠΌ мСньшС Π΄Π»ΠΈΠ½Π°, Ρ‚Π΅ΠΌ мСньшС сопротивлСниС.
  • Π§Π΅ΠΌ Π΄Π»ΠΈΠ½Π½Π΅Π΅ ΠΎΡ‚Ρ€Π΅Π·ΠΎΠΊ, Ρ‚Π΅ΠΌ мСньшС сопротивлСниС.
  • Π§Π΅ΠΌ мСньшС сСчСниС, Ρ‚Π΅ΠΌ Π²Ρ‹ΡˆΠ΅ сопротивлСниС.

ΠŸΡ€ΠΎΠ°Π½Π°Π»ΠΈΠ·ΠΈΡ€ΠΎΠ²Π°Π² эти Ρ‡Π΅Ρ‚Ρ‹Ρ€Π΅ утвСрТдСния, ΠΌΡ‹ сдСлаСм Π²Ρ‹Π²ΠΎΠ΄, Ρ‡Ρ‚ΠΎ Π·Π½Π°Ρ‡Π΅Π½ΠΈΠ΅ сопротивлСния прямо ΠΏΡ€ΠΎΠΏΠΎΡ€Ρ†ΠΈΠΎΠ½Π°Π»ΡŒΠ½ΠΎ Π΄Π»ΠΈΠ½Π΅ ΠΏΡ€ΠΎΠ²ΠΎΠ΄Π½ΠΈΠΊΠ° ΠΈ ΠΎΠ±Ρ€Π°Ρ‚Π½ΠΎ ΠΏΡ€ΠΎΠΏΠΎΡ€Ρ†ΠΈΠΎΠ½Π°Π»ΡŒΠ½ΠΎ Π΅Π³ΠΎ ΡΠ΅Ρ‡Π΅Π½ΠΈΡŽ.

УдСльноС сопротивлСниС — AP Physics 1

Если Π²Ρ‹ считаСтС, Ρ‡Ρ‚ΠΎ ΠΊΠΎΠ½Ρ‚Π΅Π½Ρ‚, доступный Ρ‡Π΅Ρ€Π΅Π· Π’Π΅Π±-сайт (ΠΊΠ°ΠΊ ΠΎΠΏΡ€Π΅Π΄Π΅Π»Π΅Π½ΠΎ Π² Π½Π°ΡˆΠΈΡ… Условиях обслуТивания), Π½Π°Ρ€ΡƒΡˆΠ°Π΅Ρ‚ ΠΈΠ»ΠΈ Π΄Ρ€ΡƒΠ³ΠΈΠ΅ ваши авторскиС ΠΏΡ€Π°Π²Π°, сообщитС Π½Π°ΠΌ, ΠΎΡ‚ΠΏΡ€Π°Π²ΠΈΠ² письмСнноС ΡƒΠ²Π΅Π΄ΠΎΠΌΠ»Π΅Π½ΠΈΠ΅ (Β«Π£Π²Π΅Π΄ΠΎΠΌΠ»Π΅Π½ΠΈΠ΅ ΠΎ Π½Π°Ρ€ΡƒΡˆΠ΅Π½ΠΈΠΈΒ»), содСрТащСС Π² ΠΈΠ½Ρ„ΠΎΡ€ΠΌΠ°Ρ†ΠΈΡŽ, ΠΎΠΏΠΈΡΠ°Π½Π½ΡƒΡŽ Π½ΠΈΠΆΠ΅, Π½Π°Π·Π½Π°Ρ‡Π΅Π½Π½ΠΎΠΌΡƒ Π½ΠΈΠΆΠ΅ Π°Π³Π΅Π½Ρ‚Ρƒ.Если Ρ€Π΅ΠΏΠ΅Ρ‚ΠΈΡ‚ΠΎΡ€Ρ‹ унивСрситСта ΠΏΡ€Π΅Π΄ΠΏΡ€ΠΈΠΌΡƒΡ‚ дСйствия Π² ΠΎΡ‚Π²Π΅Ρ‚ Π½Π° Π°Π½ Π£Π²Π΅Π΄ΠΎΠΌΠ»Π΅Π½ΠΈΠ΅ ΠΎ Π½Π°Ρ€ΡƒΡˆΠ΅Π½ΠΈΠΈ, ΠΎΠ½ΠΎ ΠΏΡ€Π΅Π΄ΠΏΡ€ΠΈΠΌΠ΅Ρ‚ Π΄ΠΎΠ±Ρ€ΠΎΡΠΎΠ²Π΅ΡΡ‚Π½ΡƒΡŽ ΠΏΠΎΠΏΡ‹Ρ‚ΠΊΡƒ ΡΠ²ΡΠ·Π°Ρ‚ΡŒΡΡ со стороной, которая прСдоставила Ρ‚Π°ΠΊΠΎΠΉ ΠΊΠΎΠ½Ρ‚Π΅Π½Ρ‚ срСдствами самого послСднСго адрСса элСктронной ΠΏΠΎΡ‡Ρ‚Ρ‹, Ссли Ρ‚Π°ΠΊΠΎΠ²ΠΎΠΉ имССтся, прСдоставлСнного Ρ‚Π°ΠΊΠΎΠΉ стороной Varsity Tutors.

Π’Π°ΡˆΠ΅ Π£Π²Π΅Π΄ΠΎΠΌΠ»Π΅Π½ΠΈΠ΅ ΠΎ Π½Π°Ρ€ΡƒΡˆΠ΅Π½ΠΈΠΈ ΠΏΡ€Π°Π² ΠΌΠΎΠΆΠ΅Ρ‚ Π±Ρ‹Ρ‚ΡŒ ΠΎΡ‚ΠΏΡ€Π°Π²Π»Π΅Π½ΠΎ сторонС, ΠΏΡ€Π΅Π΄ΠΎΡΡ‚Π°Π²ΠΈΠ²ΡˆΠ΅ΠΉ доступ ΠΊ ΠΊΠΎΠ½Ρ‚Π΅Π½Ρ‚Ρƒ, ΠΈΠ»ΠΈ Ρ‚Ρ€Π΅Ρ‚ΡŒΠΈΠΌ Π»ΠΈΡ†Π°ΠΌ, Ρ‚Π°ΠΊΠΈΠΌ ΠΊΠ°ΠΊ Π² качСствС ChillingEffects.org.

ΠžΠ±Ρ€Π°Ρ‚ΠΈΡ‚Π΅ Π²Π½ΠΈΠΌΠ°Π½ΠΈΠ΅, Ρ‡Ρ‚ΠΎ Π²Ρ‹ Π±ΡƒΠ΄Π΅Ρ‚Π΅ нСсти ΠΎΡ‚Π²Π΅Ρ‚ΡΡ‚Π²Π΅Π½Π½ΠΎΡΡ‚ΡŒ Π·Π° ΡƒΡ‰Π΅Ρ€Π± (Π²ΠΊΠ»ΡŽΡ‡Π°Ρ расходы ΠΈ Π³ΠΎΠ½ΠΎΡ€Π°Ρ€Ρ‹ Π°Π΄Π²ΠΎΠΊΠ°Ρ‚Π°ΠΌ), Ссли Π²Ρ‹ сущСствСнно ΠΈΡΠΊΠ°ΠΆΠ°Ρ‚ΡŒ ΠΈΠ½Ρ„ΠΎΡ€ΠΌΠ°Ρ†ΠΈΡŽ ΠΎ Ρ‚ΠΎΠΌ, Ρ‡Ρ‚ΠΎ ΠΏΡ€ΠΎΠ΄ΡƒΠΊΡ‚ ΠΈΠ»ΠΈ дСйствиС Π½Π°Ρ€ΡƒΡˆΠ°Π΅Ρ‚ ваши авторскиС ΠΏΡ€Π°Π²Π°.Π’Π°ΠΊΠΈΠΌ ΠΎΠ±Ρ€Π°Π·ΠΎΠΌ, Ссли Π²Ρ‹ Π½Π΅ ΡƒΠ²Π΅Ρ€Π΅Π½Ρ‹, Ρ‡Ρ‚ΠΎ ΠΊΠΎΠ½Ρ‚Π΅Π½Ρ‚ находится Π½Π° Π’Π΅Π±-сайтС ΠΈΠ»ΠΈ ΠΏΠΎ ссылкС с Π½Π΅Π³ΠΎ Π½Π°Ρ€ΡƒΡˆΠ°Π΅Ρ‚ ваши авторскиС ΠΏΡ€Π°Π²Π°, Π²Π°ΠΌ слСдуСт сначала ΠΎΠ±Ρ€Π°Ρ‚ΠΈΡ‚ΡŒΡΡ ΠΊ ΡŽΡ€ΠΈΡΡ‚Ρƒ.

Π§Ρ‚ΠΎΠ±Ρ‹ ΠΎΡ‚ΠΏΡ€Π°Π²ΠΈΡ‚ΡŒ ΡƒΠ²Π΅Π΄ΠΎΠΌΠ»Π΅Π½ΠΈΠ΅, Π²Ρ‹ΠΏΠΎΠ»Π½ΠΈΡ‚Π΅ ΡΠ»Π΅Π΄ΡƒΡŽΡ‰ΠΈΠ΅ дСйствия:

Π’Ρ‹ Π΄ΠΎΠ»ΠΆΠ½Ρ‹ Π²ΠΊΠ»ΡŽΡ‡ΠΈΡ‚ΡŒ ΡΠ»Π΅Π΄ΡƒΡŽΡ‰Π΅Π΅:

ЀизичСская ΠΈΠ»ΠΈ элСктронная подпись правообладатСля ΠΈΠ»ΠΈ Π»ΠΈΡ†Π°, ΡƒΠΏΠΎΠ»Π½ΠΎΠΌΠΎΡ‡Π΅Π½Π½ΠΎΠ³ΠΎ Π΄Π΅ΠΉΡΡ‚Π²ΠΎΠ²Π°Ρ‚ΡŒ ΠΎΡ‚ ΠΈΡ… ΠΈΠΌΠ΅Π½ΠΈ; Π˜Π΄Π΅Π½Ρ‚ΠΈΡ„ΠΈΠΊΠ°Ρ†ΠΈΡ авторских ΠΏΡ€Π°Π², ΠΊΠΎΡ‚ΠΎΡ€Ρ‹Π΅, ΠΊΠ°ΠΊ утвСрТдаСтся, Π±Ρ‹Π»ΠΈ Π½Π°Ρ€ΡƒΡˆΠ΅Π½Ρ‹; ОписаниС Ρ…Π°Ρ€Π°ΠΊΡ‚Π΅Ρ€Π° ΠΈ Ρ‚ΠΎΡ‡Π½ΠΎΠ³ΠΎ мСстонахоТдСния ΠΊΠΎΠ½Ρ‚Π΅Π½Ρ‚Π°, ΠΊΠΎΡ‚ΠΎΡ€Ρ‹ΠΉ, ΠΏΠΎ Π²Π°ΡˆΠ΅ΠΌΡƒ мнСнию, Π½Π°Ρ€ΡƒΡˆΠ°Π΅Ρ‚ ваши авторскиС ΠΏΡ€Π°Π²Π°, Π² \ достаточно подробностСй, Ρ‡Ρ‚ΠΎΠ±Ρ‹ ΠΏΠΎΠ·Π²ΠΎΠ»ΠΈΡ‚ΡŒ Ρ€Π΅ΠΏΠ΅Ρ‚ΠΈΡ‚ΠΎΡ€Π°ΠΌ унивСрситСтских школ Π½Π°ΠΉΡ‚ΠΈ ΠΈ Ρ‚ΠΎΡ‡Π½ΠΎ ΠΈΠ΄Π΅Π½Ρ‚ΠΈΡ„ΠΈΡ†ΠΈΡ€ΠΎΠ²Π°Ρ‚ΡŒ этот ΠΊΠΎΠ½Ρ‚Π΅Π½Ρ‚; Π½Π°ΠΏΡ€ΠΈΠΌΠ΅Ρ€ Π½Π°ΠΌ трСбуСтся Π° ссылка Π½Π° ΠΊΠΎΠ½ΠΊΡ€Π΅Ρ‚Π½Ρ‹ΠΉ вопрос (Π° Π½Π΅ Ρ‚ΠΎΠ»ΡŒΠΊΠΎ Π½Π° Π½Π°Π·Π²Π°Π½ΠΈΠ΅ вопроса), ΠΊΠΎΡ‚ΠΎΡ€Ρ‹ΠΉ содСрТит содСрТаниС ΠΈ описаниС ΠΊ ΠΊΠ°ΠΊΠΎΠΉ ΠΊΠΎΠ½ΠΊΡ€Π΅Ρ‚Π½ΠΎΠΉ части вопроса — ΠΈΠ·ΠΎΠ±Ρ€Π°ΠΆΠ΅Π½ΠΈΡŽ, ссылкС, тСксту ΠΈ Ρ‚. Π΄. — относится ваша ΠΆΠ°Π»ΠΎΠ±Π°; Π’Π°ΡˆΠ΅ имя, адрСс, Π½ΠΎΠΌΠ΅Ρ€ Ρ‚Π΅Π»Π΅Ρ„ΠΎΠ½Π° ΠΈ адрСс элСктронной ΠΏΠΎΡ‡Ρ‚Ρ‹; Π° Ρ‚Π°ΠΊΠΆΠ΅ Π’Π°ΡˆΠ΅ заявлСниС: (Π°) Π²Ρ‹ добросовСстно считаСтС, Ρ‡Ρ‚ΠΎ использованиС ΠΊΠΎΠ½Ρ‚Π΅Π½Ρ‚Π°, ΠΊΠΎΡ‚ΠΎΡ€Ρ‹ΠΉ, ΠΏΠΎ Π²Π°ΡˆΠ΅ΠΌΡƒ мнСнию, Π½Π°Ρ€ΡƒΡˆΠ°Π΅Ρ‚ ваши авторскиС ΠΏΡ€Π°Π²Π° Π½Π΅ Ρ€Π°Π·Ρ€Π΅ΡˆΠ΅Π½Ρ‹ Π·Π°ΠΊΠΎΠ½ΠΎΠΌ, Π²Π»Π°Π΄Π΅Π»ΡŒΡ†Π΅ΠΌ авторских ΠΏΡ€Π°Π² ΠΈΠ»ΠΈ Π΅Π³ΠΎ Π°Π³Π΅Π½Ρ‚ΠΎΠΌ; (Π±) Ρ‡Ρ‚ΠΎ всС информация, содСрТащаяся Π² вашСм Π£Π²Π΅Π΄ΠΎΠΌΠ»Π΅Π½ΠΈΠΈ ΠΎ Π½Π°Ρ€ΡƒΡˆΠ΅Π½ΠΈΠΈ, являСтся Ρ‚ΠΎΡ‡Π½ΠΎΠΉ, ΠΈ (c) ΠΏΠΎΠ΄ страхом наказания Π·Π° Π»ΠΆΠ΅ΡΠ²ΠΈΠ΄Π΅Ρ‚Π΅Π»ΡŒΡΡ‚Π²ΠΎ, Ρ‡Ρ‚ΠΎ Π²Ρ‹ Π»ΠΈΠ±ΠΎ Π²Π»Π°Π΄Π΅Π»Π΅Ρ† авторских ΠΏΡ€Π°Π², Π»ΠΈΠ±ΠΎ Π»ΠΈΡ†ΠΎ, ΡƒΠΏΠΎΠ»Π½ΠΎΠΌΠΎΡ‡Π΅Π½Π½ΠΎΠ΅ Π΄Π΅ΠΉΡΡ‚Π²ΠΎΠ²Π°Ρ‚ΡŒ ΠΎΡ‚ ΠΈΡ… ΠΈΠΌΠ΅Π½ΠΈ.

ΠžΡ‚ΠΏΡ€Π°Π²ΡŒΡ‚Π΅ ΠΆΠ°Π»ΠΎΠ±Ρƒ Π½Π°ΡˆΠ΅ΠΌΡƒ ΡƒΠΏΠΎΠ»Π½ΠΎΠΌΠΎΡ‡Π΅Π½Π½ΠΎΠΌΡƒ Π°Π³Π΅Π½Ρ‚Ρƒ ΠΏΠΎ адрСсу:

Π§Π°Ρ€Π»ΡŒΠ· Кон Varsity Tutors LLC
101 S. Hanley Rd, Suite 300
St. Louis, MO 63105

Или Π·Π°ΠΏΠΎΠ»Π½ΠΈΡ‚Π΅ Ρ„ΠΎΡ€ΠΌΡƒ Π½ΠΈΠΆΠ΅:

Π€Π°ΠΊΡ‚ΠΎΡ€Ρ‹, Π²Π»ΠΈΡΡŽΡ‰ΠΈΠ΅ Π½Π° сопротивлСниС ΠΏΡ€ΠΎΠ²ΠΎΠ΄Π°

Π‘ΠΎΠΏΡ€ΠΎΡ‚ΠΈΠ²Π»Π΅Π½ΠΈΠ΅ ΠΎΠ±ΡŠΠ΅ΠΊΡ‚ являСтся ΠΌΠ΅Ρ€ΠΎΠΉ Ρ‚ΠΎΠ³ΠΎ, насколько рСзистСнтный Ρ‚ΠΎΠΊ Π΄ΠΎΠ»ΠΆΠ΅Π½ Ρ‚Π΅Ρ‡ΡŒ Ρ‡Π΅Ρ€Π΅Π· этот ΠΎΠ±ΡŠΠ΅ΠΊΡ‚.Π•ΠΌΡƒ присвоСн символ R ΠΈ Π΅Π΄ΠΈΠ½ΠΈΡ†Π° измСрСния W (грСчСская Π±ΡƒΠΊΠ²Π° ΠΎΠΌΠ΅Π³Π° ΠΈ произносится ΠΊΠ°ΠΊ «ΠΎΠΌ»)

.

Волько Ρ‚ΠΎΠΊ Ρ‚Π΅Ρ‡Π΅Ρ‚ Ρ‡Π΅Ρ€Π΅Π· ΠΊΠΎΠΌΠΏΠΎΠ½Π΅Π½Ρ‚ Ρ†Π΅ΠΏΠΈ, Ссли Ρ€Π°Π·Π½ΠΎΡΡ‚ΡŒ ΠΏΠΎΡ‚Π΅Π½Ρ†ΠΈΠ°Π»ΠΎΠ² (напряТСниС) Ρ€Π°Π²Π½Π° ΠΏΠΎΠ»ΠΎΠΆΠΈΡ‚ΡŒ Ρ‡Π΅Ρ€Π΅Π· это. Π§Π΅ΠΌ большС Ρ€Π°Π·Π½ΠΎΡΡ‚ΡŒ ΠΏΠΎΡ‚Π΅Π½Ρ†ΠΈΠ°Π»ΠΎΠ² Π½Π° ΠΊΠΎΠ½Ρ†Π°Ρ…, Ρ‚Π΅ΠΌ большС большС Ρ‚Π΅ΠΊΡƒΡ‰ΠΈΠΉ ΠΏΠΎΡ‚ΠΎΠΊ. Π­Ρ‚ΠΎ ΠΏΠΎΡ‚ΠΎΠΌΡƒ, Ρ‡Ρ‚ΠΎ Π΅ΡΡ‚ΡŒ Π±ΠΎΠ»Π΅Π΅ крутая элСктричСская Π½Π°ΠΊΠ»ΠΎΠ½ Π½Π° мСстС, Ρ‡Ρ‚ΠΎΠ±Ρ‹ заряды скользили Π²Π½ΠΈΠ· … ΠΊΡ€ΡƒΡ‡Π΅ Π½Π°ΠΊΠ»ΠΎΠ½ — быстрСС Π³ΠΎΡ€ΠΊΠ°!

Π£Ρ€Π°Π²Π½Π΅Π½ΠΈΠ΅ Π½ΠΈΠΆΠ΅ ΠΈΡΠΏΠΎΠ»ΡŒΠ·ΡƒΠ΅Ρ‚ΡΡ для опрСдСлСния сопротивлСния ΠΊΠΎΠΌΠΏΠΎΠ½Π΅Π½Ρ‚Π° Π½Π° основС ΠΈΠ·ΠΌΠ΅Ρ€Π΅Π½ΠΈΠΉ Ρ‚ΠΎΠΊ, ΠΏΡ€ΠΎΡ‚Π΅ΠΊΠ°ΡŽΡ‰ΠΈΠΉ Ρ‡Π΅Ρ€Π΅Π· Π½Π΅Π³ΠΎ, ΠΈ Ρ€Π°Π·Π½ΠΎΡΡ‚ΡŒ ΠΏΠΎΡ‚Π΅Π½Ρ†ΠΈΠ°Π»ΠΎΠ² Π½Π° Π΅Π³ΠΎ заканчиваСтся.

Π‘Π°Π½ΠΊΠ° Π²Ρ‹ набрасываСтС ΠΏΡ€ΠΈΠ½Ρ†ΠΈΠΏΠΈΠ°Π»ΡŒΠ½ΡƒΡŽ схСму подходящСй схСмы, ΠΊΠΎΡ‚ΠΎΡ€ΡƒΡŽ Π²Ρ‹ ΠΌΠΎΠ³Π»ΠΈ Π±Ρ‹ ΠΈΡΠΏΠΎΠ»ΡŒΠ·ΠΎΠ²Π°Ρ‚ΡŒ, Ρ‡Ρ‚ΠΎΠ±Ρ‹ Π½Π°ΠΉΡ‚ΠΈ ΠΎΡ‚ΠΊΡƒΠ΄Π° такая информация?

Π’Ρ‹ Π΄ΠΎΠ»ΠΆΠ΅Π½ Π±Ρ‹Ρ‚ΡŒ способСн!

Π’ = I R

Π“Π΄Π΅ Π’ = Ρ€Π°Π·Π½ΠΎΡΡ‚ΡŒ ΠΏΠΎΡ‚Π΅Π½Ρ†ΠΈΠ°Π»ΠΎΠ² Π² Π²ΠΎΠ»ΡŒΡ‚Π°Ρ… (Π’)

Π― = Ρ‚ΠΎΠΊ Π² Π°ΠΌΠΏΠ΅Ρ€Π°Ρ… (A) ΠΈ

R = сопротивлСниС Π² Ом ()

Current — курс расхода заряда .Заряд, Ρ‚Π΅ΠΊΡƒΡ‰ΠΈΠΉ ΠΏΠΎ ΠΏΡ€ΠΎΠ²ΠΎΠ΄Ρƒ, пСрСносится элСктронами. ΠΊΠΎΡ‚ΠΎΡ€Ρ‹Π΅ Π²Ρ€Π°Ρ‰Π°ΡŽΡ‚ΡΡ Π²ΠΎΠΊΡ€ΡƒΠ³ Π°Ρ‚ΠΎΠΌΠΎΠ², ΡΠΎΡΡ‚Π°Π²Π»ΡΡŽΡ‰ΠΈΡ… ΠΏΡ€ΠΎΠ²ΠΎΠ»ΠΎΠΊΡƒ. Как Ρ‚ΠΎΠ»ΡŒΠΊΠΎ Ρ€Π°Π·Π½ΠΎΡΡ‚ΡŒ ΠΏΠΎΡ‚Π΅Π½Ρ†ΠΈΠ°Π»ΠΎΠ² элСктроны ΠΎΠ±Ρ‹Ρ‡Π½ΠΎ Π΄Ρ€Π΅ΠΉΡ„ΡƒΡŽΡ‚ Π² ΠΎΠ΄Π½ΠΎΠΌ Π½Π°ΠΏΡ€Π°Π²Π»Π΅Π½ΠΈΠΈ (Π²Ρ‹ ΠΌΠΎΠΆΠ΅Ρ‚Π΅ ΠΏΠΎΠ΄ΡƒΠΌΠ°Ρ‚ΡŒ ΠΊΠ°ΠΊ скатываниС ΠΏΠΎ склону). Π§Π΅ΠΌ большС p.d. Ρ‚Π΅ΠΌ быстрСС ΠΈΡ… срСдний ΡΠΊΠΎΡ€ΠΎΡΡ‚ΡŒ, ΠΈ Ρ‡Π΅ΠΌ большС заряд пСрСмСстится Π·Π° Ρ‚ΠΎΡ‡ΠΊΡƒ Π² сСкунду — большС Π’Π΅ΠΊΡƒΡ‰ΠΈΠΉ!

Π£Π²Π΅Π»ΠΈΡ‡Π΅Π½ΠΈΠ΅ числа элСктронов, ΠΈΡΠΏΡ‹Ρ‚Ρ‹Π²Π°ΡŽΡ‰ΠΈΡ… «Π½Π°ΠΊΠ»ΠΎΠ½», Π±ΡƒΠ΄Π΅Ρ‚ ΠΎΠ·Π½Π°Ρ‡Π°Ρ‚ΡŒ больший ΠΏΠΎΡ‚ΠΎΠΊ Ρ‡Π΅Ρ€Π΅Π· Ρ‚ΠΎΡ‡ΠΊΡƒ Π² сСкунда ΠΈ Ρ‚Π°ΠΊ большС заряда ΠΏΡ€ΠΎΡ…ΠΎΠ΄ΠΈΡ‚ Ρ‚ΠΎΡ‡ΠΊΡƒ Π·Π° сСкунду ΠΈ большС Ρ‚ΠΎΠΊΠ° ΠΏΠΎΡ‚ΠΎΠΊΠΈ.

ВсСго Ρ‡Π΅Ρ‚Ρ‹Ρ€Π΅ Ρ„Π°ΠΊΡ‚ΠΎΡ€Ρ‹, Π²Π»ΠΈΡΡŽΡ‰ΠΈΠ΅ Π½Π° сопротивлСниС ΠΏΡ€ΠΎΠ²ΠΎΠ΄Π°:

Π‘ΠΎΠΏΡ€ΠΎΡ‚ΠΈΠ²Π»Π΅Π½ΠΈΠ΅ ΠΏΡ€ΠΎΠΏΠΎΡ€Ρ†ΠΈΠΎΠ½Π°Π»ΡŒΠ½ΠΎ Π΄Π»ΠΈΠ½Π΅ . Если Π²Π·ΡΡ‚ΡŒ ΠΏΡ€ΠΎΠ²ΠΎΠ΄ Ρ€Π°Π·Π½ΠΎΠΉ Π΄Π»ΠΈΠ½Ρ‹ ΠΈ Π΄Π°Ρ‚ΡŒ ΠΊΠ°ΠΆΠ΄ΠΎΠΌΡƒ ΠΊΠΎΠ½ΠΊΡ€Π΅Ρ‚Π½ΡƒΡŽ Ρ€Π°Π·Π½ΠΎΡΡ‚ΡŒ ΠΏΠΎΡ‚Π΅Π½Ρ†ΠΈΠ°Π»ΠΎΠ² Π½Π° ΠΊΠΎΠ½Ρ†Π°Ρ…. Π’ Ρ‡Π΅ΠΌ Π΄Π»ΠΈΠ½Π½Π΅Π΅ ΠΏΡ€ΠΎΠ²ΠΎΠ΄, Ρ‚Π΅ΠΌ мСньшС Π²ΠΎΠ»ΡŒΡ‚ Π±ΡƒΠ΄Π΅Ρ‚ Π½Π° ΠΊΠ°ΠΆΠ΄Ρ‹ΠΉ Π΅Π³ΠΎ сантимСтр. Π­Ρ‚ΠΎΡ‚ ΠΎΠ·Π½Π°Ρ‡Π°Π΅Ρ‚, Ρ‡Ρ‚ΠΎ «ΡΠ»Π΅ΠΊΡ‚ричСская ΠΊΡ€ΡƒΡ‚ΠΈΠ·Π½Π°», Π·Π°ΡΡ‚Π°Π²Π»ΡΡŽΡ‰Π°Ρ элСктроны Π΄Π²ΠΈΠ³Π°Ρ‚ΡŒΡΡ, становится ΠΌΠ΅Π½Π΅Π΅ ΠΊΡ€ΡƒΡ‚ΠΎΠΉ ΠΏΠΎ ΠΌΠ΅Ρ€Π΅ удлинСния ΠΏΡ€ΠΎΠ²ΠΎΠ»ΠΎΠΊΠΈ, Π° срСдняя ΡΠΊΠΎΡ€ΠΎΡΡ‚ΡŒ Π΄Ρ€Π΅ΠΉΡ„Π° элСктронов ΡƒΠΌΠ΅Π½ΡŒΡˆΠ°Π΅Ρ‚ΡΡ.ΠŸΡ€Π°Π²ΠΈΠ»ΡŒΠ½Ρ‹ΠΉ Ρ‚Π΅Ρ€ΠΌΠΈΠ½ для этого «элСктричСского Π½Π°ΠΊΠ»ΠΎΠ½Π°Β» — Π³Ρ€Π°Π΄ΠΈΠ΅Π½Ρ‚ ΠΏΠΎΡ‚Π΅Π½Ρ†ΠΈΠ°Π»Π°. МСньший Π³Ρ€Π°Π΄ΠΈΠ΅Π½Ρ‚ ΠΏΠΎΡ‚Π΅Π½Ρ†ΠΈΠ°Π»Π° (мСньшС Π²ΠΎΠ»ΡŒΡ‚ Π½Π° ΠΌΠ΅Ρ‚Ρ€) ΠΎΠ·Π½Π°Ρ‡Π°Π΅Ρ‚, Ρ‡Ρ‚ΠΎ Ρ‚ΠΎΠΊ ΡƒΠΌΠ΅Π½ΡŒΡˆΠ°Π΅Ρ‚ΡΡ с ΡƒΠ²Π΅Π»ΠΈΡ‡Π΅Π½ΠΈΠ΅ΠΌ Π΄Π»ΠΈΠ½Ρ‹ ΠΈ сопротивлСния увСличиваСтся.

Π‘ΠΎΠΏΡ€ΠΎΡ‚ΠΈΠ²Π»Π΅Π½ΠΈΠ΅ ΠΎΠ±Ρ€Π°Ρ‚Π½ΠΎ ΠΏΡ€ΠΎΠΏΠΎΡ€Ρ†ΠΈΠΎΠ½Π°Π»ΡŒΠ½ΠΎ ΠΏΠ»ΠΎΡ‰Π°Π΄ΠΈ ΠΏΠΎΠΏΠ΅Ρ€Π΅Ρ‡Π½ΠΎΠ³ΠΎ сСчСния . Π§Π΅ΠΌ большС ΠΏΠ»ΠΎΡ‰Π°Π΄ΡŒ ΠΏΠΎΠΏΠ΅Ρ€Π΅Ρ‡Π½ΠΎΠ³ΠΎ сСчСния ΠΏΡ€ΠΎΠ²ΠΎΠ΄Π° Ρ‚Π΅ΠΌ большС количСство элСктронов ΠΊΠΎΡ‚ΠΎΡ€Ρ‹Π΅ ΠΈΡΠΏΡ‹Ρ‚Ρ‹Π²Π°ΡŽΡ‚ «элСктричСский Π½Π°ΠΊΠ»ΠΎΠ½Β» ΠΈΠ·-Π·Π° Ρ€Π°Π·Π½ΠΈΡ†Ρ‹ ΠΏΠΎΡ‚Π΅Π½Ρ†ΠΈΠ°Π»ΠΎΠ².ΠŸΠΎΡΠΊΠΎΠ»ΡŒΠΊΡƒ Π΄Π»ΠΈΠ½Π° ΠΏΡ€ΠΎΠ²ΠΎΠ»ΠΎΠΊΠΈ Π½Π΅ мСняСтся, ΠΊΠ°ΠΆΠ΄Ρ‹ΠΉ сантимСтр остаСтся ΠΏΡ€Π΅ΠΆΠ½ΠΈΠΌ. количСство Π²ΠΎΠ»ΡŒΡ‚ Π½Π° Π½Π΅ΠΌ — Π³Ρ€Π°Π΄ΠΈΠ΅Π½Ρ‚ ΠΏΠΎΡ‚Π΅Π½Ρ†ΠΈΠ°Π»Π° Π½Π΅ мСняСтся ΠΈ поэтому срСдняя ΡΠΊΠΎΡ€ΠΎΡΡ‚ΡŒ Π΄Ρ€Π΅ΠΉΡ„Π° ΠΎΡ‚Π΄Π΅Π»ΡŒΠ½Ρ‹Ρ… элСктронов Π½Π΅ измСняСтся. ΠΈΠ·ΠΌΠ΅Π½Π΅Π½ΠΈΠ΅. Π₯отя ΠΎΠ½ΠΈ Π½Π΅ Π΄Π²ΠΈΠ³Π°ΡŽΡ‚ΡΡ быстрСС, ΠΈΡ… большС двиТСтся, поэтому ΠΎΠ±Ρ‰Π΅Π΅ Π΄Π²ΠΈΠΆΠ΅Π½ΠΈΠ΅ заряда Π·Π° Π΄Π°Π½Π½Ρ‹ΠΉ ΠΌΠΎΠΌΠ΅Π½Ρ‚ Π²Ρ€Π΅ΠΌΠ΅Π½ΠΈ большС ΠΈ Ρ‚Π΅ΠΊΡƒΡ‰ΠΈΠΉ ΠΏΠΎΡ‚ΠΎΠΊ увСличиваСтся. Π­Ρ‚ΠΎ ΠΎΠ·Π½Π°Ρ‡Π°Π΅Ρ‚, Ρ‡Ρ‚ΠΎ сопротивлСниС ΡƒΠΌΠ΅Π½ΡŒΡˆΠ°Π΅Ρ‚ΡΡ. Π­Ρ‚ΠΎ Π΄Π΅Π»Π°Π΅Ρ‚ Π½Π΅ приводят ΠΊ прямолинСйному Π³Ρ€Π°Ρ„ΠΈΠΊΡƒ, Ρ‚Π°ΠΊ ΠΊΠ°ΠΊ ΠΏΠ»ΠΎΡ‰Π°Π΄ΡŒ ΠΏΠΎΠΏΠ΅Ρ€Π΅Ρ‡Π½ΠΎΠ³ΠΎ сСчСния ΠΎΠ±Ρ€Π°Ρ‚Π½ΠΎ ΠΏΡ€ΠΎΠΏΠΎΡ€Ρ†ΠΈΠΎΠ½Π°Π»ΡŒΠ½ΠΎ ΡΠΎΠΏΡ€ΠΎΡ‚ΠΈΠ²Π»Π΅Π½ΠΈΡŽ Π½Π΅ прямо ΠΏΡ€ΠΎΠΏΠΎΡ€Ρ†ΠΈΠΎΠ½Π°Π»ΡŒΠ½ΠΎ Π­Ρ‚ΠΎ.

Π€ΠΈΠ·ΠΈΠΊΠΈ Π»ΡŽΠ±ΡΡ‚ ΡΡ‚Ρ€ΠΎΠΈΡ‚ΡŒ ΠΎΡ‚Π½ΠΎΡˆΠ΅Π½ΠΈΡ ΠΏΠΎ прямой Π»ΠΈΠ½ΠΈΠΈ, Ссли ΠΌΠΎΠ³ΡƒΡ‚ …. Ρ‚Ρ‹ моТСшь ΠΏΠΎΠ΄ΡƒΠΌΠ°Ρ‚ΡŒ способа ΠΏΠΎΠ»ΡƒΡ‡ΠΈΡ‚ΡŒ прямой Π³Ρ€Π°Ρ„ΠΈΠΊ Ρ‡Π΅Ρ€Π΅Π· Π½Π°Ρ‡Π°Π»ΠΎ ΠΊΠΎΠΎΡ€Π΄ΠΈΠ½Π°Ρ‚? Π§Ρ‚ΠΎ Π±Ρ‹ Ρƒ вас Π΅ΡΡ‚ΡŒ ΡΡŽΠΆΠ΅Ρ‚?

Π‘ΠΎΠΏΡ€ΠΎΡ‚ΠΈΠ²Π»Π΅Π½ΠΈΠ΅ зависит ΠΏΠΎ ΠΌΠ°Ρ‚Π΅Ρ€ΠΈΠ°Π»Ρƒ ΠΏΡ€ΠΎΠ²ΠΎΠ»ΠΎΠΊΠ° . Π’Π΅ΠΌ ΠΏΠ»ΠΎΡ‚Π½Π΅Π΅ Π°Ρ‚ΠΎΠΌ ΡƒΠ΄Π΅Ρ€ΠΆΠΈΠ²Π°Π΅Ρ‚ свои внСшниС элСктроны, Ρ‚Π΅ΠΌ Ρ‚Ρ€ΡƒΠ΄Π½Π΅Π΅ Π±ΡƒΠ΄Π΅Ρ‚ ΡΠ΄Π΅Π»Π°Ρ‚ΡŒ Ρ‚Π΅ΠΊΡƒΡ‰ΠΈΠΉ ΠΏΠΎΡ‚ΠΎΠΊ.ЭлСктронная конфигурация Π°Ρ‚ΠΎΠΌΠ° опрСдСляСт насколько Π°Ρ‚ΠΎΠΌ Π±ΡƒΠ΄Π΅Ρ‚ Π³ΠΎΡ‚ΠΎΠ² ΠΏΠΎΠ·Π²ΠΎΠ»ΠΈΡ‚ΡŒ элСктрону ΡƒΠΉΡ‚ΠΈ ΠΈ Π±Π»ΡƒΠΆΠ΄Π°Ρ‚ΡŒ Ρ‡Π΅Ρ€Π΅Π· Ρ€Π΅ΡˆΠ΅Ρ‚ΠΊΡƒ. Если ΠΎΠ±ΠΎΠ»ΠΎΡ‡ΠΊΠ° ΠΏΠΎΡ‡Ρ‚ΠΈ Π·Π°ΠΏΠΎΠ»Π½Π΅Π½Π°, Π°Ρ‚ΠΎΠΌ Π½Π΅ΠΎΡ…ΠΎΡ‚Π½ΠΎ ΠΏΠΎΠ·Π²ΠΎΠ»ΡΡ‚ΡŒ Π΅Π³ΠΎ элСктронам Π±Π»ΡƒΠΆΠ΄Π°Ρ‚ΡŒ, Π° ΠΌΠ°Ρ‚Π΅Ρ€ΠΈΠ°Π», Π² ΠΊΠΎΡ‚ΠΎΡ€ΠΎΠΌ ΠΎΠ½ находится, являСтся изолятором. Если внСшняя ΠΎΠ±ΠΎΠ»ΠΎΡ‡ΠΊΠ° (ΠΈΠ»ΠΈ ΠΏΠΎΠ΄ΠΎΠ±ΠΎΠ»ΠΎΡ‡ΠΊΠ° с ΠΏΠ΅Ρ€Π΅Ρ…ΠΎΠ΄Π½Ρ‹ΠΌΠΈ ΠΌΠ΅Ρ‚Π°Π»Π»Π°ΠΌΠΈ) мСньшС Ρ‡Π΅ΠΌ Π½Π°ΠΏΠΎΠ»ΠΎΠ²ΠΈΠ½Ρƒ Π·Π°ΠΏΠΎΠ»Π½Π΅Π½, Ρ‚ΠΎΠ³Π΄Π° Π°Ρ‚ΠΎΠΌ Π³ΠΎΡ‚ΠΎΠ² ΠΏΠΎΠ·Π²ΠΎΠ»ΠΈΡ‚ΡŒ элСктронам Π±Π»ΡƒΠΆΠ΄Π°Ρ‚ΡŒ Π° ΠΌΠ°Ρ‚Π΅Ρ€ΠΈΠ°Π» — ΠΏΡ€ΠΎΠ²ΠΎΠ΄Π½ΠΈΠΊ.

А Π³Ρ€Π°Ρ„ΠΈΠΊ для этого Π±ΡƒΠ΄Π΅Ρ‚ гистограммой, Π° Π½Π΅ Π»ΠΈΠ½Π΅ΠΉΠ½Ρ‹ΠΌ Π³Ρ€Π°Ρ„ΠΈΠΊΠΎΠΌ.

Π‘ΠΎΠΏΡ€ΠΎΡ‚ΠΈΠ²Π»Π΅Π½ΠΈΠ΅ увСличиваСтся с Ρ‚Π΅ΠΌΠΏΠ΅Ρ€Π°Ρ‚ΡƒΡ€ΠΎΠΉ ΠΏΡ€ΠΎΠ²ΠΎΠ»ΠΎΠΊΠΈ 90–250. Π‘ΠΎΠ»Π΅Π΅ горячий ΠΏΡ€ΠΎΠ²ΠΎΠ΄ ΠΈΠΌΠ΅Π΅Ρ‚ большСС сопротивлСниС ΠΈΠ·-Π·Π° ΠΏΠΎΠ²Ρ‹ΡˆΠ΅Π½Π½ΠΎΠΉ Π²ΠΈΠ±Ρ€Π°Ρ†ΠΈΠΈ Π°Ρ‚ΠΎΠΌΠ½ΠΎΠΉ Ρ€Π΅ΡˆΠ΅Ρ‚ΠΊΠΈ. Когда ΠΌΠ°Ρ‚Π΅Ρ€ΠΈΠ°Π» нагрСваСтся, Π°Ρ‚ΠΎΠΌΡ‹ Π² Ρ€Π΅ΡˆΠ΅Ρ‚ΠΊΠ΅ Π²ΠΈΠ±Ρ€ΠΈΡ€ΡƒΡŽΡ‚ сильнСС. Π­Ρ‚ΠΎ затрудняСт Π΄Π²ΠΈΠΆΠ΅Π½ΠΈΠ΅ элСктронов Π±Π΅Π· взаимодСйствия. с Π°Ρ‚ΠΎΠΌΠΎΠΌ ΠΈ ΡƒΠ²Π΅Π»ΠΈΡ‡ΠΈΠ²Π°Π΅Ρ‚ сопротивлСниС. Бвязь ΠΌΠ΅ΠΆΠ΄Ρƒ сопротивлСниСм ΠΈ Ρ‚Π΅ΠΌΠΏΠ΅Ρ€Π°Ρ‚ΡƒΡ€Π° Π½Π΅ ΠΈΠ· простых.

((Π°Π»ΡŒΡ„Π°) коэффициСнт тСрмичСского сопротивлСния)

——————————————-

На ΡƒΡ€ΠΎΠ²Π½Π΅ A ΠΌΡ‹ Π±ΡƒΠ΄Π΅ΠΌ помСститС эти уравнСния Π² ΡƒΡ€Π°Π²Π½Π΅Π½ΠΈΠ΅. ΠŸΡ€ΠΎΠ²Π΅Ρ€ΡΡ‚ΡŒ ΠΏΡ€ΠΎΡ‡Ρ‚ΠΈ страницу ΠΈ Ρ‡ΠΈΡ‚Π°ΠΉ дальшС …..

Π—Π°ΠΊΠΎΠ½ Ома

ΠœΡ‹ рассматриваСм Ρ„ΡƒΠ½Π΄Π°ΠΌΠ΅Π½Ρ‚Π°Π»ΡŒΠ½ΡƒΡŽ связь Π² элСктроникС ΠΈ Ρ„ΠΈΠ·ΠΈΠΊΠ΅.

Π—Π°ΠΊΠΎΠ½ Ома Π±Ρ‹Π» ΠΎΡ‚ΠΊΡ€Ρ‹Ρ‚ Π“Π΅ΠΎΡ€Π³ΠΎΠΌ Омом Π² 1837 Π³ΠΎΠ΄Ρƒ, ΠΈ это основноС ΡƒΡ€Π°Π²Π½Π΅Π½ΠΈΠ΅, ΠΊΠΎΡ‚ΠΎΡ€ΠΎΠ΅ управляСт ΠΌΠ½ΠΎΠ³ΠΈΠΌΠΈ схСмами.Π’Ρ€ΠΈ основных ΠΈΠ½Π³Ρ€Π΅Π΄ΠΈΠ΅Π½Ρ‚Π° — это Ρ‚ΠΎΠΊ Ρ‡Π΅Ρ€Π΅Π· ΠΏΡ€ΠΎΡΡ‚ΡƒΡŽ Ρ†Π΅ΠΏΡŒ, ΠΏΡ€ΠΈΠ»ΠΎΠΆΠ΅Π½Π½ΠΎΠ΅ напряТСниС (ΠΎΠ±Ρ‹Ρ‡Π½ΠΎ ΠΎΡ‚ Π±Π°Ρ‚Π°Ρ€Π΅ΠΈ) ΠΈ сопротивлСниС устройства, ΠΊΠΎΡ‚ΠΎΡ€ΠΎΠ΅ ΠΈΡΠΏΠΎΠ»ΡŒΠ·ΡƒΠ΅Ρ‚ Ρ‚ΠΎΠΊ для выполнСния Π½Π΅ΠΊΠΎΡ‚ΠΎΡ€ΠΎΠΉ Ρ€Π°Π±ΠΎΡ‚Ρ‹, ΠΎΠ±Ρ‹Ρ‡Π½ΠΎ Ρ‚Π΅ΠΏΠ»Π° ΠΈΠ»ΠΈ свСта. На этом этапС Π²Ρ‹ ΡƒΠ·Π½Π°Π΅Ρ‚Π΅ ΠΎ
  • матСматичСская Ρ„ΠΎΡ€ΠΌΡƒΠ»ΠΈΡ€ΠΎΠ²ΠΊΠ° Π·Π°ΠΊΠΎΠ½Π° Ома ΠΈ основная обратная Π·Π°Π²ΠΈΡΠΈΠΌΠΎΡΡ‚ΡŒ, ΠΊΠΎΡ‚ΠΎΡ€ΡƒΡŽ ΠΎΠ½ ΠΊΠΎΠ΄ΠΈΡ€ΡƒΠ΅Ρ‚
  • ΠΊΠ°ΠΊ аналогия с Π²ΠΎΠ΄ΠΎΠΏΡ€ΠΎΠ²ΠΎΠ΄Π½ΠΎΠΉ Ρ‚Ρ€ΡƒΠ±ΠΎΠΉ ΠΌΠΎΠΆΠ΅Ρ‚ ΠΏΠΎΠΌΠΎΡ‡ΡŒ ΠΏΠΎΠ½ΡΡ‚ΡŒ Π·Π½Π°Ρ‡Π΅Π½ΠΈΠ΅ Π·Π°ΠΊΠΎΠ½Π° Ома.

Π—Π°ΠΊΠΎΠ½ Ома

Π—Π°ΠΊΠΎΠ½ Ома гласит: if (normalsize {V}) — это напряТСниС (ΠΈΠ·ΠΌΠ΅Ρ€Π΅Π½Π½ΠΎΠ΅ Π² Π²ΠΎΠ»ΡŒΡ‚Π°Ρ…) Π½Π° рСзисторС (normalsize {R}) (ΠΈΠ·ΠΌΠ΅Ρ€Π΅Π½Π½ΠΎΠ΅ Π² ΠΎΠΌΠ°Ρ…), ΠΊΠΎΡ‚ΠΎΡ€ΠΎΠ΅ потрСбляСт Ρ‚ΠΎΠΊ (normalsize {I}) (ΠΈΠ·ΠΌΠ΅Ρ€Π΅Π½Π½Ρ‹ΠΉ Π² Π°ΠΌΠΏΠ΅Ρ€Π°Ρ…), Ρ‚ΠΎ [Π‘ΠΎΠ»ΡŒΡˆΠΎΠΉ {V = IR}.] РСзистор — это ΠΎΠ±ΡŠΠ΅ΠΊΡ‚, ΠΊΠΎΡ‚ΠΎΡ€Ρ‹ΠΉ ΠΈΡΠΏΠΎΠ»ΡŒΠ·ΡƒΠ΅Ρ‚ ΡΠ»Π΅ΠΊΡ‚Ρ€ΠΈΡ‡Π΅ΡΠΊΡƒΡŽ ΡΠ½Π΅Ρ€Π³ΠΈΡŽ ΠΈ ΠΏΡ€Π΅ΠΎΠ±Ρ€Π°Π·ΡƒΠ΅Ρ‚ Π΅Π΅ Π²ΠΎ Ρ‡Ρ‚ΠΎ-Ρ‚ΠΎ Π΅Ρ‰Π΅, Π½Π°ΠΏΡ€ΠΈΠΌΠ΅Ρ€, Ρ‚Π΅ΠΏΠ»ΠΎ ΠΈΠ»ΠΈ свСт. ΠŸΡ€ΠΈΠΌΠ΅Ρ€ΠΎΠΌ ΠΌΠΎΠΆΠ΅Ρ‚ ΡΠ»ΡƒΠΆΠΈΡ‚ΡŒ тостСр. ЭлСктроэнСргия, ΠΏΡ€ΠΎΡ‚Π΅ΠΊΠ°ΡŽΡ‰Π°Ρ Ρ‡Π΅Ρ€Π΅Π· тостСр, питаСтся ΠΎΡ‚ ΠΏΠ΅Ρ€Π΅ΠΏΠ°Π΄Π° напряТСния, ΠΏΠΎΠ΄Π°Π²Π°Π΅ΠΌΠΎΠ³ΠΎ Ρ‡Π΅Ρ€Π΅Π· ΡΠ»Π΅ΠΊΡ‚Ρ€ΠΈΡ‡Π΅ΡΠΊΡƒΡŽ Ρ€ΠΎΠ·Π΅Ρ‚ΠΊΡƒ. Π§Π΅ΠΌ Π²Ρ‹ΡˆΠ΅ напряТСниС, Ρ‚Π΅ΠΌ большС Ρ‚ΠΎΠΊΠ° (Π½ΠΎΡ€ΠΌΠ°Π»ΡŒΠ½ΠΎΠ³ΠΎ Ρ€Π°Π·ΠΌΠ΅Ρ€Π° {I}) ΠΏΡ€ΠΎΡ…ΠΎΠ΄ΠΈΡ‚ Ρ‡Π΅Ρ€Π΅Π· тостСр. Π˜Ρ‚Π°ΠΊ, для фиксированного рСзистора (Π½ΠΎΡ€ΠΌΠ°Π»ΡŒΠ½Ρ‹ΠΉ Ρ€Π°Π·ΠΌΠ΅Ρ€ {R}) Π·Π°ΠΊΠΎΠ½ Ома устанавливаСт Π»ΠΈΠ½Π΅ΠΉΠ½ΡƒΡŽ ΠΏΡ€ΠΎΠΏΠΎΡ€Ρ†ΠΈΠΎΠ½Π°Π»ΡŒΠ½ΠΎΡΡ‚ΡŒ ΠΌΠ΅ΠΆΠ΄Ρƒ напряТСниСм ΠΈ Ρ‚ΠΎΠΊΠΎΠΌ. Нити тостСра Ник ΠšΠ°Ρ€ΡΠΎΠ½ Π½Π° en.wikipedia CC BY 3.0, Ρ‡Π΅Ρ€Π΅Π· Wikimedia Commons Однако ΠΌΡ‹ ΠΌΠΎΠΆΠ΅ΠΌ Π²Π·Π³Π»ΡΠ½ΡƒΡ‚ΡŒ Π½Π° Π·Π°ΠΊΠΎΠ½ ΠΈ ΠΏΠΎ-Π΄Ρ€ΡƒΠ³ΠΎΠΌΡƒ. Если ΠΌΡ‹ рассматриваСм напряТСниС (Π½ΠΎΡ€ΠΌΠ°Π»ΡŒΠ½Ρ‹ΠΉ Ρ€Π°Π·ΠΌΠ΅Ρ€ {V}) ΠΊΠ°ΠΊ фиксированноС, Ρ‚ΠΎ сопротивлСниС ΠΈ Ρ‚ΠΎΠΊ ΠΎΠ±Ρ€Π°Ρ‚Π½ΠΎ ΠΏΡ€ΠΎΠΏΠΎΡ€Ρ†ΠΈΠΎΠ½Π°Π»ΡŒΠ½Ρ‹, ΠΏΠΎΡΠΊΠΎΠ»ΡŒΠΊΡƒ ΠΈΡ… ΠΏΡ€ΠΎΠΈΠ·Π²Π΅Π΄Π΅Π½ΠΈΠ΅ постоянно ΠΈ Ρ€Π°Π²Π½ΠΎ фиксированному Π½Π°ΠΏΡ€ΡΠΆΠ΅Π½ΠΈΡŽ. Если ΠΌΡ‹ ΡƒΠ²Π΅Π»ΠΈΡ‡ΠΈΠ²Π°Π΅ΠΌ сопротивлСниС, Ρ‚ΠΎ Ρ‚ΠΎΠΊ ΡƒΠΌΠ΅Π½ΡŒΡˆΠ°Π΅Ρ‚ΡΡ, Π° Ссли ΠΌΡ‹ ΡƒΠΌΠ΅Π½ΡŒΡˆΠ°Π΅ΠΌ сопротивлСниС, Ρ‚ΠΎ Ρ‚ΠΎΠΊ увСличиваСтся. Π­Ρ‚ΠΎ ситуация с Ρ†Π΅ΠΏΡŒΡŽ, Ρ€Π°Π±ΠΎΡ‚Π°ΡŽΡ‰Π΅ΠΉ ΠΎΡ‚ Π±Π°Ρ‚Π°Ρ€Π΅ΠΈ, ΠΈΠ»ΠΈ с элСктричСством Π² нашСм Π΄ΠΎΠΌΠ΅, Π³Π΄Π΅ ΠΏΠΎΠ΄Π°Π²Π°Π΅ΠΌΠΎΠ΅ напряТСниС являСтся постоянным ((Π½ΠΎΡ€ΠΌΠ°Π»ΡŒΠ½Ρ‹ΠΉ Ρ€Π°Π·ΠΌΠ΅Ρ€ {110-120}) Π²ΠΎΠ»ΡŒΡ‚ Π² Π±ΠΎΠ»ΡŒΡˆΠΈΠ½ΡΡ‚Π²Π΅ стран АмСрики (Π½ΠΎΡ€ΠΌΠ°Π»ΡŒΠ½Ρ‹ΠΉ Ρ€Π°Π·ΠΌΠ΅Ρ€ {220-230}) Π²ΠΎΠ»ΡŒΡ‚ Π² Π•Π²Ρ€ΠΎΠΏΠ΅, Австралии ΠΈ Π±ΠΎΠ»ΡŒΡˆΠΈΠ½ΡΡ‚Π²Π΅ стран Азии).Однако, строго говоря, Π² этом случаС напряТСниС мСняСтся ΠΏΠΎ Π½Π°ΠΏΡ€Π°Π²Π»Π΅Π½ΠΈΡŽ. Π’ ΠΏΡ€Π΅Π΄Π΅Π»ΡŒΠ½ΠΎΠΌ случаС, ΠΊΠΎΠ³Π΄Π° сопротивлСниС становится Ρ€Π°Π²Π½Ρ‹ΠΌ Π½ΡƒΠ»ΡŽ, Π½Π°ΠΏΡ€ΠΈΠΌΠ΅Ρ€, Ссли Π²Ρ‹ замСняСтС тостСр Π½Π° ΠΏΡ€ΠΎΠ²ΠΎΠ΄, Ρ‚ΠΎ Ρ‚Π΅Ρ‡Π΅Ρ‚ бСсконСчно большой Ρ‚ΠΎΠΊ. Π—Π°Ρ‚Π΅ΠΌ Π²ΠΎΠ·Π½ΠΈΠΊΠ°Π΅Ρ‚ ΠΊΠΎΡ€ΠΎΡ‚ΠΊΠΎΠ΅ Π·Π°ΠΌΡ‹ΠΊΠ°Π½ΠΈΠ΅ , часто с катастрофичСскими послСдствиями, особСнно Ссли Ρƒ вас Π½Π΅Ρ‚ ΠΏΠ»Π°Π²ΠΊΠΎΠ³ΠΎ прСдохранитСля, ΠΊΠΎΡ‚ΠΎΡ€Ρ‹ΠΉ Ρ€Π°Π·Ρ€Ρ‹Π²Π°Π΅Ρ‚ Ρ†Π΅ΠΏΡŒ Π² Ρ‚Π°ΠΊΠΎΠΉ Π°Π²Π°Ρ€ΠΈΠΉΠ½ΠΎΠΉ ситуации.

НСкоторыС ΠΏΡ€ΠΈΠΌΠ΅Ρ€Ρ‹

Если ΠΌΡ‹ ΠΏΠΎΠ΄ΠΊΠ»ΡŽΡ‡ΠΈΠΌ Π»Π°ΠΌΠΏΡƒ ΠΊ Ρ†Π΅ΠΏΠΈ, Ρ€Π°Π±ΠΎΡ‚Π°ΡŽΡ‰Π΅ΠΉ ΠΎΡ‚ Π±Π°Ρ‚Π°Ρ€Π΅ΠΈ Π½Π° 6 Π’, ΠΈ потрСбляСм Ρ‚ΠΎΠΊ 3 А, Ρ‚ΠΎΠ³Π΄Π° сопротивлСниС (Π½ΠΎΡ€ΠΌΠ°Π»ΡŒΠ½Ρ‹ΠΉ Ρ€Π°Π·ΠΌΠ΅Ρ€ R) Π±ΡƒΠ΄Π΅Ρ‚ Ρ€Π°Π²Π½ΠΎ [Large R = frac {V} {I} = frac 63 = 2; text {ohms}.] Π’Π΅ΠΏΠ΅Ρ€ΡŒ, Ссли ΠΌΡ‹ ΠΏΠΎΠ΄ΠΊΠ»ΡŽΡ‡ΠΈΠΌ Ρ‚Ρƒ ΠΆΠ΅ Π»Π°ΠΌΠΏΡƒ ΠΊ 10-Π²ΠΎΠ»ΡŒΡ‚ΠΎΠ²ΠΎΠΉ Π±Π°Ρ‚Π°Ρ€Π΅Π΅, Ρ‚ΠΎ Ρ‚ΠΎΠΊ (Π½ΠΎΡ€ΠΌΠ°Π»ΡŒΠ½Ρ‹ΠΉ Ρ€Π°Π·ΠΌΠ΅Ρ€ I) Π±ΡƒΠ΄Π΅Ρ‚ [Π‘ΠΎΠ»ΡŒΡˆΠΎΠΉ I = Π³ΠΈΠ΄Ρ€ΠΎΡ€Π°Π·Ρ€Ρ‹Π² {V} {R} = Π³ΠΈΠ΄Ρ€ΠΎΡ€Π°Π·Ρ€Ρ‹Π² {10} 2 = 5; тСкст {Π°ΠΌΠΏΠ΅Ρ€Ρ‹}.] Если ΠΌΡ‹ Ρ…ΠΎΡ‚ΠΈΠΌ ΡΠ΄Π΅Π»Π°Ρ‚ΡŒ свСт ярчС, Π½Π°ΠΌ Π½ΡƒΠΆΠ½ΠΎ ΡƒΠ²Π΅Π»ΠΈΡ‡ΠΈΡ‚ΡŒ Ρ‚ΠΎΠΊ, скаТСм, Π΄ΠΎ 8 Π°ΠΌΠΏΠ΅Ρ€, Ρ‚ΠΎΠ³Π΄Π° Π½Π°ΠΌ Π½ΡƒΠΆΠ½ΠΎ ΡƒΠ²Π΅Π»ΠΈΡ‡ΠΈΡ‚ΡŒ нашС напряТСниС Π΄ΠΎ [Π‘ΠΎΠ»ΡŒΡˆΠΎΠΉ V = IR = 8 Γ— 2 = 16; тСкст {Π²ΠΎΠ»ΡŒΡ‚}.]

Q1 (E): элСктричСскоС устройство ΠΏΠΎΠ΄ΠΊΠ»ΡŽΡ‡Π΅Π½ΠΎ ΠΊ Π½Π°ΠΏΡ€ΡΠΆΠ΅Π½ΠΈΡŽ 120 Π²ΠΎΠ»ΡŒΡ‚. НайдитС Ρ‚ΠΎΠΊ, Ссли сопротивлСниС 480 Ом.

Q2 (E): ΠŸΡ€Π΅Π΄ΠΏΠΎΠ»ΠΎΠΆΠΈΠΌ, Ρ‡Ρ‚ΠΎ Ρƒ нас Π΅ΡΡ‚ΡŒ батарСя с Π½Π΅ΠΊΠΎΡ‚ΠΎΡ€Ρ‹ΠΌ постоянным напряТСниСм, ΠΎΡΠ²Π΅Ρ‰Π°ΡŽΡ‰Π°Ρ Π½Π΅Π±ΠΎΠ»ΡŒΡˆΡƒΡŽ Π»Π°ΠΌΠΏΡƒ, ΠΈ Π°ΠΌΠΏΠ΅Ρ€ΠΌΠ΅Ρ‚Ρ€ ΠΏΠΎΠΊΠ°Π·Ρ‹Π²Π°Π΅Ρ‚ 40 мА, Π³Π΄Π΅ мА ΠΎΠ·Π½Π°Ρ‡Π°Π΅Ρ‚ ΠΌΠΈΠ»Π»ΠΈΠ°ΠΌΠΏΠ΅Ρ€, Ρ‡Ρ‚ΠΎ составляСт ΠΎΠ΄Π½Ρƒ Ρ‚Ρ‹ΡΡΡ‡Π½ΡƒΡŽ Ρ‡Π°ΡΡ‚ΡŒ Π°ΠΌΠΏΠ΅Ρ€.Если Ρ‚ΠΎΠΊ ΡƒΠΏΠ°Π» Π΄ΠΎ 20 мА, Ρ‡Ρ‚ΠΎ ΡΠ»ΡƒΡ‡ΠΈΠ»ΠΎΡΡŒ с сопротивлСниСм?

Как рСзистор сопротивляСтся?

РСзистор — это любоС устройство, Π·Π°ΠΌΠ΅Π΄Π»ΡΡŽΡ‰Π΅Π΅ ΠΏΡ€ΠΎΡ…ΠΎΠΆΠ΄Π΅Π½ΠΈΠ΅ Ρ‚ΠΎΠΊΠ° Π² Ρ†Π΅ΠΏΠΈ. ЭлСктричСство, ΠΏΠΎ сути, ΠΏΠ΅Ρ€Π΅ΠΌΠ΅Ρ‰Π°Π΅Ρ‚ элСктроны, ΠΈ, ΠΊΠ°ΠΊ ΠΈ Π²ΠΎΠ΄Π°, Ссли ΠΏΠΎΡ‚ΠΎΠΊ прСрываСтся, ограничиваСтся ΠΈΠ»ΠΈ сопротивляСтся , ΠΏΡ€ΠΎΡ…ΠΎΠ΄ΠΈΡ‚ мСньшС. НСкоторыС ΠΌΠ°Ρ‚Π΅Ρ€ΠΈΠ°Π»Ρ‹ ΠΈΠΌΠ΅ΡŽΡ‚ Π½ΠΈΠ·ΠΊΠΎΠ΅ сопротивлСниС, Π½Π°ΠΏΡ€ΠΈΠΌΠ΅Ρ€ мСдная ΠΏΡ€ΠΎΠ²ΠΎΠ»ΠΎΠΊΠ°, Ρ‡Ρ‚ΠΎ позволяСт элСктронам ΠΏΡ€ΠΎΡ…ΠΎΠ΄ΠΈΡ‚ΡŒ Ρ‡Π΅Ρ€Π΅Π· Π½ΠΈΡ… Π±Π΅Π· особых ΠΏΡ€ΠΎΠ±Π»Π΅ΠΌ. Π”Ρ€ΡƒΠ³ΠΈΠ΅ ΠΌΠ°Ρ‚Π΅Ρ€ΠΈΠ°Π»Ρ‹, Ρ‚Π°ΠΊΠΈΠ΅ ΠΊΠ°ΠΊ Π΄Π΅Ρ€Π΅Π²ΠΎ, ΠΎΠ±Π»Π°Π΄Π°ΡŽΡ‚ высоким сопротивлСниСм, ΠΏΠΎΡ‡Ρ‚ΠΈ ΠΌΠ³Π½ΠΎΠ²Π΅Π½Π½ΠΎ останавливая элСктричСский Ρ‚ΠΎΠΊ.На ΠΏΡ€Π°ΠΊΡ‚ΠΈΠΊΠ΅ Ρƒ нас Π΅ΡΡ‚ΡŒ Ρ‚Π°ΠΊΠΈΠ΅ Π²Π΅Ρ‰ΠΈ, ΠΊΠ°ΠΊ Π»Π°ΠΌΠΏΡ‹ ΠΈ тостСры, ΠΊΠΎΡ‚ΠΎΡ€Ρ‹Π΅ Π³Π΅Π½Π΅Ρ€ΠΈΡ€ΡƒΡŽΡ‚ свСт ΠΈΠ»ΠΈ Ρ‚Π΅ΠΏΠ»ΠΎ ΠΎΡ‚ элСктронов, замСдляя ΠΈΡ…, Π½ΠΎ всС ΠΆΠ΅ пропуская.

Ом Ρ‚Π°ΠΊΠΆΠ΅ ΠΎΠ±Π½Π°Ρ€ΡƒΠΆΠΈΠ» Π΄Ρ€ΡƒΠ³ΠΎΠΉ Π·Π°ΠΊΠΎΠ½, ΠΊΠΎΡ‚ΠΎΡ€Ρ‹ΠΉ описываСт, ΠΊΠ°ΠΊΠΎΠ΅ сопротивлСниС ΠΈΠΌΠ΅Π΅Ρ‚ Π΄Π°Π½Π½Ρ‹ΠΉ ΠΌΠ°Ρ‚Π΅Ρ€ΠΈΠ°Π», Π½Π°ΠΏΡ€ΠΈΠΌΠ΅Ρ€ кусок ΠΏΡ€ΠΎΠ²ΠΎΠ»ΠΎΡ‡Π½ΠΎΠΉ Ρ‚Ρ€ΡƒΠ±ΠΊΠΈ:

[Large R = frac {rho L} {A}]

Π³Π΄Π΅ (Π½ΠΎΡ€ΠΌΠ°Π»ΡŒΠ½Ρ‹ΠΉ Ρ€Π°Π·ΠΌΠ΅Ρ€ L) — Π΄Π»ΠΈΠ½Π° рСзистора, (Π½ΠΎΡ€ΠΌΠ°Π»ΡŒΠ½Ρ‹ΠΉ Ρ€Π°Π·ΠΌΠ΅Ρ€ rho) — Π²Π΅Π»ΠΈΡ‡ΠΈΠ½Π°, которая зависит ΠΎΡ‚ ΠΌΠ°Ρ‚Π΅Ρ€ΠΈΠ°Π»Π°, Π° (Π½ΠΎΡ€ΠΌΠ°Π»ΡŒΠ½Ρ‹ΠΉ Ρ€Π°Π·ΠΌΠ΅Ρ€ A) — ΠΏΠ»ΠΎΡ‰Π°Π΄ΡŒ ΠΏΠΎΠΏΠ΅Ρ€Π΅Ρ‡Π½ΠΎΠ³ΠΎ сСчСния рСзистора.Π˜Ρ‚Π°ΠΊ, (Π½ΠΎΡ€ΠΌΠ°Π»ΡŒΠ½Ρ‹ΠΉ Ρ€Π°Π·ΠΌΠ΅Ρ€ R) Ρ€Π°Π²Π΅Π½ , прямо ΠΏΡ€ΠΎΠΏΠΎΡ€Ρ†ΠΈΠΎΠ½Π°Π»ΡŒΠ½ΠΎ Π΄Π»ΠΈΠ½Π΅ (Π½ΠΎΡ€ΠΌΠ°Π»ΡŒΠ½Ρ‹ΠΉ Ρ€Π°Π·ΠΌΠ΅Ρ€ L): ΡƒΠ΄Π²Π°ΠΈΠ²Π°Π΅Ρ‚ Π΄Π»ΠΈΠ½Ρƒ ΠΏΡ€ΠΎΠ²ΠΎΠ»ΠΎΡ‡Π½ΠΎΠΉ Ρ‚Ρ€ΡƒΠ±ΠΊΠΈ, ΠΈ Π΅Π΅ сопротивлСниС удваиваСтся. Но (Π½ΠΎΡ€ΠΌΠ°Π»ΡŒΠ½Ρ‹ΠΉ Ρ€Π°Π·ΠΌΠ΅Ρ€ R) Ρ‚Π°ΠΊΠΆΠ΅ ΠΎΠ±Ρ€Π°Ρ‚Π½ΠΎ ΠΏΡ€ΠΎΠΏΠΎΡ€Ρ†ΠΈΠΎΠ½Π°Π»Π΅Π½ ΠΏΠ»ΠΎΡ‰Π°Π΄ΠΈ ΠΏΠΎΠΏΠ΅Ρ€Π΅Ρ‡Π½ΠΎΠ³ΠΎ сСчСния (Π½ΠΎΡ€ΠΌΠ°Π»ΡŒΠ½Ρ‹ΠΉ Ρ€Π°Π·ΠΌΠ΅Ρ€ A): ΡƒΠ΄Π²ΠΎΠΈΡ‚ΡŒ ΠΏΠ»ΠΎΡ‰Π°Π΄ΡŒ ΠΈ ΠΏΠΎΠ»ΠΎΠ²ΠΈΠ½Ρƒ сопротивлСния.

3 ΠΊΠ². (E): Ρ‚Ρ€ΡƒΠ±Ρ‡Π°Ρ‚Ρ‹ΠΉ рСзистор ΠΈΠΌΠ΅Π΅Ρ‚ Ρ„ΠΎΡ€ΠΌΡƒ ΠΏΡ€ΠΎΠ²ΠΎΠ»ΠΎΠΊΠΈ. Если ΠΌΡ‹ ΡƒΡ‚Ρ€ΠΎΠΈΠΌ Π΅Π³ΠΎ Π΄Π»ΠΈΠ½Ρƒ ΠΈ ΡƒΠΌΠ΅Π½ΡŒΡˆΠΈΠΌ Π²Π΄Π²ΠΎΠ΅ Π΄ΠΈΠ°ΠΌΠ΅Ρ‚Ρ€, Ρ‡Ρ‚ΠΎ ΠΏΡ€ΠΎΠΈΠ·ΠΎΠΉΠ΄Π΅Ρ‚ с Π΅Π³ΠΎ сопротивлСниСм?

ГидравличСская аналогия

Для понимания Π·Π°ΠΊΠΎΠ½Π° Ома ΠΈΠ½ΠΎΠ³Π΄Π° Π±Ρ‹Π²Π°Π΅Ρ‚ ΠΏΠΎΠ»Π΅Π·Π½Π° аналогия с Π³ΠΈΠ΄Ρ€Π°Π²Π»ΠΈΠΊΠΎΠΉ для Π½Π°Ρ‡ΠΈΠ½Π°ΡŽΡ‰ΠΈΡ….ΠŸΡ€Π΅Π΄ΡΡ‚Π°Π²ΡŒΡ‚Π΅ сСбС Π²ΠΎΠ΄Ρƒ, Ρ‚Π΅ΠΊΡƒΡ‰ΡƒΡŽ ΠΏΠΎ Π³ΠΎΡ€ΠΈΠ·ΠΎΠ½Ρ‚Π°Π»ΡŒΠ½ΠΎΠΉ Ρ‚Ρ€ΡƒΠ±Π΅. Π”Π°Π²Π»Π΅Π½ΠΈΠ΅ Π²ΠΎΠ΄Ρ‹ (Π½ΠΎΡ€ΠΌΠ°Π»ΡŒΠ½Ρ‹ΠΉ Ρ€Π°Π·ΠΌΠ΅Ρ€ P) Π°Π½Π°Π»ΠΎΠ³ΠΈΡ‡Π½ΠΎ Π½Π°ΠΏΡ€ΡΠΆΠ΅Π½ΠΈΡŽ (Π½ΠΎΡ€ΠΌΠ°Π»ΡŒΠ½Ρ‹ΠΉ Ρ€Π°Π·ΠΌΠ΅Ρ€ V), ΠΏΠΎΡ‚ΠΎΠΌΡƒ Ρ‡Ρ‚ΠΎ это Ρ€Π°Π·Π½ΠΈΡ†Π° Π΄Π°Π²Π»Π΅Π½ΠΈΠΉ ΠΌΠ΅ΠΆΠ΄Ρƒ двумя Ρ‚ΠΎΡ‡ΠΊΠ°ΠΌΠΈ вдоль Ρ‚Ρ€ΡƒΠ±Ρ‹, которая заставляСт Π²ΠΎΠ΄Ρƒ Ρ‚Π΅Ρ‡ΡŒ. ЀактичСский расход Π²ΠΎΠ΄Ρ‹ (Π½ΠΎΡ€ΠΌΠ°Π»ΡŒΠ½Ρ‹ΠΉ Ρ€Π°Π·ΠΌΠ΅Ρ€ F) Π² этом случаС являСтся Π°Π½Π°Π»ΠΎΠ³ΠΎΠΌ Ρ‚ΠΎΠΊΠ° (Π½ΠΎΡ€ΠΌΠ°Π»ΡŒΠ½Ρ‹ΠΉ Ρ€Π°Π·ΠΌΠ΅Ρ€ I).

А Ρ‡Ρ‚ΠΎ с Π°Π½Π°Π»ΠΎΠ³ΠΎΠΌ рСзистора? Π­Ρ‚ΠΎ ΠΌΠΎΠΆΠ½ΠΎ ΠΏΡ€Π΅Π΄ΡΡ‚Π°Π²ΠΈΡ‚ΡŒ ΠΊΠ°ΠΊ Π½Π΅Ρ‡Ρ‚ΠΎ, ΠΏΡ€Π΅ΠΏΡΡ‚ΡΡ‚Π²ΡƒΡŽΡ‰Π΅Π΅ ΠΏΠΎΡ‚ΠΎΠΊΡƒ Π²ΠΎΠ΄Ρ‹, Π½Π°ΠΏΡ€ΠΈΠΌΠ΅Ρ€, ΠΎΠ³Ρ€Π°Π½ΠΈΡ‡ΠΈΡ‚Π΅Π»ΠΈ ΠΈΠ»ΠΈ отвСрстия Π² Ρ‚Ρ€ΡƒΠ±Π°Ρ…. Если Π²ΠΎΠ΄Π° проталкиваСтся Ρ‡Π΅Ρ€Π΅Π· ΠΎΡ‡Π΅Π½ΡŒ Ρ‚ΠΎΠ½ΠΊΡƒΡŽ Ρ‚Ρ€ΡƒΠ±ΠΊΡƒ, Ρ‚ΠΎ Ρ‡Π΅ΠΌ Π΄Π»ΠΈΠ½Π½Π΅Π΅ Ρ‚Ρ€ΡƒΠ±ΠΊΠ° ΠΈ Ρ‡Π΅ΠΌ мСньшС Π΅Π΅ ΠΏΠ»ΠΎΡ‰Π°Π΄ΡŒ ΠΏΠΎΠΏΠ΅Ρ€Π΅Ρ‡Π½ΠΎΠ³ΠΎ сСчСния, Ρ‚Π΅ΠΌ большСС сопротивлСниС (Π½ΠΎΡ€ΠΌΠ°Π»ΡŒΠ½Ρ‹ΠΉ Ρ€Π°Π·ΠΌΠ΅Ρ€ R) ΠΎΠ½Π° Π±ΡƒΠ΄Π΅Ρ‚ ΠΎΠΊΠ°Π·Ρ‹Π²Π°Ρ‚ΡŒ Π½Π° расход Π²ΠΎΠ΄Ρ‹ (Π½ΠΎΡ€ΠΌΠ°Π»ΡŒΠ½Ρ‹ΠΉ Ρ€Π°Π·ΠΌΠ΅Ρ€ F).И Ρ‡Π΅ΠΌ большС сопротивлСниС, Ρ‚Π΅ΠΌ мСньшС расход.

Π‘ΠΎΠΎΡ‚Π²Π΅Ρ‚ΡΡ‚Π²ΡƒΡŽΡ‰Π΅Π΅ ΡƒΡ€Π°Π²Π½Π΅Π½ΠΈΠ΅ для нашСго гидравличСского Π°Π½Π°Π»ΠΎΠ³Π° Π² ΡΠΎΠΎΡ‚Π²Π΅Ρ‚ΡΡ‚Π²ΡƒΡŽΡ‰ΠΈΡ… Π΅Π΄ΠΈΠ½ΠΈΡ†Π°Ρ… Ρ€Π°Π²Π½ΠΎ

[Π‘ΠΎΠ»ΡŒΡˆΠΎΠΉ P = FR.]

Π’Π°ΠΊΠΈΠΌ ΠΎΠ±Ρ€Π°Π·ΠΎΠΌ, Ссли ΠΌΡ‹ сохраняСм Π΄Π°Π²Π»Π΅Π½ΠΈΠ΅ фиксированным, Ρ‚ΠΎ расход ΠΈ ΠΎΠ³Ρ€Π°Π½ΠΈΡ‡Π΅Π½ΠΈΠ΅ Π±ΡƒΠ΄ΡƒΡ‚ ΠΎΠ±Ρ€Π°Ρ‚Π½ΠΎ ΠΏΡ€ΠΎΠΏΠΎΡ€Ρ†ΠΈΠΎΠ½Π°Π»ΡŒΠ½Ρ‹: ΠΏΠΎ ΠΌΠ΅Ρ€Π΅ ΡƒΠΌΠ΅Π½ΡŒΡˆΠ΅Π½ΠΈΡ Ρ€Π°Π·ΠΌΠ΅Ρ€Π° ограничСния (Π½ΠΎΡ€ΠΌΠ°Π»ΡŒΠ½Ρ‹ΠΉ Ρ€Π°Π·ΠΌΠ΅Ρ€ R) расход (Π½ΠΎΡ€ΠΌΠ°Π»ΡŒΠ½Ρ‹ΠΉ Ρ€Π°Π·ΠΌΠ΅Ρ€ F) Π΄ΠΎΠ»ΠΆΠ΅Π½ ΡƒΠ²Π΅Π»ΠΈΡ‡ΠΈΠ²Π°Ρ‚ΡŒΡΡ.

На рисункС Π½ΠΈΠΆΠ΅ ΠΌΡ‹ ΠΎΠΆΠΈΠ΄Π°Π΅ΠΌ, Ρ‡Ρ‚ΠΎ Π±ΠΎΠ»Π΅Π΅ тонкая Ρ‚Ρ€ΡƒΠ±ΠΊΠ° Π±ΡƒΠ΄Π΅Ρ‚ Π΄Π΅ΠΉΡΡ‚Π²ΠΎΠ²Π°Ρ‚ΡŒ ΠΊΠ°ΠΊ сопротивлСниС ΠΏΠΎΡ‚ΠΎΠΊΡƒ Π² большой Ρ‚Ρ€ΡƒΠ±ΠΊΠ΅.

ΠΎΡ‚Π²Π΅Ρ‚Ρ‹

A1. По Π·Π°ΠΊΠΎΠ½Ρƒ Ома Ρ‚ΠΎΠΊ ΠΌΠΎΠΆΠ½ΠΎ Π½Π°ΠΉΡ‚ΠΈ ΠΏΠΎ

[Π‘ΠΎΠ»ΡŒΡˆΠΎΠΉ {I = Π³ΠΈΠ΄Ρ€ΠΎΡ€Π°Π·Ρ€Ρ‹Π² {V} {R} = Π³ΠΈΠ΄Ρ€ΠΎΡ€Π°Π·Ρ€Ρ‹Π² {120} {480} = 0,25; тСкст {Π°ΠΌΠΏΠ΅Ρ€Ρ‹}}.]

A2. ΠŸΡ€ΠΈ постоянном напряТСнии ΡΠΎΠΎΡ‚Π½ΠΎΡˆΠ΅Π½ΠΈΠ΅ ΠΌΠ΅ΠΆΠ΄Ρƒ Ρ‚ΠΎΠΊΠΎΠΌ ΠΈ сопротивлСниСм ΠΎΠ±Ρ€Π°Ρ‚Π½ΠΎΠ΅. Π‘Π»Π΅Π΄ΠΎΠ²Π°Ρ‚Π΅Π»ΡŒΠ½ΠΎ, Ссли Ρ‚ΠΎΠΊ ΡƒΠΌΠ΅Π½ΡŒΡˆΠ°Π΅Ρ‚ΡΡ Π²Π΄Π²ΠΎΠ΅, сопротивлСниС увСличиваСтся Π²Π΄Π²ΠΎΠ΅.

A3. Π£Ρ‚Ρ€ΠΎΠ΅Π½ΠΈΠ΅ Π΄Π»ΠΈΠ½Ρ‹ рСзистора ΡƒΠ²Π΅Π»ΠΈΡ‡ΠΈΠ²Π°Π΅Ρ‚ Π΅Π³ΠΎ сопротивлСниС Π² 3 Ρ€Π°Π·Π°, Π° ΡƒΠΌΠ΅Π½ΡŒΡˆΠ΅Π½ΠΈΠ΅ Π΅Π³ΠΎ Π΄ΠΈΠ°ΠΌΠ΅Ρ‚Ρ€Π° Π²Π΄Π²ΠΎΠ΅ ΡƒΠ²Π΅Π»ΠΈΡ‡ΠΈΠ²Π°Π΅Ρ‚ ΠΏΠ»ΠΎΡ‰Π°Π΄ΡŒ ΠΏΠΎΠΏΠ΅Ρ€Π΅Ρ‡Π½ΠΎΠ³ΠΎ сСчСния Π½Π° 1/4.Π’ Ρ†Π΅Π»ΠΎΠΌ сопротивлСниС измСняСтся Π² Ρ€Π°Π· (frac {3} {1/4} = 12).

ВСкущая Ρ„ΠΎΡ€ΠΌΡƒΠ»Π° — Π§Ρ‚ΠΎ Ρ‚Π°ΠΊΠΎΠ΅ тСкущая Ρ„ΠΎΡ€ΠΌΡƒΠ»Π°? ΠŸΡ€ΠΈΠΌΠ΅Ρ€Ρ‹

ВСкущая Ρ„ΠΎΡ€ΠΌΡƒΠ»Π° ΠΏΠΎΠ»ΡƒΡ‡Π΅Π½Π° ΠΈΠ· Π·Π°ΠΊΠΎΠ½Π° Ома. Π’ΠΎΠΊ опрСдСляСтся ΠΊΠ°ΠΊ ΠΏΠΎΡ‚ΠΎΠΊ элСктронов Π² элСктричСской Ρ†Π΅ΠΏΠΈ. ΠŸΠΎΡ‚ΠΎΠΊ элСктронов происходит ΠΈΠ·-Π·Π° разности ΠΏΠΎΡ‚Π΅Π½Ρ†ΠΈΠ°Π»ΠΎΠ². Π‘ΠΈΠ»Π° Ρ‚ΠΎΠΊΠ° Ρ‚Π°ΠΊΠΆΠ΅ извСстна ΠΊΠ°ΠΊ ΡΠΊΠΎΡ€ΠΎΡΡ‚ΡŒ измСнСния заряда Π²ΠΎ Π²Ρ€Π΅ΠΌΠ΅Π½ΠΈ.Π‘ΠΈΠ»Π° Ρ‚ΠΎΠΊΠ° обозначаСтся Π±ΡƒΠΊΠ²ΠΎΠΉ I, Π° Π΅Π΄ΠΈΠ½ΠΈΡ†Π° измСрСния Ρ‚ΠΎΠΊΠ° Π² систСмС БИ — АмпСр. Π”Π°Π²Π°ΠΉΡ‚Π΅ ΠΈΠ·ΡƒΡ‡ΠΈΠΌ ΠΏΡ€ΠΈΠΌΠ΅Π½Π΅Π½ΠΈΠ΅ Ρ‚Π΅ΠΊΡƒΡ‰Π΅ΠΉ Ρ„ΠΎΡ€ΠΌΡƒΠ»Ρ‹ Π² Ρ€Π°Π·Π΄Π΅Π»Π΅ Π½ΠΈΠΆΠ΅.

Какова тСкущая Ρ„ΠΎΡ€ΠΌΡƒΠ»Π°?

Богласно Π·Π°ΠΊΠΎΠ½Ρƒ Ома, Ρ‚ΠΎΠΊ — это ΠΎΡ‚Π½ΠΎΡˆΠ΅Π½ΠΈΠ΅ разности ΠΏΠΎΡ‚Π΅Π½Ρ†ΠΈΠ°Π»ΠΎΠ² ΠΈ сопротивлСния. Π’Π°ΠΊΠΈΠΌ ΠΎΠ±Ρ€Π°Π·ΠΎΠΌ, тСкущая Ρ„ΠΎΡ€ΠΌΡƒΠ»Π° ΠΈΠΌΠ΅Π΅Ρ‚ Π²ΠΈΠ΄: I = V / R

Π³Π΄Π΅

  • Π― ΠΏΡ€Π΅Π΄ΡΡ‚Π°Π²Π»ΡΡŽ Ρ‚ΠΎΠΊ Π² Π°ΠΌΠΏΠ΅Ρ€Π°Ρ…,
  • Π’ — Ρ€Π°Π·Π½ΠΎΡΡ‚ΡŒ ΠΏΠΎΡ‚Π΅Π½Ρ†ΠΈΠ°Π»ΠΎΠ² Π² Π²ΠΎΠ»ΡŒΡ‚Π°Ρ…
  • R — сопротивлСниС Π² Ом (Ом).

Π”Π°Π²Π°ΠΉΡ‚Π΅ посмотрим Π½Π° ΠΏΡ€ΠΈΠΌΠ΅Π½Π΅Π½ΠΈΠ΅ Ρ‚Π΅ΠΊΡƒΡ‰Π΅ΠΉ Ρ„ΠΎΡ€ΠΌΡƒΠ»Ρ‹ Π² ΡΠ»Π΅Π΄ΡƒΡŽΡ‰Π΅ΠΌ Ρ€Π°Π·Π΄Π΅Π»Π΅ Ρ€Π΅ΡˆΠ΅Π½Π½Ρ‹Ρ… ΠΏΡ€ΠΈΠΌΠ΅Ρ€ΠΎΠ².

Π₯ΠΎΡ‚ΠΈΡ‚Π΅ Π½Π°ΠΉΡ‚ΠΈ слоТныС матСматичСскиС Ρ€Π΅ΡˆΠ΅Π½ΠΈΡ Π·Π° сСкунды?

Π’ΠΎΡΠΏΠΎΠ»ΡŒΠ·ΡƒΠΉΡ‚Π΅ΡΡŒ нашим бСсплатным ΠΎΠ½Π»Π°ΠΉΠ½-ΠΊΠ°Π»ΡŒΠΊΡƒΠ»ΡΡ‚ΠΎΡ€ΠΎΠΌ для Ρ€Π΅ΡˆΠ΅Π½ΠΈΡ слоТных вопросов. Π‘ Cuemath Π½Π°Ρ…ΠΎΠ΄ΠΈΡ‚Π΅ Ρ€Π΅ΡˆΠ΅Π½ΠΈΡ простым ΠΈ Π»Π΅Π³ΠΊΠΈΠΌ способом.

Π—Π°Π±Ρ€ΠΎΠ½ΠΈΡ€ΡƒΠΉΡ‚Π΅ Π±Π΅ΡΠΏΠ»Π°Ρ‚Π½ΡƒΡŽ ΠΏΡ€ΠΎΠ±Π½ΡƒΡŽ Π²Π΅Ρ€ΡΠΈΡŽ Класс

ΠŸΡ€ΠΈΠΌΠ΅Ρ€Ρ‹ использования Ρ‚Π΅ΠΊΡƒΡ‰Π΅ΠΉ Ρ„ΠΎΡ€ΠΌΡƒΠ»Ρ‹

ΠŸΡ€ΠΈΠΌΠ΅Ρ€ 1: Π’ элСктричСской Ρ†Π΅ΠΏΠΈ Ρ€Π°Π·Π½ΠΎΡΡ‚ΡŒ ΠΏΠΎΡ‚Π΅Π½Ρ†ΠΈΠ°Π»ΠΎΠ² ΠΈ сопротивлСниС Π·Π°Π΄Π°ΡŽΡ‚ΡΡ ΠΊΠ°ΠΊ 20 Π’ ΠΈ 4 Ом соотвСтствСнно.Π˜ΡΠΏΠΎΠ»ΡŒΠ·ΡƒΡ Ρ„ΠΎΡ€ΠΌΡƒΠ»Ρƒ Ρ‚ΠΎΠΊΠ°, Π½Π°ΠΉΠ΄ΠΈΡ‚Π΅ Ρ‚ΠΎΠΊ, ΠΏΡ€ΠΎΡ‚Π΅ΠΊΠ°ΡŽΡ‰ΠΈΠΉ Π² Ρ†Π΅ΠΏΠΈ.

РСшСниС:

Π§Ρ‚ΠΎΠ±Ρ‹ Π½Π°ΠΉΡ‚ΠΈ: Π’ΠΎΠΊ (I), ΠΏΡ€ΠΎΡ‚Π΅ΠΊΠ°ΡŽΡ‰ΠΈΠΉ Π² Ρ†Π΅ΠΏΠΈ.
Π”Π°Π½ΠΎ:
V = 20 Π’, R = 4 Ом
Π˜ΡΠΏΠΎΠ»ΡŒΠ·ΡƒΡ Ρ‚Π΅ΠΊΡƒΡ‰ΡƒΡŽ Ρ„ΠΎΡ€ΠΌΡƒΠ»Ρƒ
I = V / R
I = 20/4
I = 5

ΠžΡ‚Π²Π΅Ρ‚: Π’ Ρ†Π΅ΠΏΠΈ ΠΏΡ€ΠΎΡ‚Π΅ΠΊΠ°Π΅Ρ‚ Ρ‚ΠΎΠΊ 5 Π°ΠΌΠΏΠ΅Ρ€.

ΠŸΡ€ΠΈΠΌΠ΅Ρ€ 2: ΠŸΠΎΠ»Π½Ρ‹ΠΉ Ρ‚ΠΎΠΊ, ΠΏΡ€ΠΎΡ‚Π΅ΠΊΠ°ΡŽΡ‰ΠΈΠΉ Π² элСктричСской Ρ†Π΅ΠΏΠΈ, составляСт 50 АмпСр, Π° сопротивлСниС ΠΏΡ€ΠΎΠ²ΠΎΠ΄ΠΎΠ² — 14 Ом.Π˜ΡΠΏΠΎΠ»ΡŒΠ·ΡƒΡ Ρ‚Π΅ΠΊΡƒΡ‰ΡƒΡŽ Ρ„ΠΎΡ€ΠΌΡƒΠ»Ρƒ, Π½Π°ΠΉΠ΄ΠΈΡ‚Π΅ Ρ€Π°Π·Π½ΠΎΡΡ‚ΡŒ ΠΏΠΎΡ‚Π΅Π½Ρ†ΠΈΠ°Π»ΠΎΠ².

РСшСниС:

Π§Ρ‚ΠΎΠ±Ρ‹ Π½Π°ΠΉΡ‚ΠΈ Ρ€Π°Π·Π½ΠΎΡΡ‚ΡŒ ΠΏΠΎΡ‚Π΅Π½Ρ†ΠΈΠ°Π»ΠΎΠ²:
Π”Π°Π½ΠΎ:
I = 50 А, R = 14 Ом
Π˜ΡΠΏΠΎΠ»ΡŒΠ·ΡƒΡ Ρ‚Π΅ΠΊΡƒΡ‰ΡƒΡŽ Ρ„ΠΎΡ€ΠΌΡƒΠ»Ρƒ
I = V / R
50 = V / 14
V = 50 Γ— 14
V = 700

ΠžΡ‚Π²Π΅Ρ‚: Π Π°Π·Π½ΠΈΡ†Π° ΠΏΠΎΡ‚Π΅Π½Ρ†ΠΈΠ°Π»ΠΎΠ² 700 Π’.

ΠŸΡ€ΠΈΠΌΠ΅Ρ€ 3: Π’ элСктричСской Ρ†Π΅ΠΏΠΈ Ρ€Π°Π·Π½ΠΎΡΡ‚ΡŒ ΠΏΠΎΡ‚Π΅Π½Ρ†ΠΈΠ°Π»ΠΎΠ² составляСт 20 Π’, Π° Π·Π½Π°Ρ‡Π΅Π½ΠΈΠ΅ Ρ‚ΠΎΠΊΠ° составляСт 5 А соотвСтствСнно.Π˜ΡΠΏΠΎΠ»ΡŒΠ·ΡƒΡ Ρ„ΠΎΡ€ΠΌΡƒΠ»Ρƒ Ρ‚ΠΎΠΊΠ°, Π½Π°ΠΉΠ΄ΠΈΡ‚Π΅ сопротивлСниС Ρ†Π΅ΠΏΠΈ.

РСшСниС:

Π§Ρ‚ΠΎΠ±Ρ‹ Π½Π°ΠΉΡ‚ΠΈ сопротивлСниС (R) Ρ†Π΅ΠΏΠΈ:
Π”Π°Π½ΠΎ:
V = 20 Π’, I = 5 А
Π˜ΡΠΏΠΎΠ»ΡŒΠ·ΡƒΡ Ρ‚Π΅ΠΊΡƒΡ‰ΡƒΡŽ Ρ„ΠΎΡ€ΠΌΡƒΠ»Ρƒ
R = V / I
R = 20/5
R = 4 Ом

ΠžΡ‚Π²Π΅Ρ‚: Π‘ΠΎΠΏΡ€ΠΎΡ‚ΠΈΠ²Π»Π΅Π½ΠΈΠ΅ Ρ†Π΅ΠΏΠΈ 4 Ом.

Часто Π·Π°Π΄Π°Π²Π°Π΅ΠΌΡ‹Π΅ вопросы ΠΎ Ρ‚Π΅ΠΊΡƒΡ‰Π΅ΠΉ Ρ„ΠΎΡ€ΠΌΡƒΠ»Π΅

Как Ρ€Π°ΡΡΡ‡ΠΈΡ‚Π°Ρ‚ΡŒ Ρ‚ΠΎΠΊ ΠΏΠΎ Ρ‚Π΅ΠΊΡƒΡ‰Π΅ΠΉ Ρ„ΠΎΡ€ΠΌΡƒΠ»Π΅?

Если Π·Π°Π΄Π°Π½Ρ‹ напряТСниС (Π’) ΠΈ сопротивлСниС (R) любой Ρ†Π΅ΠΏΠΈ, ΠΌΡ‹ ΠΌΠΎΠΆΠ΅ΠΌ ΠΈΡΠΏΠΎΠ»ΡŒΠ·ΠΎΠ²Π°Ρ‚ΡŒ Ρ„ΠΎΡ€ΠΌΡƒΠ»Ρƒ Ρ‚ΠΎΠΊΠ° для вычислСния Ρ‚ΠΎΠΊΠ°, Ρ‚.Π΅.Π΅., I = V / R (Π°ΠΌΠΏΠ΅Ρ€Ρ‹).

Как Ρ€Π°ΡΡΡ‡ΠΈΡ‚Π°Ρ‚ΡŒ напряТСниС ΠΏΠΎ Ρ„ΠΎΡ€ΠΌΡƒΠ»Π΅ Ρ‚ΠΎΠΊΠ°?

Если Π·Π°Π΄Π°Π½Ρ‹ Ρ‚ΠΎΠΊ (I) ΠΈ сопротивлСниС (R) любой Ρ†Π΅ΠΏΠΈ, ΠΌΡ‹ ΠΌΠΎΠΆΠ΅ΠΌ ΡΠΎΡΡ‚Π°Π²ΠΈΡ‚ΡŒ Ρ„ΠΎΡ€ΠΌΡƒΠ»Ρƒ Ρ‚ΠΎΠΊΠ° для вычислСния напряТСния, Ρ‚ΠΎ Π΅ΡΡ‚ΡŒ V = IR (Π’ΠΎΠ»ΡŒΡ‚).

Как Ρ€Π°ΡΡΡ‡ΠΈΡ‚Π°Ρ‚ΡŒ сопротивлСниС ΠΏΠΎ Ρ‚Π΅ΠΊΡƒΡ‰Π΅ΠΉ Ρ„ΠΎΡ€ΠΌΡƒΠ»Π΅?

Если Π·Π°Π΄Π°Π½Ρ‹ Ρ‚ΠΎΠΊ (I) ΠΈ Ρ€Π°Π·Π½ΠΎΡΡ‚ΡŒ ΠΏΠΎΡ‚Π΅Π½Ρ†ΠΈΠ°Π»ΠΎΠ² (V) любой Ρ†Π΅ΠΏΠΈ, ΠΌΡ‹ ΠΌΠΎΠΆΠ΅ΠΌ ΡΠΎΡΡ‚Π°Π²ΠΈΡ‚ΡŒ Ρ„ΠΎΡ€ΠΌΡƒΠ»Ρƒ Ρ‚ΠΎΠΊΠ° для вычислСния сопротивлСния, Ρ‚ΠΎ Π΅ΡΡ‚ΡŒ R = V / I (Ом Ом).

Π§Ρ‚ΠΎ Ρ‚Π°ΠΊΠΎΠ΅ ΠΎΠΏΡ€Π΅Π΄Π΅Π»Π΅Π½ΠΈΠ΅ Ρ‚Π΅ΠΊΡƒΡ‰Π΅ΠΉ Ρ„ΠΎΡ€ΠΌΡƒΠ»Ρ‹? ΠΠ°ΠΏΠΈΡˆΠΈΡ‚Π΅ Π΅Π³ΠΎ Π΅Π΄ΠΈΠ½ΠΈΡ†Ρƒ БИ.

Π’ΠΎΠΊ — это ΠΎΡ‚Π½ΠΎΡˆΠ΅Π½ΠΈΠ΅ разности ΠΏΠΎΡ‚Π΅Π½Ρ†ΠΈΠ°Π»ΠΎΠ² ΠΈ сопротивлСния. Он прСдставлСн ΠΊΠ°ΠΊ (I). ВСкущая Ρ„ΠΎΡ€ΠΌΡƒΠ»Π° прСдставлСна ​​как I = V / R. Π•Π΄ΠΈΠ½ΠΈΡ†Π° измСрСния Ρ‚ΠΎΠΊΠ° Π² систСмС БИ — АмпСр (АмпСр).

ОбъяснСниС элСктричСского сопротивлСния

| ΠžΠΏΡ€Π΅Π΄Π΅Π»Π΅Π½ΠΈΠ΅, Π΅Π΄ΠΈΠ½ΠΈΡ†Π°, Ρ„ΠΎΡ€ΠΌΡƒΠ»Π°, ΠΏΡ€ΠΈΠΌΠ΅Ρ€Ρ‹

Π‘ΠΎΠΏΡ€ΠΎΡ‚ΠΈΠ²Π»Π΅Π½ΠΈΠ΅ ΠžΠΏΡ€Π΅Π΄Π΅Π»Π΅Π½ΠΈΠ΅: Π­Ρ‚ΠΎ противодСйствиС Ρ‚ΠΎΠΊΡƒ ΠΈ обозначаСтся Π±ΡƒΠΊΠ²ΠΎΠΉ R. Π•Π³ΠΎ Π΅Π΄ΠΈΠ½ΠΈΡ†Π΅ΠΉ являСтся Ом , обозначаСмая грСчСской Π±ΡƒΠΊΠ²ΠΎΠΉ Ξ©.

НапряТСниС , ΠΏΡ€ΠΈΠ»ΠΎΠΆΠ΅Π½Π½ΠΎΠ΅ ΠΊ ΠΏΡ€ΠΎΠ²ΠΎΠ΄Π½ΠΈΠΊΡƒ, Π²Ρ‹Π·Ρ‹Π²Π°Π΅Ρ‚ чистый Π΄Ρ€Π΅ΠΉΡ„ свободных элСктронов ΠΏΠΎ Π΄Π»ΠΈΠ½Π΅ ΠΏΡ€ΠΎΠ²ΠΎΠ΄Π½ΠΈΠΊΠ°.ΠžΡ‚Ρ‚Π°Π»ΠΊΠΈΠ²Π°Π½ΠΈΠ΅ элСктронов, ΠΏΠΎΠΏΠ°Π΄Π°ΡŽΡ‰ΠΈΡ… Π² ΠΏΡ€ΠΎΠ²ΠΎΠ΄Π½ΠΈΠΊ ΠΎΡ‚ источника энСргии схСмы, ускоряСт свободныС элСктроны вдоль ΠΏΡ€ΠΎΠ²ΠΎΠ΄Π½ΠΈΠΊΠ°. Π’Π°ΠΊΠΈΠΌ ΠΎΠ±Ρ€Π°Π·ΠΎΠΌ, энСргия ΠΎΡ‚ источника пСрСдаСтся свободным элСктронам Π² Π²ΠΈΠ΄Π΅ кинСтичСской энСргии.

Когда двиТущиСся элСктроны ΡΡ‚Π°Π»ΠΊΠΈΠ²Π°ΡŽΡ‚ΡΡ с Π°Ρ‚ΠΎΠΌΠ°ΠΌΠΈ Π² ΠΏΡ€ΠΎΠ²ΠΎΠ΄Π½ΠΈΠΊΠ΅, нСкоторая кинСтичСская энСргия пСрСдаСтся ΠΎΡ‚ элСктронов ΠΊ Π°Ρ‚ΠΎΠΌΠ°ΠΌ. ΠŸΠ΅Ρ€Π΅Π΄Π°Π²Π°Π΅ΠΌΠ°Ρ энСргия проявляСтся Π² Π²ΠΈΠ΄Π΅ Ρ‚Π΅ΠΏΠ»Π°, ΠΏΠΎΡΠΊΠΎΠ»ΡŒΠΊΡƒ ΡƒΠ²Π΅Π»ΠΈΡ‡ΠΈΠ²Π°Π΅Ρ‚ колСбания Π°Ρ‚ΠΎΠΌΠΎΠ² Π² Ρ€Π΅ΡˆΠ΅Ρ‚Ρ‡Π°Ρ‚ΠΎΠΉ ​​структурС ΠΏΡ€ΠΎΠ²ΠΎΠ΄Π½ΠΈΠΊΠ°.

БтолкновСния ΠΌΠ΅ΠΆΠ΄Ρƒ свободными элСктронами ΠΈ Π°Ρ‚ΠΎΠΌΠ°ΠΌΠΈ ΡƒΠΌΠ΅Π½ΡŒΡˆΠ°ΡŽΡ‚ ΡΠΊΠΎΡ€ΠΎΡΡ‚ΡŒ, с ΠΊΠΎΡ‚ΠΎΡ€ΠΎΠΉ элСктроны Π΄Ρ€Π΅ΠΉΡ„ΡƒΡŽΡ‚ Π² ΠΎΡ‚Π²Π΅Ρ‚ Π½Π° ΠΏΡ€ΠΈΠ»ΠΎΠΆΠ΅Π½Π½ΠΎΠ΅ напряТСниС.Π§Π΅ΠΌ большС частота столкновСний Π² ΠΌΠ°Ρ‚Π΅Ρ€ΠΈΠ°Π»Π΅, Ρ‚Π΅ΠΌ Π²Ρ‹ΡˆΠ΅ Π΅Π³ΠΎ сопротивлСниС.

Π’ΠΎΠΊ, ΠΏΡ€ΠΎΡ‚Π΅ΠΊΠ°ΡŽΡ‰ΠΈΠΉ Ρ‡Π΅Ρ€Π΅Π· сопротивлСниС, всСгда выдСляСт Ρ‚Π΅ΠΏΠ»ΠΎ. Π­Ρ‚ΠΎ Ρ‚Π΅ΠΏΠ»ΠΎ ΠΏΠ΅Ρ€Π΅Π΄Π°ΡŽΡ‚ Ρ‚Π°ΠΊΠΈΠ΅ устройства, ΠΊΠ°ΠΊ элСмСнты ΠΏΠ΅Ρ‡ΠΈ ΠΈ Π»Π°ΠΌΠΏΡ‹ накаливания.

Однако Π²ΠΎ ΠΌΠ½ΠΎΠ³ΠΈΡ… цСпях выдСляСмоС Ρ‚Π΅ΠΏΠ»ΠΎ являСтся Π½Π΅ΠΈΠ·Π±Π΅ΠΆΠ½ΠΎΠΉ ΠΏΠΎΡ‚Π΅Ρ€Π΅ΠΉ энСргии Π² систСмС. Π’ Π½Π΅ΠΊΠΎΡ‚ΠΎΡ€Ρ‹Ρ… случаях трСбуСтся вСнтиляция ΠΈΠ»ΠΈ Π΄Ρ€ΡƒΠ³ΠΎΠ΅ ΠΎΡ…Π»Π°ΠΆΠ΄Π΅Π½ΠΈΠ΅, Ρ‡Ρ‚ΠΎΠ±Ρ‹ ΠΎΡ‚Ρ€Π°Π±ΠΎΡ‚Π°Π½Π½ΠΎΠ΅ Ρ‚Π΅ΠΏΠ»ΠΎ Π½Π΅ ΠΏΠΎΠ²Ρ€Π΅Π΄ΠΈΠ»ΠΎ ΠΊΠΎΠ½Ρ‚ΡƒΡ€.

РСзистор — это ΠΊΠΎΠΌΠΏΠΎΠ½Π΅Π½Ρ‚, ΠΈΠΌΠ΅ΡŽΡ‰ΠΈΠΉ ΠΎΠΏΡ€Π΅Π΄Π΅Π»Π΅Π½Π½ΠΎΠ΅ сопротивлСниС.Если ΠΌΡ‹ ΠΏΠΎΠ΄ΠΊΠ»ΡŽΡ‡ΠΈΠΌ рСзистор ΠΌΠ΅ΠΆΠ΄Ρƒ Π»Π°ΠΌΠΏΠΎΠΉ ΠΈ источником напряТСния, ΠΊΠ°ΠΊ ΠΏΠΎΠΊΠ°Π·Π°Π½ΠΎ Π½Π° рисункС 1, ΠΎΠ±Ρ‰Π΅Π΅ сопротивлСниС Ρ†Π΅ΠΏΠΈ увСличится, Π° Ρ‚ΠΎΠΊ соотвСтствСнно ΡƒΠΌΠ΅Π½ΡŒΡˆΠΈΡ‚ΡΡ.

Π‘ΠΊΠΎΡ€ΠΎΡΡ‚ΡŒ, с ΠΊΠΎΡ‚ΠΎΡ€ΠΎΠΉ источник ΠΏΠ΅Ρ€Π΅Π΄Π°Π΅Ρ‚ ΡΠ½Π΅Ρ€Π³ΠΈΡŽ Π² Ρ†Π΅ΠΏΡŒ, Ρ‚Π°ΠΊΠΆΠ΅ ΡƒΠΌΠ΅Π½ΡŒΡˆΠ°Π΅Ρ‚ΡΡ, ΠΈ эта энСргия дСлится ΠΌΠ΅ΠΆΠ΄Ρƒ Π»Π°ΠΌΠΏΠΎΠΉ ΠΈ рСзистором. Π’Π°ΠΊΠΈΠΌ ΠΎΠ±Ρ€Π°Π·ΠΎΠΌ, Π΄ΠΎΠ±Π°Π²Π»Π΅Π½ΠΈΠ΅ рСзистора Π² схСму сниТаСт ΡΡ€ΠΊΠΎΡΡ‚ΡŒ Π»Π°ΠΌΠΏΡ‹.

Рисунок 1 ИспользованиС рСзистора для ограничСния Ρ‚ΠΎΠΊΠ°

Π€Π°ΠΊΡ‚ΠΎΡ€Ρ‹, Π²Π»ΠΈΡΡŽΡ‰ΠΈΠ΅ Π½Π° элСктричСскоС сопротивлСниС

ΠŸΡ€Π΅Π΄ΠΏΠΎΠ»ΠΎΠΆΠΈΠΌ, Ρƒ нас Π΅ΡΡ‚ΡŒ Π΄Π²Π° куска ΠΏΡ€ΠΎΠ²ΠΎΠ΄Π°, ΠΊΠΎΡ‚ΠΎΡ€Ρ‹Π΅ ΠΈΠ΄Π΅Π½Ρ‚ΠΈΡ‡Π½Ρ‹, Π·Π° ΠΈΡΠΊΠ»ΡŽΡ‡Π΅Π½ΠΈΠ΅ΠΌ Ρ‚ΠΎΠ³ΠΎ, Ρ‡Ρ‚ΠΎ ΠΎΠ΄ΠΈΠ½ Π² Π΄Π²Π° Ρ€Π°Π·Π° Π΄Π»ΠΈΠ½Π½Π΅Π΅ Π΄Ρ€ΡƒΠ³ΠΎΠ³ΠΎ.

Для свободных элСктронов, ΠΏΠ΅Ρ€Π΅ΠΌΠ΅Ρ‰Π°ΡŽΡ‰ΠΈΡ…ΡΡ ΠΏΠΎ всСй Π΄Π»ΠΈΠ½Π΅ этих ΠΏΡ€ΠΎΠ²ΠΎΠ΄ΠΎΠ², срСдний ΠΈΠ½Ρ‚Π΅Ρ€Π²Π°Π» ΠΌΠ΅ΠΆΠ΄Ρƒ столкновСниями с Π°Ρ‚ΠΎΠΌΠ°ΠΌΠΈ Π² ΠΏΡ€ΠΎΠ²ΠΎΠ΄Π΅ Ρ‚Π°ΠΊΠΎΠΉ ΠΆΠ΅. Π’Π°ΠΊΠΈΠΌ ΠΎΠ±Ρ€Π°Π·ΠΎΠΌ, элСктроны, проходящиС Ρ‡Π΅Ρ€Π΅Π· Π±ΠΎΠ»Π΅Π΅ Π΄Π»ΠΈΠ½Π½Ρ‹ΠΉ ΠΏΡ€ΠΎΠ²ΠΎΠ΄, ΠΈΠΌΠ΅ΡŽΡ‚ Π² Π΄Π²Π° Ρ€Π°Π·Π° большС столкновСний, Ρ‡Π΅ΠΌ элСктроны, проходящиС Ρ‡Π΅Ρ€Π΅Π· Π±ΠΎΠ»Π΅Π΅ ΠΊΠΎΡ€ΠΎΡ‚ΠΊΠΈΠΉ ΠΏΡ€ΠΎΠ²ΠΎΠ΄.

Π‘Π»Π΅Π΄ΠΎΠ²Π°Ρ‚Π΅Π»ΡŒΠ½ΠΎ, , сопротивлСниС Π±ΠΎΠ»Π΅Π΅ Π΄Π»ΠΈΠ½Π½ΠΎΠ³ΠΎ ΠΏΡ€ΠΎΠ²ΠΎΠ΄Π° элСктричСскому Ρ‚ΠΎΠΊΡƒ Π² Π΄Π²Π° Ρ€Π°Π·Π° большС, Ρ‡Π΅ΠΌ сопротивлСниС Π±ΠΎΠ»Π΅Π΅ ΠΊΠΎΡ€ΠΎΡ‚ΠΊΠΎΠ³ΠΎ ΠΏΡ€ΠΎΠ²ΠΎΠ΄Π°. ΠœΡ‹ ΠΌΠΎΠΆΠ΅ΠΌ ΠΎΠ±ΠΎΠ±Ρ‰ΠΈΡ‚ΡŒ это сравнСниС:

Π‘ΠΎΠΏΡ€ΠΎΡ‚ΠΈΠ²Π»Π΅Π½ΠΈΠ΅ ΠΏΡ€ΠΎΠ²ΠΎΠ΄Π½ΠΈΠΊΠ° прямо ΠΏΡ€ΠΎΠΏΠΎΡ€Ρ†ΠΈΠΎΠ½Π°Π»ΡŒΠ½ΠΎ Π΅Π³ΠΎ Π΄Π»ΠΈΠ½Π΅.

\ [R \ alpha l \]

Π—Π°Ρ‚Π΅ΠΌ ΠΌΡ‹ сравниваСм Π΄Π²Π° куска ΠΏΡ€ΠΎΠ²ΠΎΠ΄Π°, ΠΊΠΎΡ‚ΠΎΡ€Ρ‹Π΅ ΠΈΠ΄Π΅Π½Ρ‚ΠΈΡ‡Π½Ρ‹, Π·Π° ΠΈΡΠΊΠ»ΡŽΡ‡Π΅Π½ΠΈΠ΅ΠΌ Ρ‚ΠΎΠ³ΠΎ, Ρ‡Ρ‚ΠΎ ΠΎΠ΄ΠΈΠ½ ΠΈΠΌΠ΅Π΅Ρ‚ ΡƒΠ΄Π²ΠΎΠ΅Π½Π½ΡƒΡŽ ΠΏΠ»ΠΎΡ‰Π°Π΄ΡŒ ΠΏΠΎΠΏΠ΅Ρ€Π΅Ρ‡Π½ΠΎΠ³ΠΎ сСчСния Π΄Ρ€ΡƒΠ³ΠΎΠ³ΠΎ.

Π‘ΠΎΠ»Π΅Π΅ толстый ΠΏΡ€ΠΎΠ²ΠΎΠ΄ ΠΈΠΌΠ΅Π΅Ρ‚ Ρ‚Ρƒ ΠΆΠ΅ ΠΏΠ»ΠΎΡ‰Π°Π΄ΡŒ ΠΏΠΎΠΏΠ΅Ρ€Π΅Ρ‡Π½ΠΎΠ³ΠΎ сСчСния, Ρ‡Ρ‚ΠΎ ΠΈ Π΄Π²Π° куска Π±ΠΎΠ»Π΅Π΅ Ρ‚ΠΎΠ½ΠΊΠΎΠ³ΠΎ ΠΏΡ€ΠΎΠ²ΠΎΠ΄Π°, соСдинСнныС вмСстС Π½Π° ΠΎΠ±ΠΎΠΈΡ… ΠΊΠΎΠ½Ρ†Π°Ρ… ( соСдинСны ΠΏΠ°Ρ€Π°Π»Π»Π΅Π»ΡŒΠ½ΠΎ, ).

ΠšΠ°ΠΆΠ΄Ρ‹ΠΉ ΠΈΠ· Π΄Π²ΡƒΡ… ΠΏΡ€ΠΎΠ²ΠΎΠ΄ΠΎΠ² мСньшСго Π΄ΠΈΠ°ΠΌΠ΅Ρ‚Ρ€Π° Π±ΡƒΠ΄Π΅Ρ‚ ΠΏΡ€ΠΎΠΏΡƒΡΠΊΠ°Ρ‚ΡŒ ΠΎΠ΄ΠΈΠ½Π°ΠΊΠΎΠ²Ρ‹ΠΉ Ρ‚ΠΎΠΊ ΠΏΡ€ΠΈ ΠΏΠΎΠ΄ΠΊΠ»ΡŽΡ‡Π΅Π½ΠΈΠΈ ΠΊ Π·Π°Π΄Π°Π½Π½ΠΎΠΌΡƒ источнику напряТСния. Π‘Π»Π΅Π΄ΠΎΠ²Π°Ρ‚Π΅Π»ΡŒΠ½ΠΎ, Π±ΠΎΠ»Π΅Π΅ толстый ΠΏΡ€ΠΎΠ²ΠΎΠ΄ пропускаСт Π² Π΄Π²Π° Ρ€Π°Π·Π° большС Ρ‚ΠΎΠΊΠ°, Ρ‡Π΅ΠΌ Π±ΠΎΠ»Π΅Π΅ Ρ‚ΠΎΠ½ΠΊΠΈΠΉ ΠΏΡ€ΠΎΠ²ΠΎΠ΄ для Π΄Π°Π½Π½ΠΎΠ³ΠΎ ΠΏΡ€ΠΈΠ»ΠΎΠΆΠ΅Π½Π½ΠΎΠ³ΠΎ напряТСния.

ΠŸΠΎΡΠΊΠΎΠ»ΡŒΠΊΡƒ R = V / I, Π±ΠΎΠ»Π΅Π΅ толстая ΠΏΡ€ΠΎΠ²ΠΎΠ»ΠΎΠΊΠ° ΠΈΠΌΠ΅Π΅Ρ‚ ΠΏΠΎΠ»ΠΎΠ²ΠΈΠ½Ρƒ сопротивлСния Π±ΠΎΠ»Π΅Π΅ Ρ‚ΠΎΠ½ΠΊΠΎΠΉ ΠΏΡ€ΠΎΠ²ΠΎΠ»ΠΎΠΊΠΈ. ΠžΠΏΡΡ‚ΡŒ ΠΆΠ΅, ΠΌΡ‹ ΠΌΠΎΠΆΠ΅ΠΌ ΠΎΠ±ΠΎΠ±Ρ‰ΠΈΡ‚ΡŒ сравнСниС:

Π‘ΠΎΠΏΡ€ΠΎΡ‚ΠΈΠ²Π»Π΅Π½ΠΈΠ΅ ΠΏΡ€ΠΎΠ²ΠΎΠ΄Π½ΠΈΠΊΠ° ΠΎΠ±Ρ€Π°Ρ‚Π½ΠΎ ΠΏΡ€ΠΎΠΏΠΎΡ€Ρ†ΠΈΠΎΠ½Π°Π»ΡŒΠ½ΠΎ Π΅Π³ΠΎ ΠΏΠ»ΠΎΡ‰Π°Π΄ΠΈ ΠΏΠΎΠΏΠ΅Ρ€Π΅Ρ‡Π½ΠΎΠ³ΠΎ сСчСния.

\ [R \ alpha \ frac {1} {A} \]

Как ΠΌΡ‹ Π·Π½Π°Π΅ΠΌ, Π½Π΅ΠΊΠΎΡ‚ΠΎΡ€Ρ‹Π΅ ΠΌΠ°Ρ‚Π΅Ρ€ΠΈΠ°Π»Ρ‹ содСрТат большС свободных элСктронов Π½Π° Π΅Π΄ΠΈΠ½ΠΈΡ†Ρƒ объСма, Ρ‡Π΅ΠΌ Π΄Ρ€ΡƒΠ³ΠΈΠ΅. Π˜Π½Ρ‚Π΅Ρ€Π²Π°Π»Ρ‹ ΠΌΠ΅ΠΆΠ΄Ρƒ столкновСниями свободного элСктрона с Π°Ρ‚ΠΎΠΌΠ°ΠΌΠΈ Π² ΠΌΠ°Ρ‚Π΅Ρ€ΠΈΠ°Π»Π΅ ΠΈ энСргия, пСрСдаваСмая Π² Ρ€Π΅Π·ΡƒΠ»ΡŒΡ‚Π°Ρ‚Π΅ столкновСний, Ρ‚Π°ΠΊΠΆΠ΅ зависят ΠΎΡ‚ молСкулярной структуры ΠΌΠ°Ρ‚Π΅Ρ€ΠΈΠ°Π»Π°.

НапримСр, , сСрСбряный ΠΏΡ€ΠΎΠ²ΠΎΠ΄ ΠΈΠΌΠ΅Π΅Ρ‚ мСньшСС сопротивлСниС, Ρ‡Π΅ΠΌ ΠΌΠ΅Π΄Π½Ρ‹ΠΉ ΠΏΡ€ΠΎΠ²ΠΎΠ΄ с Ρ‚Π°ΠΊΠΈΠΌΠΈ ΠΆΠ΅ Ρ€Π°Π·ΠΌΠ΅Ρ€Π°ΠΌΠΈ, Π° ΠΌΠ΅Π΄Π½Ρ‹ΠΉ ΠΏΡ€ΠΎΠ²ΠΎΠ΄ ΠΈΠΌΠ΅Π΅Ρ‚ Π±ΠΎΠ»Π΅Π΅ Π½ΠΈΠ·ΠΊΠΎΠ΅ сопротивлСниС, Ρ‡Π΅ΠΌ Π°Π»ΡŽΠΌΠΈΠ½ΠΈΠ΅Π²Ρ‹ΠΉ ΠΏΡ€ΠΎΠ²ΠΎΠ΄ Ρ‚ΠΎΠ³ΠΎ ΠΆΠ΅ Ρ€Π°Π·ΠΌΠ΅Ρ€Π°.

Π‘ΠΎΠΏΡ€ΠΎΡ‚ΠΈΠ²Π»Π΅Π½ΠΈΠ΅ ΠΏΡ€ΠΎΠ²ΠΎΠ΄Π½ΠΈΠΊΠ° зависит ΠΎΡ‚ Π΅Π³ΠΎ состава.

УдСльноС сопротивлСниС

Π‘ΠΎΠΏΡ€ΠΎΡ‚ΠΈΠ²Π»Π΅Π½ΠΈΠ΅ ΠΏΡ€ΠΎΠ²ΠΎΠ΄Π½ΠΈΠΊΠ° прямо ΠΏΡ€ΠΎΠΏΠΎΡ€Ρ†ΠΈΠΎΠ½Π°Π»ΡŒΠ½ΠΎ Π΅Π³ΠΎ Π΄Π»ΠΈΠ½Π΅ ΠΈ ΠΎΠ±Ρ€Π°Ρ‚Π½ΠΎ ΠΏΡ€ΠΎΠΏΠΎΡ€Ρ†ΠΈΠΎΠ½Π°Π»ΡŒΠ½ΠΎ Π΅Π³ΠΎ ΠΏΠ»ΠΎΡ‰Π°Π΄ΠΈ ΠΏΠΎΠΏΠ΅Ρ€Π΅Ρ‡Π½ΠΎΠ³ΠΎ сСчСния.Π‘Π»Π΅Π΄ΠΎΠ²Π°Ρ‚Π΅Π»ΡŒΠ½ΠΎ, ΠΌΡ‹ ΠΌΠΎΠΆΠ΅ΠΌ Ρ€Π°ΡΡΡ‡ΠΈΡ‚Π°Ρ‚ΡŒ сопротивлСниС для Π»ΡŽΠ±Ρ‹Ρ… Ρ€Π°Π·ΠΌΠ΅Ρ€ΠΎΠ² ΠΏΡ€ΠΎΠ²ΠΎΠ΄Π½ΠΈΠΊΠ°, Ссли ΠΌΡ‹ Π·Π½Π°Π΅ΠΌ сопротивлСниС ΠΎΡ‚Ρ€Π΅Π·ΠΊΠ° ΠΌΠ°Ρ‚Π΅Ρ€ΠΈΠ°Π»Π° с ΠΎΠ΄Π½ΠΎΡ€ΠΎΠ΄Π½ΠΎΠΉ ΠΏΠ»ΠΎΡ‰Π°Π΄ΡŒΡŽ ΠΏΠΎΠΏΠ΅Ρ€Π΅Ρ‡Π½ΠΎΠ³ΠΎ сСчСния, ΠΏΡ€ΠΈ условии отсутствия ΠΈΠ·ΠΌΠ΅Π½Π΅Π½ΠΈΠΉ Ρ‚Π΅ΠΌΠΏΠ΅Ρ€Π°Ρ‚ΡƒΡ€Ρ‹.

\ [\ begin {matrix} \ frac {{{R} _ {2}}} {{{R} _ {1}}} = \ frac {{{l} _ {2}}} {{{ l} _ {1}}} \ times \ frac {{{A} _ {1}}} {{{A} _ {2}}} & {} & \ left (1 \ right) \\\ end { ΠΌΠ°Ρ‚Ρ€ΠΈΡ†Π°} \]

Π“Π΄Π΅ R — сопротивлСниС ΠΏΡ€ΠΎΠ²ΠΎΠ΄Π½ΠΈΠΊΠ°, l — Π΄Π»ΠΈΠ½Π°, Π° A — ΠΏΠ»ΠΎΡ‰Π°Π΄ΡŒ ΠΏΠΎΠΏΠ΅Ρ€Π΅Ρ‡Π½ΠΎΠ³ΠΎ сСчСния.

ΠŸΡ€ΠΈΠΌΠ΅Ρ€ расчСта элСктричСского сопротивлСния 1

ΠŸΡ€ΠΎΠ²ΠΎΠ΄Π½ΠΈΠΊ 1.{2}}} = 3.4 \ Omega \]

УдСльноС сопротивлСниС — удобная Π²Π΅Π»ΠΈΡ‡ΠΈΠ½Π° для расчСта сопротивлСния Π΄Π°Π½Π½ΠΎΠ³ΠΎ ΠΏΡ€ΠΎΠ²ΠΎΠ΄Π½ΠΈΠΊΠ°.

УдСльноС сопротивлСниС ΠžΠΏΡ€Π΅Π΄Π΅Π»Π΅Π½ΠΈΠ΅: УдСльноС сопротивлСниС ΠΌΠ°Ρ‚Π΅Ρ€ΠΈΠ°Π»Π° — это сопротивлСниС Π΅Π΄ΠΈΠ½ΠΈΡ†Ρ‹ Π΄Π»ΠΈΠ½Ρ‹ ΠΌΠ°Ρ‚Π΅Ρ€ΠΈΠ°Π»Π° с Π΅Π΄ΠΈΠ½ΠΈΡ†Π΅ΠΉ ΠΏΠ»ΠΎΡ‰Π°Π΄ΠΈ ΠΏΠΎΠΏΠ΅Ρ€Π΅Ρ‡Π½ΠΎΠ³ΠΎ сСчСния. Π‘ΡƒΠΊΠ²Π΅Π½Π½Ρ‹ΠΌ ΠΎΠ±ΠΎΠ·Π½Π°Ρ‡Π΅Π½ΠΈΠ΅ΠΌ ΡƒΠ΄Π΅Π»ΡŒΠ½ΠΎΠ³ΠΎ сопротивлСния являСтся грСчСская Π±ΡƒΠΊΠ²Π° ρ (Ρ€ΠΎ).

Π’ SI ΡƒΠ΄Π΅Π»ΡŒΠ½ΠΎΠ΅ сопротивлСниС ΠΌΠ°Ρ‚Π΅Ρ€ΠΈΠ°Π»Π° — это сопротивлСниС ΠΌΠ΅ΠΆΠ΄Ρƒ ΠΏΡ€ΠΎΡ‚ΠΈΠ²ΠΎΠΏΠΎΠ»ΠΎΠΆΠ½Ρ‹ΠΌΠΈ гранями ΠΊΡƒΠ±Π° ΠΌΠ°Ρ‚Π΅Ρ€ΠΈΠ°Π»Π° Ρ€Π°Π·ΠΌΠ΅Ρ€ΠΎΠΌ 1 ΠΌ вдоль ΠΊΠ°ΠΆΠ΄ΠΎΠΉ стороны. ΠœΡ‹ ΠΌΠΎΠΆΠ΅ΠΌ ΠΏΡ€ΠΈΠΌΠ΅Π½ΠΈΡ‚ΡŒ Ρ€Π°Π·ΠΌΠ΅Ρ€Π½Ρ‹ΠΉ Π°Π½Π°Π»ΠΈΠ· для опрСдСлСния Π΅Π΄ΠΈΠ½ΠΈΡ† ΡƒΠ΄Π΅Π»ΡŒΠ½ΠΎΠ³ΠΎ сопротивлСния.{2}}} {ΠΌΠ΅Ρ‚Ρ€} = ΠΎΠΌ-ΠΌΠ΅Ρ‚Ρ€ \\\ end {align} \]

Π•Π΄ΠΈΠ½ΠΈΡ†Π°: ΠžΠΌΠΌΠ΅Ρ‚Ρ€ — это Π΅Π΄ΠΈΠ½ΠΈΡ†Π° измСрСния ΡƒΠ΄Π΅Π»ΡŒΠ½ΠΎΠ³ΠΎ сопротивлСния Π² систСмС БИ. Π‘ΠΈΠΌΠ²ΠΎΠ» Π΅Π΄ΠΈΠ½ΠΈΡ†Ρ‹ измСрСния ΠΎΠΌΠΌΠ΅Ρ‚Ρ€Π° — Ом Β· ΠΌ.

ΠŸΠΎΡΠΊΠΎΠ»ΡŒΠΊΡƒ Ρ‚Π΅ΠΌΠΏΠ΅Ρ€Π°Ρ‚ΡƒΡ€Π° влияСт Π½Π° сопротивлСниС, значСния ΡƒΠ΄Π΅Π»ΡŒΠ½ΠΎΠ³ΠΎ сопротивлСния Π΄Π°Π½Ρ‹ для ΡƒΠΊΠ°Π·Π°Π½Π½ΠΎΠΉ Ρ‚Π΅ΠΌΠΏΠ΅Ρ€Π°Ρ‚ΡƒΡ€Ρ‹, ΠΎΠ±Ρ‹Ρ‡Π½ΠΎ 20 Β° C. Π’ Ρ‚Π°Π±Π»ΠΈΡ†Π΅ 1 ΠΏΡ€ΠΈΠ²Π΅Π΄Π΅Π½Ρ‹ значСния ΡƒΠ΄Π΅Π»ΡŒΠ½ΠΎΠ³ΠΎ сопротивлСния Π½Π°ΠΈΠ±ΠΎΠ»Π΅Π΅ распространСнных мСталличСских ΠΏΡ€ΠΎΠ²ΠΎΠ΄Π½ΠΈΠΊΠΎΠ² ΠΏΡ€ΠΈ 20 Β° C.

ΠœΠ°Ρ‚Π΅Ρ€ΠΈΠ°Π» УдСльноС сопротивлСниС (нОм)
Π‘Π΅Ρ€Π΅Π±Ρ€ΠΎ 16.4
МСдь 17,2
Алюминий 28,3
Π’ΠΎΠ»ΡŒΡ„Ρ€Π°ΠΌ 55
НикСль 78
НикСль 78 902 902 902 902 9040 902 902 9026 Π–Π΅Π»Π΅Π·ΠΎ
Нихром II Около 1100

Π’Π°Π±Π»ΠΈΡ†Π° 1 Π‘ΠΎΠΏΡ€ΠΎΡ‚ΠΈΠ²Π»Π΅Π½ΠΈΠ΅ Π½Π΅ΠΊΠΎΡ‚ΠΎΡ€Ρ‹Ρ… ΠΎΠ±Ρ‰ΠΈΡ… ΠΏΡ€ΠΎΠ²ΠΎΠ΄Π½ΠΈΠΊΠΎΠ² ΠΏΡ€ΠΈ 20 Β° C

Π€ΠΎΡ€ΠΌΡƒΠ»Π° элСктричСского сопротивлСния: Если ΡƒΡΡ‚Π°Π½ΠΎΠ²ΠΈΡ‚ΡŒ l 1 = 1 ΠΌ ΠΈ A 907 = 1 ΠΌ 2 Π² ΡƒΡ€Π°Π²Π½Π΅Π½ΠΈΠΈ 1, R 1 становится ρ.ΠŸΡ€Π΅ΠΎΠ±Ρ€Π°Π·ΠΎΠ²Π°Π½ΠΈΠ΅ уравнСния Π΄Π°Π΅Ρ‚ Ρ„ΠΎΡ€ΠΌΡƒΠ»Ρƒ для сопротивлСния любого ΠΏΡ€ΠΎΠ²ΠΎΠ΄Π½ΠΈΠΊΠ°:

\ [\ begin {matrix} R = \ rho \ frac {l} {A} & {} & \ left (2 \ right) \\\ end { ΠΌΠ°Ρ‚Ρ€ΠΈΡ†Π°} \]

Π“Π΄Π΅ R — сопротивлСниС ΠΏΡ€ΠΎΠ²ΠΎΠ΄Π½ΠΈΠΊΠ° Π² Ом, l — Π΄Π»ΠΈΠ½Π° ΠΏΡ€ΠΎΠ²ΠΎΠ΄Π½ΠΈΠΊΠ° Π² ΠΌΠ΅Ρ‚Ρ€Π°Ρ…, A — ΠΏΠ»ΠΎΡ‰Π°Π΄ΡŒ ΠΏΠΎΠΏΠ΅Ρ€Π΅Ρ‡Π½ΠΎΠ³ΠΎ сСчСния Π² ΠΊΠ²Π°Π΄Ρ€Π°Ρ‚Π½Ρ‹Ρ… ΠΌΠ΅Ρ‚Ρ€Π°Ρ…, Π° ρ — ΡƒΠ΄Π΅Π»ΡŒΠ½ΠΎΠ΅ сопротивлСниС ΠΌΠ°Ρ‚Π΅Ρ€ΠΈΠ°Π»Π° ΠΏΡ€ΠΎΠ²ΠΎΠ΄Π½ΠΈΠΊΠ° Π² Ом. ΠΌΠ΅Ρ‚Ρ€ΠΎΠ².

ΠŸΡ€ΠΈΠΌΠ΅Ρ€ расчСта элСктричСского сопротивлСния 2

НайдитС сопротивлСниС ΠΏΡ€ΠΈ 20 Β° C 200 ΠΌ алюминиСвого ΠΏΡ€ΠΎΠ²ΠΎΠ΄Π½ΠΈΠΊΠ° с ΠΏΠ»ΠΎΡ‰Π°Π΄ΡŒΡŽ ΠΏΠΎΠΏΠ΅Ρ€Π΅Ρ‡Π½ΠΎΠ³ΠΎ сСчСния 4.{2}}} = 1,4 \ Omega \]

Π’ этом Ρ€Π΅ΡˆΠ΅Π½ΠΈΠΈ ΠΏΡ€Π΅ΠΎΠ±Ρ€Π°Π·ΠΎΠ²Π°Π½ΠΈΠ΅ ΠΊΠ²Π°Π΄Ρ€Π°Ρ‚Π½Ρ‹Ρ… ΠΌΠΈΠ»Π»ΠΈΠΌΠ΅Ρ‚Ρ€ΠΎΠ² Π² ΠΊΠ²Π°Π΄Ρ€Π°Ρ‚Π½Ρ‹Π΅ ΠΌΠ΅Ρ‚Ρ€Ρ‹ позволяСт Π½Π°ΠΌ ΡΠΎΠΊΡ€Π°Ρ‚ΠΈΡ‚ΡŒ всС Π΅Π΄ΠΈΠ½ΠΈΡ†Ρ‹ измСрСния.

Π‘ΠΎΠ»ΡŒΡˆΠΈΠ½ΡΡ‚Π²ΠΎ ΠΏΡ€ΠΎΠ²ΠΎΠ΄Π½ΠΈΠΊΠΎΠ² ΠΈΠΌΠ΅ΡŽΡ‚ ΠΊΡ€ΡƒΠ³Π»ΠΎΠ΅ ΠΏΠΎΠΏΠ΅Ρ€Π΅Ρ‡Π½ΠΎΠ΅ сСчСниС. Зная Π΄ΠΈΠ°ΠΌΠ΅Ρ‚Ρ€ ΠΏΡ€ΠΎΠ²ΠΎΠ΄Π½ΠΈΠΊΠ°, ΠΌΠΎΠΆΠ½ΠΎ Ρ€Π°ΡΡΡ‡ΠΈΡ‚Π°Ρ‚ΡŒ ΠΏΠ»ΠΎΡ‰Π°Π΄ΡŒ ΠΏΠΎΠΏΠ΅Ρ€Π΅Ρ‡Π½ΠΎΠ³ΠΎ сСчСния.

ΠŸΡ€ΠΈΠΌΠ΅Ρ€ расчСта элСктричСского сопротивлСния 3

НайдитС сопротивлСниС 60 ΠΌ ΠΌΠ΅Π΄Π½ΠΎΠ³ΠΎ ΠΏΡ€ΠΎΠ²ΠΎΠ΄Π° Π΄ΠΈΠ°ΠΌΠ΅Ρ‚Ρ€ΠΎΠΌ 0,64 ΠΌΠΌ ΠΏΡ€ΠΈ Π½ΠΎΡ€ΠΌΠ°Π»ΡŒΠ½ΠΎΠΉ ΠΊΠΎΠΌΠ½Π°Ρ‚Π½ΠΎΠΉ Ρ‚Π΅ΠΌΠΏΠ΅Ρ€Π°Ρ‚ΡƒΡ€Π΅.

Π”ΠΎΠ±Π°Π²ΠΈΡ‚ΡŒ ΠΊΠΎΠΌΠΌΠ΅Π½Ρ‚Π°Ρ€ΠΈΠΉ

Π’Π°Ρˆ адрСс email Π½Π΅ Π±ΡƒΠ΄Π΅Ρ‚ ΠΎΠΏΡƒΠ±Π»ΠΈΠΊΠΎΠ²Π°Π½. ΠžΠ±ΡΠ·Π°Ρ‚Π΅Π»ΡŒΠ½Ρ‹Π΅ поля ΠΏΠΎΠΌΠ΅Ρ‡Π΅Π½Ρ‹ *